Các Dạng Bài Tập Thi Tnthpt 24 Gv

Download as docx, pdf, or txt
Download as docx, pdf, or txt
You are on page 1of 178

CÁC DẠNG BÀI TẬP THI TỐT NGHIỆP THPT – 2024

********
1. CULTURAL TIPS – CHỨC NĂNG GIAO TIẾP (câu 1&2)
Question 1: Bob and Khanh are chatting.
- Bob: “I think we should lead a green lifestyle.”
- Khanh: “____ It helps to protect the environment.”
A. Really? B. I don’t think so. C. Of course not. D. Sure.
Question 2: Giang is at Mary’s house.
- Mary: “Would you like something to drink, Giang?”
- Giang: “____”
A. Coffee, please. B. Yes, I’d like some bread.
C. Yes, I’m hungry. D. No, I prefer tea.

PRACTICE
EXERCISE 1
1. David is talking to Lucy about her painting.
- David: “What a beautiful painting!” - Lucy: “____________”
A. No problem B. It’s on the wall C. I’m glad you like it D. You’re welcome.
2. Peter and Dane are talking about environmental protection.
- Peter: “We should limit the use of plastic bags.”
- Dane: “____________. We can use paper bags instead.”
A. I completely agree. B. It’s not true. C. I don’t quite agree D. You’re wrong.
3. David is apologising to his teacher for being late.
- David: “Sorry I’m late! The traffic is so heavy.”- Teacher: “______. Come in and sit down.”
A. You’re so kind B. It’s alright C. Me neither D. Thank you
4. Peter and Mary are talking about social networks.
- Peter: “Using social networks may have negative effects on students.”
- Mary: “____________. It distracts them from their studies.”
A. I’m not sure about that B. I don’t quite agree
C. You’re wrong D. That’s quite true
5. Linda and Peter are talking about safe driving.
- Linda: “I think drink-driving should be severely punished.”
- Peter: “____________. It may cause accidents or even deaths.”
A. You must be kidding B. I don’t think so
C. I don’t understand what you mean D. I absolutely agree with you
6. A porter is talking to Mary in the hotel lobby.
- Porter: “May I help you with your suitcase?” - Mary: “____________”
A. What a shame B. Me too C. You’re welcome D. Yes, please
7. John is having dinner at Linda’s house.
- John: “This roast beef is so delicious.” - Linda:”____________”
A. sure. I’d love to B. I’m glad you like it.
C. No, don’t worry. D. I don’t either.
8. Joana and David, two lectures, are talking about library skills.
- Joana: “I think we should teach our students how to use the library.” - David: “________”
A. You’re absolutely wrong B. You must be kidding
C. I couldn’t agree with you more D. That’s not a good idea

1
9. A shop assistant is talking to a customer.
- Shop assistant: “Do you need anything else?” - Customer:”____________”
A. That’s all. Thanks B. Good job! C. With pleasure D. You’re welcome
10. Ann and Peter are talking about housework.
- Ann: “ I think children should be paid for doing the housework.”
- Peter: “____________. It’s their duty in the family.”
A. That’s what I think B. You’re exactly right
C. There’s no doubt about it D. I don’t think so
11. Ken and Tom are high-school students. They are discussing where their study group will meet.
- Ken: “Where is our study group going to meet next weekend?” - Tom: “____________.”
A. Studying in a group is great fun. B. We are too busy on weekdays.
C. Why don’t you look at the atlas? D. The library would be best.
12. Mike and Lane are university students. They are talking about Lane’s upcoming high-school
reunion.
- Mike: “So, you have your fifth high-school reunion coming up?”- Lane: “
A. Oh, the school reunion was wonderful. B. No. You’re in no mood for the event.
C. The food at the reunion was excellent. D. Yeah. I’m really looking forward to it.
13. A waiter in a restaurant is talking to a customer who has just finished his meal there.
- Waiter: “Here’s your bill, sir.” - Customer: “____________”
A. Don’t mention it. B. Can I pay by credit card?
C. What do you have? D. You’re welcome.
14. Two close friends Tom and Kyle are talking about Kyle’s upcoming birthday.
- Tom: “Can I bring a friend to your birthday party? - Kyle: “____________”
A. It’s my honour. B. Let’s do it then. C. The more the merrier. D. That’s right.
15. Two friends Diana and Anne are talking about Anne’s new blouse.
- Diana: “That blouse suits you perfectly, Anne.” - Anne: “____________’’.
A. Never mind. B. Don’t mention it. C. Thank you. D. You’re welcome.
16. Mary is talking to a porter in the hotel lobby.
- Porter: “Shall I help you with your suitcase?” - Mary: “____________”
A. Not a chance. B. That’s very kind of you.
C. I can’t agree more. D. What a pity!
17. Susan accidentally stepped on Denise’s foot.
- Susan: “Oops! I’m sorry, Denise.” - Denise: “____________”
A. You shouldn’t do that. B. It’s alright.
C. You are welcome. D. It’s nonsense.
18. Hana and Jenifer are talking about a book they have just read.
- Hana: “The book is really interesting and educational.” - Jenifer: “____________”
A. I’d love it. B. That’s nice of you to say so.
C. I couldn’t agree more. D. Don’t mention it.
19. Jolie and Tom are meeting at the supermarket.
- Jolie: “Hi, Tom. How are you doing?” - Tom: “________. How about you?”
A. I’m waiting for my sister B. I’m shopping for food
C. I’m doing nothing D. I’m doing well
20. Maria and Alex are talking about the environment.
- Maria: “Our environment is getting more and more polluted. Do you think so?”
- Alex: “____________. It’s really worrying.”
A. I’ll think about that B. I don’t agree C. I don’t think so D. I can’t agree more

2
EXERCISE 2.
Question 1. Mary is talking to John, who has just helped her with her homework.
- Mary: “Thanks a lot for your help.” - John: “________.”
A. My happiness B. My excitement C. My delight D. My pleasure
Question 2. Linda is thanking Peter for giving her a book that she’s been looking for.
- Linda: “Thank you for the book. I’ve been looking for it for months.” - Peter: “________.”
A. You can say that again B. Thank you for looking for it
C. I like reading books D. I’m glad you like it
Question 3. Minh is asking his mother about attending a Chinese language course.
Minh: “Do I have to take that Chinese language course?” Minh’s mother: “______.”
A. Thank you. B. Welcome to China. C. It’s up to you. D. No, you have to
Question 4. Trang is thanking Mai for visiting her.
- Trang: “Thank you very much for spending time with me.” - Mai: “________.”
A. It’s quite OK B. That’s right C. I’d love to D. You’re welcome
Question 5. John is talking to Tom about the result of the driving test.
- John: “I’ve passed my driving test.” - Tom: “________”
A. Congratulations! B. It’s nice of you to say so.
C. That’s a good idea. D. Good luck.
Question 6. John is talking to Tom about the final exam tomorrow.
- John: “I’ll have the final exam tomorrow.” - Tom: “________”
A. Good luck! B. It’s wonderful. C. That’s a good idea. D. You’re welcome.
Question 7. Peter is talking to Mary about her skin.
- Peter: “You should eat more fruits and vegetables.” - Mary: “________”
A. No problem. B. I don’t agree. C. Ok, thank you. D. It’s me.
Question 8. Minh is very bored with his studying and he tells Tom about this.
- Tom: “You had better work harder.” - Minh: “________”
A. I have worked hard B. I don’t agree. C. Yes, I will. Thanks. D. You are wrong.
Question 9. Peter and Mary are going across the road..
- Mary: “Peter, be careful! There's a car coming!” - Peter: “________”
A. Of course. B. Never mind. C. Ok. I will. D. Thanks. You make my day.
Question 10. Peter and Laura are walking in the park.
- Laura: "Hey, look out! The tree is falling down." - Peter: “________”
A. Thanks. B. Oh, how terrific. C. Poor you. D. Really?
Question 11. A manager is talking to his staff, Joe.
- Manager: “Joe, your sales are slow this month. You must get out there and hustle more!” - Joe:
“____” A. Don’t mention it B. That is a good idea.
C. Thank you, Sir. I will do it. D. I can’t agree with you more.
Question 12. A tourist is asking Minh for the way to the post office.
- Tourist: “Where is the nearest post office?” - Minh: “________”
A. Don’t worry. B. I’m afraid not. C. It’s over there. D. I don’t think so.
Question 13. An is asking Hoa for the way to the history museum.
- An: “Excuse me. Can you tell me how to get to the history museum?” - Hoa: “________”
A. Sure. It’s just 2 blocks from here. B. No, of course not.
C. I don’t think you can do this. D. Not at all. I love it.
Question 14. John is talking to Mary about keeping in touch with him.
- John: “Don’t forget to drop me a line when you settle down in London.” - Mary: “________”
A. I drop you a line when I settle down. B. I don’t. I’ll keep you in touch.
3
C. I will. I’ll not keep you in touch. D. I won’t. I’ll keep you posted.
Question 15. A customer is talking to a salesgirl at the clothes shop.
- Customer: "Can I try this skirt on?" - Salesgirl: “________”
A. No, the shop is closed in half an hour. B. Sorry, only cash is accepted here.
C. Yes, it costs one hundred and fifty dollars. D. Sure, the changing rooms are over there.
Question 16. Paul and Miranda are in the screening room.
Paul: “Let me adjust the seat for you!” Miranda: “________”
A. That would be lovely. Thank you. B. Yes, I would do it for you.
C. You must be kidding. D. No worries. I can seat here.
Question 17. Khanh is requesting her sister to help her with the chore.
Khanh: “I wonder if you could do the washing for me today?” Khanh’s sister: “______.”
A. No problem. B. Why do I have to?
C. Thank you so much. D. Yes, you do it now.
Question 18. Lauren and Thomas are studying in the library.
Lauren: “Do you need me to check your Math?” Thomas: “_______________”
A. Oh, that would be great. B. I’m trying to fix it.
C. What? Why? D. I can’t believe that.
Question 19. Joe and Paul are in the office.
Joe: “Would you like me to fill up your water glass?” Paul: “____________”
A. Sure, no problem. B. I wish you said that earlier.
C. Thanks but I can handle it. D. I’m sorry but I can’t.
Question 20. Mia meets Lana on her way home.
Mia: “I would be glad to give you a ride." Lana: “________”
A. Don’t worry. I’ll do it. B. Yes. I can help you.
C. I don’t think so. D. Thank you! That would be great.
Question 21. Sarah and Nathan are at a party.
Nathan: “Can I get you another piece of cake?” Sarah: “________”
A. It’s time to dance, isn’t it? B. I can’t take it.
C. Thanks but I’m full. D. My parents must yell at me.
Question 22. Carol and Jennifer are going to the cinema at weekends.
Carol: “Do you want me to pick you up?” Jennifer: “________”
A. You’re welcome. B. Sorry but I’m a bit busy.
C. If you don’t mind. D. I need to go now.
Question 23. Kate is sick when Jonathan visits her.
Jonathan: “I’ll go to the pharmacy for you.” Kate: “________”
A. You’ll love it. B. I think it’s ok.
C. That’s a good idea. D. Thank you but I’m fine.
Question 24. Mark and Barbara are in the office.
Mark: “I’ll do the photocopying if you like.” Barbara: “________”
A. Don’t worry. I’ll do it. B. Oh, let me think first.
C. The photocopier is broken. D. I’m fine.
Question 2 5. James meets Laura at the train station.
James: “Can I help you?” Laura: “________”
A. I can make it. No worries. B. Yes, please. I’d like to know what time the train leaves.
C. No way. I missed my train. D. I can find where my train is, finally.
Question 26. Both secretaries, Sarah and Michelle, are in the office.
Sarah: “Would you like me to answer the phone?” Michelle: “________”
4
A. Sure. If you don’t mind. B. Would you?
C. As your wish. D. I can’t answer the phone now.
Question 27. Steven is at the clinic.
Receptionist: “Would you like a magazine to read while you’re waiting?” - Steven: “_____”
A. I can’t wait for long. B. Please. I’m thirsty.
C. Yes please. I’d love one. D. I need an eye exam.
Question 28. Helen is at a clothes shop.
Assistant: “Good afternoon. How can I help you?” Helen: “________”
A. They fit fine. I’ll take them. B. I’m looking for the bigger size.
C. It looks ok, but I think I’ll leave it. D. No thanks. I’m just looking.
Question 29. William and Janet have just finished the meeting with their customers.
William: “Shall I bring your laptop back to the office?” Janet: “________”
A. Thank you. That would be great. B. Oh, I can help you.
C. No. I forgot my laptop. D. I can’t carry it.
Question 30. Tony is trying to repair his computer.
Edward: “Would you like me to help you with this?” Tony: “________”
A. It’s OK. I can do it myself. B. I’ll call you later.
C. I’m sorry for damaging your computer. D. I’ve just fixed it.

2. SO SÁNH HƠN – SO SÁNH NHẤT.


A. SO SÁNH HƠN. *Với tính từ ngắn: S1 + V + ADJ (ER) + THAN + S2
* Với tính từ dài: S1 + V + MORE + ADJ + THAN + S2

*Tip: - Đề có ______than  chọn …er/ more/ less.

Question 1:It is ....... in the city than it is in the country.


A. noisily B. more noisier C. noisier D. noisy
Question 2:The English test was ....... than I thought it would be.
A. the easier B. more easy C. easiest D. easier
Question 3: English is thought to be ....... than Math.
A. harder B. the more hard C. hardest D. the hardest
Question 4: My house is .......than hers.
A. cheap B. cheaper C. more cheap D. most cheaper
Question 5. Her office is ....... away than mine.
A. father B . more far C. farther D. farer
Question 6 Tom is ....... than David.
A. handsome B. the more handsome C. more handsome D. the most handsome
Question 8: He runs ....... than his brother.
A. fast B. faster C. more fast D. fastest
Question 10: My grandmother is ....... than my grandfather.
A. old B. older C. more old D. oldest
Question 12: The food at the new restaurant is ....... than the old one.
A. delicious B. more delicious C. the most delicious D. deliciouser
Question 13: She speaks French ....... than her sister.
A. fluently B. more fluently C. the most fluently D. fluentest
Question 14: This movie is ....... than the one we saw last week.
A. better B. good C. the best D. more good

5
Question 15: He is ....... about his work than his hobbies.
A. more serious B. serious C. the most serious D. seriouser
Question 16: The red dress is ....... than the blue one.
A. pretty B. more pretty C. the prettiest D. prettier
Question 17: This city is ....... than the one I visited last month.
A. big B. bigger C. the biggest D. more big
Question 18: She sings ....... than her friend.
A. beautifully B. more beautifully C. the most beautifully D. beautifuler
Question 19: His laptop is ....... than hers.
A. expensive B. the most expensive C. expensiver D. more expensive
Question 20: The cake was ....... than I expected.
A. sweet B. sweeter C. the sweetest D. more sweet

B. SO SÁNH NHẤT. * Với tính từ ngắn: S + V + THE + est …


* Với tính từ dài: S + V + the most ….
Question 3: Hong is student in her class.
A. the most tall B. the tallest C. the more tall D. the taller
*Tip: - Đề có ______ among/ of/ in/ on/ ever  chọn the -est/ the most

Exercise: Choose the best answer to complete the following sentences.


Question 1: I think spring is _________ season of the year.
A. beautiful B. very beautiful C. more beautiful D. the most beautiful
Question 2: “Why did you buy these oranges? ” “They were ____________ I could find.”
A. cheapest B. cheapest ones C. the cheapest ones D. the most cheapest
Question 3: I thought this test would be difficult, but it is ________ test I’ve ever taken.
A. the easiest B. easier C. the most easy D. easily
Question 4: China is the country with ____________ population.
A. the larger B. the more large C. the largest D. the most large
Question5: The use of detail is ______________ method of developing a controlling idea and
almost all students employ this method.
A. more common B. common C. most common D. the most common
Question 6: What is the __________ movie you have ever seen?
A. worst B. baddest C. most bad D. the worst
Question 7: The Missouri River is the __________ river in the United States.
A. most long B. longer C. longest D. the longer
Question 8: Our crazy Uncle Pete is the __________ fisherman in all of Oregon.
A. talented B. most talented C. more talented D. talentedly
Question 9: I’m positive that I am __________and beautiful than my opponent.
A. more intelligent B. most intelligent C. as intelligent D. intelligent
Question 10: Rare animal extinction is one of the ________ things that could have happened.
A. good B. best C. worse D. worst
Question 11: It has been a really bad journey, but I think the ____ is behind us.
A. good B. better C. worst D. worse
Question 12: Normally, in order for a political party to form a government, it has to have
____________ number of seats in Parliament.
A. the largest B. as large as C. larger than D. so large

6
Question 13: Ozone depletion has been _____________ at the poles, especially over Antarctica,
where a seasonal ozone layer "hole’ appears.
A. as severe as B. so severe as C. more severe than D. the most severe
Question 14: Of the three courses I did, this one is ________________ interesting.
A. the hardest and more B. harder but more
C. the harder but more D. the hardest but most
Question 15: The __________ source of power in 1970 was from natural gas with almost 25%.
A. most second important B. second most important
C. second important most D. most important second
Question 16: This is _______________ wedding party I’ve ever attended.
A. the more memorable B. more memorable
C. the most memorable D. most memorable
Q. 17: Chocolates and flowers are by far __presents for mothers on Mother’s Day in the UK.
A. more popular B. the more popular C. less popular D. the most popular
Question 18: Chuseok is one of __________ celebrations in the Korean calendar when Koreans
give thanks to nature.
A. the most important B. the more important C. the less important D. more important
Question 19: June has become __________ month for weddings in many countries when the
symbolic flowers, roses, usually bloom.
A. more popular B. less popular C. the most popular D. the least popular
Question 20: Before doing something important, Vietnamese people always try to choose
__________ time for it.
A. better B. less good C. the least good D. the best
Question 21: Phil is ____________ person we have ever known.
A. the happier B. the happiest C. happier D. happiest
Question 22.Summer is _______ season of the year.
A. hottest B. the hottest C. the hotter D. the more hottest
Question 23.It is one of _______ books I have ever had.
A. most useful B. the most useful
C. the most useful than D. the most useful as
Question 24.Of all athletes, Alex is _______.
A. the less qualified B. the less and less qualified
C. the more qualified D. the least qualified
Question 25.Among the students in my class, Peter is _______
A. most active B. the most active C. the more active D. more active
Question 26. He was _______ interested in science.
A. the little B. the least C. the less D. least
Question 27._______ planet to see in the night sky is Venus.
A. The easy B. The easiest C. The more easy D. The easier
Question 28. Of the three courses I did, this one is……………..interesting.
A. the hardest and more B. harder but more
C. the harder but more D. the hardest but most
Question 29. Potential dehydration is……………..that a land animal faces.
A. the often greatest hazard B. the greatest often hazard
C. often the greatest hazard D. often the hazard greatest
Question 30. She sings ……….. among the singers I have known.
A. the most beautiful B. the more beautiful

7
C. the most beautifully D. the more beautifully
Question 31. She is ....... student in my class.
A. most hard-working B. more hard-working
C. the most hard-working D. as hard-working

3. WORD FORM – TỪ LOẠI


* Tên các từ loại: Danh từ, Tính từ Trạng từ, Động từ.
* Thứ tự trong câu: Tính – danh – động - trạng, trạng - tính, trạng - trạng
Question 4: The school’s going to have a swimming ____ for the first-year students.
A. competition B. compete C. competitively D. competitive
(Câu này dựa vào chữ “have a” => có 1, cần đi với Danh từ. Swimming ko phải d.từ,
(đóng vai trò là tính từ), nên ta chọn competition)

* Trật tự từ
STT Quy tắc Ví dụ
1 Tobe (am/is/are/was/were) + ADJ The book is so interesting that I can’t
put it down.
She looks happier than yesterday.
2 V + Adv He runs more quickly than me.
3 - a/an/the + (adj) + N The development of industry causes air
- my/your/our/his/her/their/its + (adj) + N pollution.
- this/that/these/those) + (adj) + N He failed the exam because of his

8
- some/any/many/much + (adj) + N laziness.
- Cụm N1 + in/on/of/… + Cụm N2 This machine has been out of order.
There are many people waiting for the
last bus.
4 Khi có “and/or/but” thì hai vế cân nhau The International Red Cross helps people
(cùng chức năng từ loại/ngữ pháp/ngữ in need without any discrimination based
nghĩa) on nationality, race, religion, class or
political opinions.
PRACTICE 1
1. He has been very interested in doing research on _______ since he was at high school.
a. biology b. biological c. biologist d. biologically
2. Are you sure that boys are more _______ than girls?
a. act b. active c. action d. activity
3. You are old enough to take _______ for what you have done.
a. responsible b. responsibility c. responsibly d. irresponsible
4. These quick and easy _______ can be effective in the short term, but they have a cost.
a. solve b. solvable c. solutions d. solvability
5. He was looking at his parents _______, waiting for recommendations.
a. obey b. obedience c. obedient d. obediently
6. John cannot make a _______ to get married to Mary or stay single until he can afford a
house and a car.
a. decide b. decision c. decisive d. decisively
7. . It was such a boring speech that I fell _________ .
A. asleep B. sleep C. sleepy D. sleepily
8. The speaker said nothing worth listening to.
A. complete B. completing C. completely D. completion
9. My mother used to be a woman of great _______, but now she gets old and looks pale.
a. beauty b. beautiful c. beautifully d. beautify
10. Although they are twins, they have almost the same appearance but they are seldom in ___.
a. agree b. agreeable c. agreement d. agreeably
11. The more _______ and positive you look, the better you will feel.
a. confide b. confident c. confidently d. confidence
12. My parents will have celebrated 30 years of _______ by next week.
a. marry b. married c. marriageable d. marriage
13. Many Vietnamese people ______ their lives for the revolutionary cause of the nation.
a. sacrifice b. sacrificed c. sacrificial d. sacrificially
14. Most of us would maintain that physical ______ does not play a major part in how we react
to the people we meet.
a. attract b. attractive c. attractiveness d. attractively
15. They had a ______ candlelit dinner last night and she accepted his proposal of marriage.
a. romance b. romantic c. romantically d. romanticize
16. They started, as _______ gatherings but they have become increasingly formalized in the
last few years.
a. informal b. informally c. informalize d. informality
17. Children who are isolated and lonely seem to have poor language and ________.
a. communicate b. communication c. communicative d. communicator

9
18. The lecturer explained the problem very clearly and is always _______ in response to
questions.
a. attention b. attentive c. attentively d. attentiveness
19. He is not really friendly and makes no attempt to be _______.
a. society b. sociably c. socialize d. sociable
20. Smoke, dirt and noise are all kinds of ___.
A. pollute B. polluting C. pollutant D. pollution
21. English is now an effective medium of international ___.
A. communication B. commute C. communicative D. communicate
22. I liked the book till the end but it had a (n) ending which I didn’t like.
A. bored B. boring C. boredom D. boringly
23. She sent me a _______ letter thanking me for my invitation.
a. polite b. politely c. politeness d. impoliteness
24. Mr. Timpson's behavior and comments on occasions were inappropriate and fell below the
_______ standards.
a. accept b. acceptable c. acceptance d. accepting
25. Many teenagers show signs of anxiety and _______ when being asked about their future.
a. depress b. depression c. depressing d. depressed
26. As an _______, Mr. Pike is very worried about the increasing of juvenile delinquency.
a. educate b. education c. educator d. educative
27. The United Nations Secretary-General has often spoken of the need for individual _______
and human rights in his speeches.
a. free b. freedom c. freely d. freeing
28. He did not do well at school and left with few _______ qualifications.
a. academic b. academy c. academician d. academically
29. The Minister of the Education and Training Department appeared on TV last night to __ his
new policy.
a. public b. publicly c. publicize d. publicizing
30. He owed his success not to privilege but to self-education and a driving desire for _______.
a. achieve b. achiever c. achievement d. achievable
31. Despite many recent _______ advances, there are parts where schools are not equipped with
computers.
a. technology b. technological c. technologically d. technologist
32. I am so _______ that I cannot say anything, but keep silent.
a. nerve b. nervous c. nervously d. nervousness
33. Many people have objected to the use of animals in _____ experiments.
a. science b. scientist c. scientific d. scientifically
34. Thank you for _____ me
A. invite B. inviting C. invited D. invitation
35. My ______ pastime is going dancing with my friends.
A. favourite B. favoured C. favour D. favourable
PRACTICE 2
Question 1.Many librarians and teachers are now accepting graphic novels as proper literature for
children as they________young people and motivate them to read.
A. attract B. attractively C. attraction D. attractive
Question 2.People tend to be unaware of the messages they are sending to others. So, it
is________ to consider your own body language before dealing with people from other cultures.

10
A. usefully B. use C. usefulness D. useful
Question 3.Candidates are advised to dress formally to make a good________on job interviewers.
A. impress B. impressively C. impression D. impressive
Question 4.The new song has ________been selected for Euro 2016.
A. officially B. official C. office D. officer
Question 5.Many species of plants and animals are in________ of extinction.
A. dangerous B. endangered C. danger D. dangerously
Question 6.With his good sense of humour, Martin is quite________ with the students.
A. popularly B. popular C. popularise D. popularity
Question 7.Many Vietnamese people ______ their lives for the revolutionary cause of the nation
A. sacrifice B. sacrificed C. sacrificial D. sacrificially
Question 8.Most of us would maintain that physical ______ does not play a major part in how we
react to the people we meet.
A. attract B. attractive C. attractiveness D. attractively
Question 9.This book is not really ________.It is a waste of money buying it.
A. inform B. information C. informative D. informatively
Question 10.The wedding day was________chosen by the parents of the groom.
A. careless B. careful C. carefully D. carelessly
Question 11.To Americans, it is impolite to ask someone about age, ________and salary.
A. many B. married C. marrying D. marriage
Question 12.In the past people believed that women’s roles as mothers and wives are ________.
A. nature B. natural C. naturism D. naturalist
Question 13.How many means of________do you use on a regular basis?
A. communication B. communicating C. communicator D. communicative
Question 14.The girl seems to be more ________
A. confine B. confident C. confidently D. confidence
Question 15.I think mobile phones are________for people of all ages.
A. usage B. use C. usefully D. useful
Question 16.Swimming produces both________and physical benefits.
A. psychology B. psychological C. psychologist D. psychologically
Question 17.It is not always easy to make a good________at the last minute.
A. decide B. decision C. decisive D. decisively
Question 18.Cultural ______ can be found in such simple processes as giving or receiving a gift.
A. differences B. different C. differently D. Differ
Question 19.The job market is constantly changing due to innovative technology and
new________.
A. competition B. competitively C. competitive D. compete
Question 20.I think mobile phones are______for people of all ages.
A. usage B. use C. usefully D. useful
Question 21.Almost half of turtles and tortoises are known to be threatened with ____.
A. extinct B. extinction C. extinctive D. extinctly
Question 22.Current extinction rates are at least 100 to 1,000 times higher than _______ rates
found in the fossil record.
A. nature B. natural C. naturally D. naturalness
Question 23.Not many people find reading more ________ than watching TV.
A. interest B. interested C. interesting D. interestingly
Question 24.To become a novelist,you need to be ________.

11
A. imagine B. imagination C. imaginative D. imaginarily
Question 25.They are going to _______ the pool to 1.8 meter.
A. deep B. depth C. deepen D. deeply
Question 26.Life on earth would come to an end if the ___ were destroyed.
A. environment B. environmental
C. environmentally D. environmentalist
Question 27.She sent me a _______ letter thanking me for my invitation.
A. polite B. politely C. politeness D. impoliteness
Question 28. Mr.Timpson's behavior and comments on occasions were inappropriate and fell
below the _______ standards.
A. accept B. acceptable C. acceptance D. accepting
Question 29.The interviewer gave his consent to John's _______ for work and promised to give
him a job.
A. keen B. keenly C. keener D. Keenness
Question 30. It is _______ to fail a job interview, but try again.
A. disappoint B. disappointing C. disappointedly D. disappointment
Question 31.It is often a good idea to start with small, easily _______ goals.
A. achieve B. achievement C. achievable D. achiever
Question 32.Henry was a studious student. He needed no _______ to work hard.
A. encourage B. encouraging C. encouragement D. encouraged
Question 33.During the time of economic reforms, the economy has grown _______ with only a
few major setbacks.
A. constant B. constantly C. constants D. constancy
Question 34.Many teenagers show signs of anxiety and____ when being asked about their future.
A. depress B. depression C. depressing D. depressed
Question 35.Constant _______ of attack makes everyday life dangerous here.
A. threat B. threaten C. threatening D. threateningly
Question 36.No one can predict the future exactly. Things may happen _______.
A. expected B. unexpected C. expectedly D. unexpectedly
Question 37.Can you make a ________ estimate of how much our expedition in the desert may
be? A. rough B. roughly C. roughing D. roughen
Q. 38.After Peter had returned from the Sahara desert, he was confined to bed by a ______ disease.
A. mystery B. mysterious C. mysteriously D. mysteriousness
Question 39.Don't read that book. It's _____.
A. bore B. boredom C. boring D. bored
Question 40.English is the most________ used language in the world.
A. wider B. widest C. width D. widely
Question 41.He cycled and had an accident.
A. careful B. careless C. carefully D. carelessly
Question 42.Sometimes it is ________ to find suitable books for our children.
A. difficult B. difficulty C. difficultly D. difficulties
Q. 43.A book may be evaluated by a reader or professional writer to create a book ________.
A. review B. reviewing C. reviewer D. reviewed
Question 44.Read the book ________ and you can find the information you need.
A. care B. careful C. carefulness D. carefully
Question 45.To become a novelist, you need to be ________.
A. imagine B. imagination C. imaginative D. imaginarily

12
Question 46.They are going to _______ the pool to 1.8 meter.
A. deep B. depth C. deepen D. deeply
Question 47.The referee had no hesitation in awarding the visiting team a _______.
A. penalty B. penalize C. penal D. penalization
Question 48. Not many places at the universities are left, so choice is on a severe _______.
A. limiting B. limitation C. delimitation D. limited
Question 49.I am so _______ that I cannot say anything, but keep silent.
A. nerve B. nervous C. nervously D. nervousness
Question 50._______ speaking, I do not really like my present job.
A. Honest B. Honesty C. Honestly D. Dishonest

4. Rút gọn mệnh đề quan hệ bằng cách dùng quá khứ phân từ V3/ed
(Past participle phrases) hoặc hiện tại phân từ - V-ing (Present participle phrase)

=> Form: Noun, Ving/ V3, ed……. V chính….

* Question 5: Air pollution has become serious, ____ health problems for many people.
A. is caused B. to cause C. causing D. caused
(Causing = which causes…., mang nghĩa chủ động, đồng thời tiếp nối ý của cả mệnh đề
phía trước)

PRACTICE 1. Rút gọn mệnh đề quan hệ bằng cách dùng quá khứ phân từ - V3/ed
Question 1:The building ______ by the architect won an award for its unique design.
A. designs B. designing C. designed D. is designing
Question 2:The song ______ by the musician topped the charts for several weeks.
A. composed B. composing C. composes D. is composing
Question 3:The website ______ by the web developer is user-friendly and easy to navigate.
A. develops B. developing C. developed D. is developing
Question 4:The movie ______ by the producer will be released next month.
A. produces B. produced C. producing D. is producing
Question 5:The cake ______ by the baker looks delicious and smells amazing.
A. bakes B. baking C. bake D. baked
Question 6:The report ______ by the analyst provides insights into the company's financial
performance.
A. analyzes B. analyzed C. analyzing D. is analyzing
Question 7:The painting ______ by the artist captures the beauty of the landscape.
A. paints B. painting C. painted D. is painting
Question 8:The experiment ______ by the scientist yielded unexpected results.
A. conducted B. is conducting C. conducting D. be conducted
Question 9:The play ______ by the playwright received rave reviews from the critics.
A. writes B. writing C. which written D. written
Question 10:The dish ______ by the chef is a fusion of different cuisines and flavors.
A. cooks B. cooking C. cooked D. is cooking
Question 11:The charity fundraiser, _____ by a local non-profit, raised thousands of dollars for a
good cause. A. held B. were held C. holding D. were holding

13
Question 12:The annual music festival, _____ in Chicago this year, attracts music lovers from all
over the country. A. held B. were held C. holding D. hold
Question 13:The speech __________ by the politician was widely televised and watched by
millions of people.
A. gives B. giving C. given D. is giving
Question 14:The dish ________ by the chef was absolutely delicious and earned a Michelin star.
A. cooked B. cooks C. cooking D. is cooking
Question 15:The sculpture __________ by the artist was inspired by nature and received critical
acclaim.
A. sculpts B. sculpted C. sculpting D. is sculpting
Question 16:The book __________ by the author has been adapted into a successful movie.
A. writes B. writing C. written D. is writing
Question 17: The website __________ by the designer is visually stunning and easy to use.
A. designs B. designing C. was designed D. designed
Question 18.The song __________ by the singer was an instant hit and topped the charts for
months.
A. sings B. singing C. sung D. is singing
Question 19: The project __________ by the team was completed ahead of schedule and under
budget.
A. is managing B. managed C. managing D. managered
Question 20: The car __________ by the mechanic runs smoothly and efficiently.
A. repairs B. repairing C. repaired D. is repairing

(Further practice)
Rút gọn mệnh đề quan hệ bằng cách dùng V3/ed hoặc Gerund (Ving).
Choose the best option.
1. Do you know the woman who is coming towards us?
A. comes B. coming C. came D. come
2. I come from a city that is located in the southern part of the country.
A. located B. locating C. being locate D. locate
3. The children who attend that school receive a good education.
A. attend B. attended C. attending D. being attended
4. The fence which surrounds our house is made of wood.
A. surrounds B. surround C. surrounded D. surrounding
5. Be sure to follow the instructions that are given at the top of the page.
A. gives B. gave C. given D. giving
6. They live in the house that was built in 1890.
A. build B. built C. building D. builded
7. The man who is talking to the policeman is my uncle.
A. talking B. talked C. talk D. talks
8. The number of students who have been counted is quite high.
A. to count B. counts C. counting D. counted
9. George is the man who was chosen to represent the committee at the convention.
A. choosing B. to be chose C. chose D. chosen
10. The man who is standing there is a clown.
A. to be stood B. stood C. standing D. stand
11. The envelop which lies on the table has no stamp on it.

14
A. lies B. lying C. to lie D. lay
12. The student don't know how to do exercise which were given by the teacher yesterday.
A. given B. giving C. gave D. be giving
13. The diagrams which were made by young Faraday were sent to Sir Humphry Davy at the end
of 1812.
A. makes B. making C. made D. make
14. The gentleman who lives next door to me is a well-known orator.
A. living B. lived C. lives D. be lived
15. All the astronauts who are orbiting the earth in space capsules are weightless.
A. orbited B. orbiting C. orbit D. being orbited
16. I saw many houses that were destroyed by the storm.
A. to destroy B. destroying C. destroyed D. destroy
Bài tập từ các đề minh họa
1. All students _______ in the discussion will be given a present.
A. participated B. participate C. participating D. to participate
2. A quilt that looks ordinary _______ may become a work of abstract art when it is hung on a
white wall.
A. to be lying in bed B. to be on bed C. lying on a bed D. lies on a bed
3. The British Council has announced the postponement of all IELTS tests _______ to be held in
Vietnam from Thursday.
A. which authorize B. to authorize C. authorized D. authorizing
4.The letter _______ me of the theft hadn't been signed.
A. to accuse B. that accuses C. accused D. accusing
5. I wanted to take a photo of the man _______ to one of the guests near the window.
A. have talked B. talked C. talking D. was talking
6. People _______ smartphones have access to a wealth of information and entertainment at their
fingertips.
A. owning B. are owned C. are owning D. owned
7. Mr. Robert, _______ from the company, is now facing legal charges.
A. fired B. was fired C. was fired D. firing
8. The earthquake, _______ in Turkey, destroyed many high buildings.
A. was happening B. happening C. happened D. was happened
9. The new Town Hall, _______ in 2021, is expected to receive several awards.
A. building B. built C. was building D. was built
10. Ninh Binh, _______ 90 kilometers southeast of Hanoi, has been ranked seventh in the most
welcoming regions at the 11th annual Traveller Review Awards of travel website Booking.com this
year.
A. is located B. locating C. is locating D. located
11. Scientists _______ on the site found many ancient tools, tombs and pottery, and made some
important.
A. working B. worked C. to work D. to be working
12. They stood on the bridge _______ the two small towns of London.
A. connects B. was connecting C. connected D. connecting
13. The young man _______ after the court was found innocent of all the charges against him.
A. was released B. was releasing C. releasing D. released
14. Who owns the car _______ outside?
A. is parking B. parked C. parking D. is parked

15
15. The window _______ in the storm last night is being repaired now.
A. was breaking B. was broken C. breaking D. broken
16. The star apple tree _______ by Uncle Ho many years ago is still flourishing well now.
A. was planted B. planted C. to plant D. planting
31. The recipe _______ by the chef is a closely guarded secret and has made them famous.
A. created B. creates C. creating D. is creating
17. Videos of celebrities _______ to Vietnamese pop star Hoang ThuyLinh's "See Tinh" (See
Love) have gone viral in South Korea and China and many other countries.
A. to dance B. dance C. danced D. dancing
18. The speech _______ by the politician was well-received by the audience and made headlines.
A. delivered B. was delivered C. delivering D. was delivering
19. Many of the items such as rings, key chains, or souvenirs are made of unusual shells or unique
stones illegally _______ from the protected areas.
A. is removing B. is removed C. removing D. removed
20. The little girl _______ with my son in the living room is my brother's daughter.
A. was played B. to play C. was playing D. playing
21. The road _______ badly in the flood last month has been repaired.
A. was damaging B. damage C. damaged D. was damaged
22. Ngoc has been actively participating in some environmental campaigns _______ by the Youth
Club.
A. launch B. been launched C. which launch D. launched
23. The ideas _______ in this essay are not particularly original.
A. are discussed B. discussed C. which are discussing D. discussing
24. Fifty years ago, many people _______ from tuberculosis eventually died.
A. to suffer B. who suffering C. suffered D. suffering
25. The earthquake, ___ Turkey a few weeks ago, caused irreparable damage to some parts of this
country.
A. hit B. was hit C. hitting D. to hit
26. The new policy will be helpful for people _______ calls from unverified mobile phone
numbers.
A. received B. to receive C. receive D. receiving
27. The Indians _______ in Peru before the discovery of America by Europeans belonged to the
Inca culture.
A. lived B. living C. were living D. had lived
28. Science fiction is a kind of books or films _______ space travel or life on other planets, based
on imagined scientific discoveries of the future.
A. showing B. which showing C. to show D. to showing
29. The scientists _______ on this project try to find the solutions to air pollution.
A. working B. worked C. are working D. who working
30. The phone system _______ last Friday seems to work very smoothly.
A. to replace B. replacing C. replaced D. were replaced
31A range of services _______ are introduced to meet the specific industry and market dynamics
the clients face.
A. carefully designing B. having designed carefully
C. were carefully designed D. carefully designed
32. ] Many of the pictures _______ from outer space are presently on display in the public library.
A. sent B. sending C. that sent D. were sent

16
33. The relics _______ in the area are intact.
A. excavated B. are excavating C. excavating D. are excavated
35. The movie is set in an island _______ off the west coast.
A. was locating B. is located C. locating D. located
35. The museum _______ a collection of priceless artifacts was a must-see for history buffs.
A. housed B. was housed C. was housing D. housing
36. ] The proposal _______ by the environmentalists to grow more trees has received approval
from the council.
A. was suggested B. which suggested C. suggesting D. suggested
37. Do you know the boy _______ the windows last night?
A. to break B. breaking C. broken D. having broken
38. Psychologists strongly advise parents with small kids not to allow them to watch videos
_______ sensitive content.
A. contain B. containing C. are contained D. contained

5. ARTICLES (MẠO TỪ) A-AN-THE

Question 6: In ____ evening, we sometimes eat out with our friends.


A. an B. the C. Ø (no article) D. a
*Những trường hợp không dùng mạo từ:
 Trước tên của các môn học. Ex: We are studying English with Ms. Tam
 Trước tên của các môn thể thao. Ex: I can play badminton.
 Trước danh từ số nhiều không xác định. Ex: Jill loves cats.
 Trước danh từ không đếm được. Ex: I am drinking water.
 Trước tên của các phương tiện giao thông, nhưng phải đứng sau giới từ "by".
 Trước danh từ chỉ màu sắc. Ex: That car is red.
 Sau tính từ sở hữu hoặc sau danh từ ở sở hữu cách. Ex: It’s Tan’s book.
 Trước tên của các bữa ăn. Ex: I often have fried rice for breakfast.
 Trước thứ, ngày, tháng, năm, mùa (nếu không xác định). Ex: Today is Sunday.
 Trước tên quốc gia (trừ các quốc gia đã được liệt kê ở mục 2), tên châu lục, tên
núi, tên hồ, tên đường. Ex: We live in Vinh Long.
 Trước danh từ trừu tượng. Ex: Love makes us blind. 

PRACTICE 1
Question 1. _____ book you lent me last week is so interesting that I have read it twice.
A. The B. A C. An D: Ø (no article)
Question 2._____ apple a day keeps the doctor away.
A. An B. The C. A D: Ø (no article
Question 3. He is wearing _____ old jacket.
A. A B. An C. The D. Ø (no article)
Question 4. I need to buy _____ new phone.
A. A B. An C. The D. Ø (no article)

17
Question 5. Have you seen _____ latest episode of your favorite TV show?
A. A B. An C. The D. Ø (no article)
Question 6_________Water boils at 100oC
A. An B. The C. a D. Ø (no article)
Question 7. I saw _____ beautiful flower on my walk this morning.
A. A B. An C. The D. Ø (no article)
Question 8. She is reading _____ interesting book about space exploration.
A. A B. An C. The D. Ø (no article)
Question 9. They came here by________train.
A. A B. An C. The D. Ø (no article)
Question 10. I try my best to get the scholarship to study in ________USA.
A. A B. An C. The D. Ø (no article)
Question 11. Can you pass me _____ salt, please?
A. the B. A C. Both A and B D. Ø (no article)
Question 12. We went to _____ park and had a picnic.
A. A B. An C. The D. Ø (no article)
Question 13. She's allergic to _____ shellfish, so she can't eat sushi.
A. A B. An C. The D: Ø (no article)
Question 14. I saw _____ amazing movie last night.
A. A B. An C. The D. Ø (no article)
Question 15. They live in _____ old house in the city center.
A. A B. An C. The D. Ø (no article)
Question 16. Could you turn off _____ lights when you leave the room?
A. A B. An C. The D. Ø (no article)
Question 17. We need _____ eggs for the cake recipe.
A. A B. An C. The D. Ø (no article)
Question 18. She wants to learn _____ new language.
A. A B. An C. The D. Ø (no article)
Question 19. He's drinking _____ cup of coffee before work.
A. A B. An C. The D. Ø (no article)
Question 20._____ sun sets in the west.
A. An B. The C.an D. Ø (no article)
Question 21. He's listening to _____ music while he works.
A. A B. An C. The D. Ø (no article)
Question 22. She's wearing _____ red dress to the party tonight.
A. the B. an C. a D. Ø (no article)
Question 23. The girl in __________blue dress is my cousin.
A. The B. A C. An D. Ø (no article)
Question 24. I need to buy _____ new shoes for work.
A. A B. An C. The D.Ø (no article)
Question 25. He's listening to _____ old song on the radio.
A. A B. An C. The D. Ø (no article)
Question 26. _____ computer is very slow today.
A. The B. A C. An D. Ø (no article)
Question 27. She's learning to play _____ guitar.
A. A B. An C. The D. Ø (no article)
Question 28. Could you pass me _____ orange juice, please?

18
A. A B. An C. The D. Ø (no article)
Question 29.We saw _____ beautiful flowers in the garden.
A. A B. An C. The D. Ø (no article)
Question 30 . _____ moon is visible in the night sky.
A. X B. A C. The D. A
Question 31. She's reading _____ newspaper.
A. A B. An C. The D. Ø (no article)
Question 32. I saw _____ interesting documentary on TV last night.
A. A B. An C. The D. Ø (no article)
Question 33. They went on _____ excursion to Europe last summer.
A. A B. An C. The D. Ø (no article)
Question 34. _____ elephant at the zoo was eating a watermelon.
A. A B. An C. The D. Ø (no article)
Question 36. We had _____ amazing time at the beach yesterday.
A. A B. An C. The D. Ø (no article)
Question 37. She's allergic to peanuts, so she can't eat _____ peanut butter.
A. A B. An C. The D.Ø (no article)
Question 38. If I was/ were _________bird, I would be _______white dove. 
A. a/a B. the/the C. a/the D. X/ X
Question 39. We went to ____________school six days a week.
A. A B. An C. The D. Ø (no article)
Question 40. He's wearing _____ green shirt today.
A. A B. An C. The D. Ø (no article)
Question 41. She's drinking _____ cup of tea to relax.
A. A B. An C. The D. Ø (no article)
Question 42.I need to buy _____ new computer for my job.
A. A B. An C. The D. Ø (no article)
Question 43._____ sun is very hot in the summer.
A. An B. The C. A D. Ø (no article)
Question 44. She's studying _____ French at school.
A. A B. An C. The D. Ø (no article)
Question 45. He's driving _____ red sports car on the highway.
A. A B. An C. The D. Ø (no article)
Question 46. How far is it from your house to __________ school?.
A. An B. a C. Both A and B D. Ø (no article)
Question 47. She's eating _____ apple for a snack.
A. A B. An C. The D. Ø (no article)
Question 48. I saw _____ shooting star in the sky last night.
A. A B. An C. The D. Ø (no article)
Practice 2.
1.There are billions of stars in _____ space.
A. a B. an C. Ø D. the
2.He tried to park his car but _____ space wasn’t big enough.
A. the B. a C. an D. Ø
3.Can you turn off _____ television, please?
A. Ø B. a C. an D. the
4.We had _____ meal in a restaurant.

19
A. a B. Ø C. the D. an
5.Thank you. That was ______ very nice lunch.
A. a B. an C. the D. Ø
6.Jill went to ______ hospital to see her friend.
A. Ø B. the C. a D. an
7.Mrs Lan went to ______ school to teach English lessons.
A. Ø B. the C. a D. an
8.We visited _______ two years ago.
A. Canada and the United States B. the Canada and the United States
C. the Canada and United States D. Canada and United States
9.Are you going away next week? No,_______ week after next.
A. a B. the C. some D. Ø
10.We haven’t been to ______ for years.
A. cinema B. the cinema C. a cinema D. any cinema
11.It took us quite a long time to get here. It was ______ journey.
A. three hour B. the three- hours C. a three- hour D. three- hours
12.I can’t work here. There’s so much ______.
A. noise B. noises C. the noise D. a noise
13.I’ve seen ______ good film recently.
A. Ø B. the C. a D. an
14.I often watch ______ television for two hours every night.
A. some B. the C. any D. Ø
15.The injured man was taken to _____ hospital
A. Ø B. a C. the D. an
16.She went out without _____ money.
A. any B. an C. a D. Ø
17.Did _______ police find ______ person who stole your bicycle?
A. a /a B. the / the C. a / the D. the / a
18.Can anyone give me_________hand, please because I have just fallen over?
A. a B. an C. the D. Ø
19.I don’t know what to do. It’s ______________problem.
A. quite difficult a B. a quite difficult C. quite a difficult D. the quite difficult
20. I have left my book in ____________.kitchen and I would like you to get it for me.
A. a B. an C. the D. Ø

6. PHRASAL VERBS (CỤM ĐỘNG TỪ)

Câu trong đề MH
Question 7: His health has improved a lot since he ____ smoking.
A. put on B. got up C. gave up D. took off
(sức khỏe ông ấy khá hơn nhiều kể từ khi ông ta: mặc vào/ thức dậy/ từ bỏ/ tháo ra…thuốc
lá => chọn “gave up”)

20
 Hãy làm bài tập để thấy nó rất….mênh mông 

PRACTICE 1
Mark the letter A, B, C, or D on your answer sheet to indicate the correct answer to each of the
following questions.
1. She decided to ______ the meeting because she wasn't feeling well.
A. put out B. put off C. put away D. put on
2. I need to ______ on my assignments this weekend.
A. catch up B. catch on C. catch out D. catch in
21
3. The manager promised to ______ the issue and get back to us.
A. look through B. look up C. look into D. look out
4. He ______ in tears when he heard the news.
A. broke into B. broke off C. broke down D. broke up
5. We need to______ the mess before the guests arrive.
A. clean off B. clean up C. clean out D. clean away
6. Don't forget to ______ some groceries on your way home.
A. pick in B. pick off C. pick out D. pick up
7. I accidentally ______ an old friend at the mall yesterday.
A. ran up B. ran over C. ran into D. ran out
8. She always ______ late for class.
A. shows off B. shows out C. shows in D. shows up
9. The company plans to _____ a survey to gather feedback from customers.
A. carry on B. carry out C. carry off D. carry away
10. He needs to ______ his fear of public speaking if he wants to succeed in his career.
A. get in B. get over C. get off D. get by
11. We should ______ the large task into smaller ones to make it more manageable.
A. break off B. break up C. break through D. break in
12. Can you ______ the worksheets to the students, please?
A. hand off B. hand down C. hand in D. hand out
13. She needs to ______ on caffeine because it's affecting her sleep.
A. cut down B. cut off C. cut out D. cut in
14. Let's take ______ the situation before we make a final decision.
A. up with B. up on C. stock of D. out of
15. Being self-reliant is what many young people ______.
A. take care of B. strive for C. cope with D. figure out
16. Her explanation of the phenomenon was so confusing that I couldn't ______ anything.
A. take in B. put back C. call for D. go over
17. My grandfather died because of the war. However, his great spirit was __ for all generations.
A. given down B. handed down C. hold down D. went down
18. I could not ______ the lecture at all. It was too difficult for me.
A. hold on B. make off C. get along D. take in
19. They ______ an old cottage in the Scottish Highlands because it was in bad condition.
A. did up B. filled up C. set up D. turn up
20. Please help me with this math problem. I can’t ______ the answer.
A. end up B. face up to C. come up with D. run into
21. Such approaches should be supported and mainstreamed in health interventions in order to
______ positive behavior change.
A. put off B. set off C. bring about D. hold up
22. I am thinking of changing my current job because I can’t ______ my low salary.
A. live on B. pay out C. save up D. set out

22
23. We must continually learn and acquire new knowledge if we are to adapt and ______ changing
events.
A. lose contact with B. catch sight of C. do away with D. keep up with
24. If Bill loses his job, at least they have Mary’s income to ______ on.
A. fall behind B. fall for C. fall back D. fall under
25. She ______ her job as an event manager to become a self-employed baker.
A. gave up B. gave away C. gave over D. gave off
26. Mary met her second husband not long after her first marriage ______.
A. broke in B. broke down C. broke up D. broke off
27. She had been depressed all day but she started to ______ after she heard that she was
promoted.
A. turn up B. cheer up C. take up D. break up
28. I could hear voices but I couldn't ______ what they were saying.
A. bring about B. turn up C. try out D. make out
29. Peter was bitterly disappointed at the test result, but I think he’ll soon ______ it.
A. put through B. get over C. make up D. turn into
30. The province’s personnel administration department announces to award prizes to the staff
who can ______ good solution
A. come up with B. catch up with C. go through with D. live up to
31. The firefighters arrived at the scene just in time to ______ the fire.
A. call back B. stand up C. count on D. put out
32. You will be brought before the disciplinary panel to ______ your behavior.
A. turn down B. take up C. account for D. put down
33. She had been depressed all day but she started to ______ after she heard that she was
promoted.
A. turn up B. cheer up C. take up D. break up
34. He’s a widower who ______ three children.
A. takes after B. makes off C. cuts down D. brings up
35. Jane______ the word in the dictionary whenever she is not sure of its spelling or meaning.
A. looks up B. stands for C. takes after D. turns on
36. The online game “Dumb ways to die” quickly_ with young people after being released in 2013.
A. took on B. caught up C. caught on D. took up
37. Laura decided to ______ the job offer as she didn’t want to move to a new city.
A. turn down B. take over C. go after D. make for
38. Pete has ______ strange hobbies like collecting bottle caps and inventing secret codes.
A. gone on B. taken up C. gone with D. taken off
39. Amanda ______ her boyfriend during her trip to Vietnam, and she has been single ever since.
A. broken up with B. fallen out with C. faced up with D. put up with
40. Such approaches should be supported and mainstreamed in health interventions in order to
______ positive behavior change.
A. put off B. set off C. bring about D. hold up
41. Peter was bitterly disappointed at the test result, but I think he’ll soon ______ it.
23
A. put through B. get over C. make up D. turn into
42. As Joe's roommate, I find him a fairly nice fellow, even if at times it is not easy to ______ his
noisy behavior.
A. put up with B. look up to C. get on with D. catch up with
43. It's a long walk tomorrow. We need to ______ as early as possible.
A. set up B. set in C. set off D. set about
44. Red is the symbol of danger because it ______ among other colours.
A. stands out B. stands by C. takes off D. settles down
45. A good friend should ______ you whatever happens.
A. be in favor of B. take after C. stand by D. bring around
46. “My daughter, Mary, tries to _ see me at least once a week,” Mrs. Jones told the social worker.
A. call up B. go up C. come on D. drop in
47. They ______ a big fortune when they were young, so they didn’t have to work hard.
A. came into B. went up C. came across D. went around
48. I can't ______ what he's doing; it's so dark down here.
A. make out B. see through C. look into D. show up
49. While Jack was looking through his old albums the other day, he ______ this photograph of
his parents’ wedding.
A. took after B. made up C. turned down D. came across
50. I ______ to see Chris on my way home.
A. dropped back B. dropped in C. dropped out D. dropped off
PRACTICE 2

1. Such approaches should be supported and mainstreamed in health interventions in order


to ________ positive behavior change.
A. bring about B. hold up C. set off D. put off
2. Why do you ______ talking about money all the time?
A. keep on B. give up C. take after D. get up
3. Urbanization programs are being ____________ in many parts of the worlds, espcially in
densely populated regions with limited land and resources.
A. carried out B. struggled for C. brought back D. happened
4. He's a widower who _________ three children.
A. makes off B. takes after C. cuts down D. brings up
5. Peter __________ some of his old love letters in his wife's desk.
A. turned up B. came across C. got on D. went over
6. "Sorry for being late. I was ____________ in the traffic for more than an hour”
A. held up B. carried on C. put off D. taken after
7. In these difficult times, the company may have to ____________ 100 laborers
A. take up B. turn down C. put off D. lay off
8. The boy's favourite activity after school is _____________ a walk in the park near his home.
A. picking up B. going for C. looking for D. taking up
9. As usual, my parents are awake at the crack of dawn when the alarm hasn’t_______.
A. put out B. gone off C. run out D. got up
24
10. Every year several languages_______. Some people think that this is not important because life
will be easier if there are fewer languages in the world.
A. die off B. die out C. die away D. die down
11. We________ a number of high-school graduates each year as interns to give them some work
experience before they begin college.
A. take on B. set up C. put on D. go off
12. Numerous companies have been________since the outbreak of the Covid 19.
A. fallen through B. wiped out C. carried out D. taken up
13. The volunteer team _____ students with various visual, hearing, physical and cognitive
impairments every two months to give them both financial and spiritual support.
A. call back B. call out C. call on D. call off
14. Thousands of American innovators________ thousands of practical ideas and technologies in
the fight against the coronavirus.
A. looked into B. came up with C. settled down D. turned into
15. When they________ for the beach the sun was shining, but by the time they arrived it had
clouded over.
A. set off B. left out C. carried out D. went through
16. We arranged to meet at 7.30, but she didn't______.
A. take off B. turn down C. take up D. turn up
17. They argue all right, but they always_______ before long.
A. pull up B. put up C. turn up D. make up
18. She has lived with her aunt since her parents______ 8 years ago.
A. switched off B. put off C. passed away D. brought up
19. Let's wait here for her, I'm sure she'll _____.
A. turn off B. tum over C. tum down D. turn up
20. Some old people _______ golf when they retire.
A. look back B. put on C. switch off D. take up
21. David is a rich man because he________ a lot of money last year.
A. went through B. checked in C. fell behind D. came into
22. Laura decided to _______ the job offer as she didn’t want to move to a new city.
A. turn down B. take over C. go after D. make for
23. This species of African elephants is_______.
A. dying on B. dying down C. dying out D. dying of
24. Vietnam failed to ______ in the second leg of the 2022 AFF Cup after losing 0-1 and let
Thailand lift the trophy.
A. sort it out B. bring it about C. look it over D. turn it around
25. Michael________ his wet boots and sat by the fire to warm himself.
A. put off B. put on C. took off D. took on
26. I was late for school this morning because my alarm clock did not ______.
A. put off B. turn off C. send off D. go off
27. Jane_______ the word in the dictionary whenever she is not sure of its spelling or meaning.
A. looks up B. turns on C. takes after D. stands for
25
28. She________ the job offer because she didn't want to move to another city.
A. turned out B. turned up C. turned off D. turned down
29. Do you find it easier to___________ what's happening when you watch a film in English?
A. see through B. work out C. make up D. get round
30. Everyone was surprised when that little-known band_______ the night's biggest award.
A. took off B. ran off C. went off D. carried off
31. I had to ________the job offer because the salary was too low.
A. get down B. get up C. turn down D. turn up
32. I can’t ________ the noise from the neighbors. They sing karaoke so loudly, even at late night.
A. catch up with B. put up with C. take up on D. cut down on
33. I could hear voices but I couldn't_______ what they were saying.
A. make out B. turn up C. try out D. bring about
34. The government hopes to ______ its plans for introducing cable TV.
A. carry out B. take after C. carry back D. turn out
35. Don't smoke in the forest. Fires________ easily at this time of the year.
A. take off B. put out C. break out D. turn down
36. We should_______ research to identify the real problems.
A. take on B. put off C. look up D. carry out
37. People are destroying the environment by ________large areas of forest.
A. breaking up B. building up C. cutting down D. moving away
38. After a three-hour operation, she hasn’t _______ yet.
A. cut down B. taken off C. put out D. come around
39. The new CEO plans to ______ the company and implement some major changes in the way it
operates. A. pull out B. get over C. take over D. take off
40. Carol's performance didn't really_______ her usual high standard.
A. come up to B. come down to C. get round to D. feel up to
41. I_______ going to the movies when I moved out of Ho Chi Minh City.
A. put off B. looked up C. gave up D. turn off
42. I couldn't________ what he had talked about because I didn't get used to his accent.
A. come up B. break up C. make out D. get off
43. Joe_________ a lot of money, which changed his life completely.
A. came up with B. came into C. came in for D. came down with
44. Mrs. Susan________ the job offer because it involved too much travelling.
A. looked for B. kept on C. turned down D. took up
45. In the Soviet Union, Serbia, Austria and Hungary, there has always been a habit of______
shoes and wearing slippers when they have come home.
A. taking off B. taking up C. putting on D. putting with
46. Please_______ and see us some time - you're always welcome.
A. come to B. come about C. come away D. come round
47. Yesterday I had to catch a taxi because my car ________ when I was on my way to work.
A. took after B. gave away C. turned on D. broke down

26
48. The state government will _______ a new restriction on personal weapons.
A. stay over B. take off C. bring in D. put aside
49. I was late for work this morning because my alarm didn't_____.
A. get over B. hold up C. go off D. put on
50 Unless we do something now, hundreds of plant and animal species will______.
A. make up B. point out C. cut up D. die out
51. Our business is doing so well that we'll soon have to_______ new staff to help us.
A. keep up B. take on C. put off D. give in
52. Many people wonder why the trend "Get-go!" has________ with young people.
A. caught on B. appealed to C. gotten on D. brought about
53. I think your father should_______ smoking because his health becomes worse.
A. set up B. look after C. take on D. give up
54. Before I sign, I'd like to_______ the details of the agreement one more time.
A. go ahead B. go on C. go off D. go over
55. If you _______unpleasant thing or an illness, you recover from it.
A. look after B. take after C. go over D. get over
56. The teacher told her to________ her socializing, because it was affecting her schoolwork.
A. cut down on B. get on with C. drop out of D. face up to
57. It was so foggy that drivers couldn't_______ the traffic signs.
A. make out B. keep out C. take out D. break out
58. We have always had to_______ extra staff at Christmas.
A. take over B. take up C. take on D. take in
59. Governments should_______ international laws against terrorism.
A. turn out B. take off C. bring in D. see through
60. The boy was happy to have______ the solution to the mathematical problem.
A. worked out B. turned down C. got in D. switch off
61. She had been depressed all day but she started to_______ after she heard that she was
promoted. A. break up B. cheer up C. take up D. turn up
62. She has spent too much money on clothes. She should_______ her expenses.
A. cut out B. cut off C. cut down D. cut up
63. John, could you_______ my handbag while I go out for a minute?
A. take place B. take over C. take part in D. take care of
64. Luckily, they successfully_______ the fire and saved all of the children.
A. took out B. took off C. take off D. put out
65. Laura decided to_______ the job offer as she didn't want to move to a new city.
A. take over B. make for C. go after D. turn down
66. What may happen if John does not_______ in time for the field trip to the Colosseum?
A. turn up B. turn down C. turn back D. turn over
67. This picture makes me________ the time we were at elementary schools.
A. drop out of B. talk back to C. think back on D. make up for
68. Sportsmen______ their political differences on the sports field.

27
A. cake up B. put aside C. keep part D. deal with
69. After her parents died, she was______ by her grandmother.
A. brought back B. grown up C. brought up D. come into
70. This theory is quite difficult to take in. Could you_______ for me?
A. get it across B. break it down C. put it up D. turn it off
71. Deforestation _________ devastative floods in many parts of the world.
A. makes up B. starts out C. results in D. brings up
72. Her explanation of the phenomenon was so confusing that I couldn't_____ anything.
A. call for B. put back C. take in D. go over

7. PASSIVE VOICE

*Form: S + “be” + V3/ed….(by O)

Question 8: The kitchen ____ by my mother last Sunday morning.


A. was cleaning B. cleans C. has cleaned D. was cleaned

1. The telephones ______ by Alexander Graham Bell.


A. invented B. is inventing C. was invented D. is invented
2. The flood victims ______ with food and clean water by the volunteers.
A. provided B. were provided C. were providing D. provide
3. Dung ______ as the captain of his school's football team.
A. was selected B. had selected C. is selecting D. has selected
4. Most of the movie scenes ______ in a picturesque coastal town last year.
A. film B. were filmed C. have filmed D. are filming
5. The woman ______ by the police last week.
A. was freed B. were freed C. has been freed D. has freed
6. We can't go along this road because it ______.
A. repaired B. is repairing C. is being repaired D. was repairing
7 Nowadays, many serious childhood diseases ______ by early immunization.
A. can be prevented B. are preventing C. prevent D. can prevent
8. The road to our village ______ next year.
A. widened B. can widen C. has widened D. will be widened
9. Wallets _______to keep money and some important documents, such as identity card or
A. are used B. have used C. will use D. use
10. The car ______ three days ago.
A. washed B. was washing C. was washed D. has washed
11. This old building ______ a hundred years ago.
A. has built B. builds C. was built D. was building
12. The first round of France's 2022 presidential election ______ on 10 April.
A. was held B. held C. had held D. was holding
13. There was anger at the proposal that a UN peacekeeping force ______ to the area.

28
A. is sent B. must be sent C. be sent D. sent
14. Last night Laura ______ a big diamond ring by her husband at her birthday party.
A. was giving B. gave C. gives D. was given
15. Vietnamese soldiers and medical workers ______ to Turkey for earthquake relief.
A. were sent B. have sent C. send D. had sent
16. Do you know that many useful things ______ by accidents?
A. were invented B. is inventing C. invented D. have invented
17. This house ______ by my grandfather in 1970.
A. was built B. has built C. built D. was build
18. How many houses _____ by the storm last night?
A. are destroyed B. have been destroyed C. were destroyed D. are being destroyed
19. The longest fish in the contest was eighteen inches long". - "It ______ by Thelma Rivers”.
A. was catching B. caught C. was caught D. catch
20. Do you think that teachers ______ by robots in the future?
A. replace B. were replaced C. have replaced D. will be replaced
21. Such shameful things _______ in public places.
A. ought not to have allowed B. ought to not be allowed
C. ought not to allow D. ought not to be allowed
22. I hope that when my birthday comes, I_______ a new motorbike by my father.
A. gives B. will give C. was given D. will be given
23. This old villa _______ on the top of the hill many years ago.
A. built B. was building C. was built D. has built
24. In the last decade, remarkable progress________ in the field of robotics.
A. has achieved B. has been achieved C. was achieved D. is achieved
25. Acupuncture is part of traditional Chinese medicine and _____ in China for thousands of years.
A. has been used B. has using C. has used D. has been using
26. Facebook users should set strong passwords to avoid____ into their accounts by strange people.
A. to be hacked B. being hacked C. hacking D. to hack
27. Peter is walking happily hoping that he_______ to the party by his girlfriend.
A. invites B. will invite C. was inviting D. will be invited
28. The contract_________ by the manager tomorrow.
A. will be signing B. will have signed C. will sign D. will be signed
29. Many new roads in my neighborhood_______ by foreign workers next month.
A. have built B. will be built C. build D. built
30. Harry Potter books _____ by J.K. Rowling.
A. are writing B. were written C. have written D. wrote
31. My new school_________ near the river in this suburb next year.
A. will be rebuilt B. is built C. is building D. built
32. Different types of music________ at the festival last month.
A. were finding B. find C. were found D. found
33. The company hopes that after the new product comes out, it ______ by lots of customers.
A. will purchase B. purchase C. will be purchased D. is purchasing
34. The restaurant ______ to accommodate more diners.
A. should be enlarged B. will enlarge C. has enlarged D. enlarges

29
35. The library's entire collection ______ by a team of dedicated volunteers.
A. organize B. will organize C. was organizing D. was organized
36. Our test papers ________ by our teachers now.
A. will mark B. will be marked C. are marking D. are being marked
37. By an unfortunate coincidence, their house______ on the day he lost his job.
A. was burgling B. burgled C. was burgled D. burgles
38. In 1911, Marie Curie________ a Nobel Prize in Chemistry in recognition of her discovery of
the two elements radium and polonium.
A. being awarded B. was awarded C. awarded D. was awarding
39. Some recommend that more field hospitals _________ to cope with another wave of
coronavirus infection.
A. build B. building C. be built D. to be built
40. A large area of the Amazonian rainforest_______ by the forest fire and many wild species were
on the verge of extinction.
A. devastate B. will devastate C. was devastating D. was devastated
41. A new supermarket ______ in our neighbourhood in June 2025.
A. builds B. will be built C. build D. will build
42. Highway 15 ______ yesterday due to a serious road accident by the local authority.
A. has closed B. was closing C. was closed D. closes
43. The medicine ______ at a high place where the children can't reach.
A. must be kept B. must keep C. must be keeping D. must not kept
44. The man got his car ________ by a mechanic yesterday.
A. maintain B. maintained C. to maintain D. be maintained
45. I think the game_______ by the other team. They are a lot better than we are.
A. wins B. will be won C. will win D. were winning
46. The winners________ by the organizers at the end of the competition.
A. will be announced B. are announcing C. announce D. will announce
47. The store has promised that my smart TV______ tomorrow.
A. will deliver B. would deliver C. will be delivered D. are delivered
48. The data from 21 top universities______ by an independent company.
A. have assessed B. assess C. were assessed D. had assessed
49. Each of the guests______ a bunch of flowers yesterday.
A. are given B. is given C. was given D. give
50. This matter_______ with as soon as possible.
A. will deal B. was dealt C. will be dealt D. deals
51. People believe that a new album ______ by that famous singer next year.
A. will be released B. is releasing C. will release D. releases
52. Two new schools ______ in our area because of the growth in population.
A. built B. will be built C. will build D. have built
53. Since the computer_______, it has made considerable changes to our daily life.
A. had invented B. was inventing C. was invented D. invented
54. The trees in the garden ______ in the wind.
A. damaged B. are damaged C. were damaged D. damage
55. A new school_______ in Ca Mau province by FPT Corporation in the near future.
A. will be built B. be built C. built D. build
30
56. The new sports_______ by the mayor as soon as it is completed.
A. will be opened B. opens C. was opening D. will open
57. Some experts believe that many complicated tasks______ by robots in the future.
A. have been done B. did C. are done D. will be done
58. The concert______ by thousands of people as I predict.
A. will have attended B. attended C. is attending D. will be attended
59. Facebook_______ for intentionally leaking non-users' personal data.
A. has been fined B. fines C. has fined D. fined
60. The current constitutional problem______ by the top legal minds in the country.
A. have studied B. is being studied C. were studied D. is studying
61. It is clear that our breathing________ by the pollution in the city if the number of cars keeps
growing.
A. will be affected B. was affecting C. was being affected D. will be affecting
62. The company hopes that their new product_______ by many customers.
A. buys B. will buy C. was buying D. will be bought
63. The project_______ by a team of experts in the field next month.
A. completes B. will complete C. will be completed D. was completing
64. My house_______ by my grandfather in two months.
A. was built B. will be built C. were built D. has built
65. The volunteer recognition awards ______ at the annual ceremony next year.
A. will present B. are presenting C. will be presented D. present
66. The report on the vulnerable species ______ by the research team tomorrow.
A. will be completed B. completed C. will complete D. is completing
67. A lot of valuable gifts______ to customers on the 20th birthday of the local bank.
A. will be given B. gave C. are giving D. have given
68. We hope that the museum_______ by millions of people when it opens next week.
A. visits B. will be visited C. was visiting D. will visit
69. They_______ this story by their grandmother when they visited her last week.
A. were told B. are told C. is telling D. had told
70. Their date of the wedding_________ for 26th May.
A. has fixed B. fixes C. fixed D. is fixed
71. The trees________ more and more in our schoolyard next year.
A. will plant B. will be planted C. are planted D. plant
72. I don’t remember _________ of the decision to change the company policy on vacations. When
was it decided?
A. telling B. to be told C. to tell D. being told
73. Students_______ with necessary skills for their future careers during this course.
A. have equipped B. had equipped C. are equipping D. will be equipped
8. DẠNG CỦA ĐỘNG TỪ (VERB FORM) – TO VO // VING
Question 9: I will never forget ____ with Tom. We had a wonderful time together.
A. working B. work C. to work D. to working
(forget +to Vo: quên chuyện nên làm, bổn phận// forget +Ving: quên chuyện đã làm rồi. Xét về
nghĩa ta chọn “working”)

31
Practice 1. Mark the letter A, B, C, or D on your answer sheet to indicate the correct answer to
each of the following questions.
1. She is looking forward to ______ you again.
A. meet B. meeting C. met D. be met
2. You can't prevent him from ______ alcohol.
A. drink B. to drink C. drinking D. drank
3. It’s no good ______ to him, he never answers letters.
A. write B. being written C. writing D. to write
4. The children loved ______ the old castle.
A. explore B. exploring C. explored D. to exploring
5. Smokers generally do know that smoking is extremely harmful, but it’s just that they can’t help ____ it.
A. doing B. to do C. do D. having done
6. He speaks English well enough______ with foreigners.
A. to communicate B. communicating C. having communicatedD. communicated
7. Most people talk too much and don’t realize how important ______ is.
A. to be listening B. to be listened C. have listened D. listening
8. If you’ve finished ______ the dictionary, I’d like to borrow it.
A. to have used B. to use C. using D. having used
9. On the whole, I enjoy ______ our local club, but I think there are far too many rules and
regulations to feel completely at ease.
A. attending B. attend C. to attend D. being attended
10. Why doesn’t he fancy ______ with us to the pub?
A. having come B. coming C. to have come D. to come
11. They decided ______ Japan for their summer holiday.
A. going B. to go C. go D. to going
12. Peter's father ordered him not ______ out late again.
A. stay B. to staying C. staying D. to stay
13. Smoking is bad for your health. You had better ______ it up.
A. to give B. give C. giving D. to be giving
14. His written French is very good but he needs to practise ______ it.
A. speaking B. speak C. having spoken D. to speak
15. I’d like ______ all of you to enjoy my party on this Friday.
A. to invite B. inviting C. invite D. not invite
16. We expect Linh ______ to the airport late as the plane will take off in 15 minutes.
A. to come B. not to come C. not coming D. coming
17. I'm not used ______ as early as this.
A. getting up B. get up C. to get up D. to getting up
18. More emphasis should be placed on ______ alternative sources such as wind, solar energy, and
tides. A. to develop B. develops C. developing D. developed
19. She refused ______ further about the current situation in the company.
A. commented B. commenting C. to commenting D. to comment

32
20. How do you manage ______ in this heat without air conditioning?
A. to live B. living C. live D. lived
21. We avoided ______ through the city during the rush hour.
A. drive B. driving C. to drive D. drove
22. We decided ______ at home for the weekend.
A. staying B. stayed C. stay D. to stay
23. James intended ______ up for an online course to improve his writing skills.
A. to sign B. signed C. signing D. sign
24. Susan advised me ______ the job offer, but I didn't want to.
A. accepting B. accepted C. to accept D. accept
25. Teenagers enjoy ______ the Internet because there’s a lot of interesting things on it.
A. surfed B. to surf C. surf D. surfing
26. My mother told me off for forgetting ______ this package to my aunt.
A. sending B. to send C. send D. sent
27. He tried ______ hard to achieve good results in the exam.
A. to work B. worked C. work D. working
28. Laura refused ______ a face mask because she thought it was unfashionable.
A. to wear B. worn C. wearing D. being worn
29. The storm is expected ______ landfall on the coast of the island.
A. make B. making C. to make D. made
30. The doctor advised the patient ______ more vegetables.
A. eat B. eats C. eating D. to get
31. My parents persuade me ______ my mind, but I can’t. I just really love him.
A. changed B. changing C. to change D. change
32. The government decided ______ 2 weeks of social distancing on this infected area.
A. to impose B. imposed C. impose D. being imposed
33. Olga promises ______ me this novel as soon as she finishes it.
A. lent B. to lend C. lending D. lend
34. Despite being quite busy, Lucy still spends her little free time ____ shopping with her friends.
A. gone B. being gone C. to go D. going
35. I remember ______ off the lights, but when I came back, the lights were still on.
A. turning B. being turned C. to turn D. to be turned
36. Hearing some noise, John stopped ______ with his friend and looked outside.
A. chatted B. chat C. chatting D. to chat
37. I don’t want my daughter ______ this post; it may hurt her a lot.
A. seeing B. to see C. seen D. saw
38. Mary suggested ______ a four-seat car to go sightseeing.
A. to renting B. to rent C. rented D. renting
39. We don't mind ______ poor people.
A. helping B. to helping C. to help D. help
40. Anyone interested in ______ the club should contact us at the address below.
33
A. join B. joining C. joins D. to join
41. John admitted ______ a mistake in his calculations.
A. make B. making C. to make D. to have made
42. My parents regretted ______ the opportunity to visit Paris last year.
A. missing B. miss C. missed D. to miss
43. Would you like ______ some chocolate?
A. having B. have C. to have D. All are correct
44. Let us ______ a camping fire.
A. to make B. make C. making D. to making
45. My parents are not strict with us. They allowed ______ up late at weekends.
A. stay B. staying C. to stay D. stayed
46. The robber admitted ______ the red mobile phone last week.
A. steal B. stolen C. to steal D. stealing
47. Will you remind me ______ this letter at the post office?
A. to post B. posting C. to be posted D. being posted
48. It takes me 15 minutes ______ to school every day.
A. go B. going C. to go D. went
49. Students should ______ their homework before going to bed.
A. finish B. finishing C. finish D. finishing
50. He offered ___________ me a ride home after the party.
A. give B. given C. to give D. gave
Practice 2
1. The college principal promised________ into the matter.
A. looked B. look C. looking D. to look
2. Have you finished___________ your hair yet?
A. to wash B. washed C. washing D. wash
3. The new government has promised________ more on health and education.
A. to spending B. to spend C. spend D. spending
4. Although he knew very little about linguistics, he pretended_____ an expert in the field.
A. being B. to be C. been D. be
5. She asked me__________ her when she was in trouble.
A. helping B. help C. to help D. helped
6. She should avoid_______ other people's feelings.
A. hurting B. to hurt C. hurt D. hurts
7. In the sustainable agriculture, farmers try_______ the use of chemicals and fertilizers.
A. limited B. limit C. limiting D. to limit
8. The cut on his leg took a very long time______.
A. healing B. heal C. for healing D. to heal
9. Cybersecurity experts recommend______ two-factor authentication to protect your online
accounts.
A. using B. used C. to using D. to use
10. His parents think it’s time for him_________ married.
A. gets B. to get C. get D. got

34
11. Thanks goodness, I finally managed_____ the problem.
A. solve B. to solving C. solving D. to solve
12. I was in two minds whether or not to repair my old laptop, but I finally decided______ the new
one.
A. to buying B. buy C. to buy D. buying
13. He promised_______ his daughter a new bicycle as a birthday present.
A. to buy B. buying C. buy D. to buying
14. Sheila can't help_____ whenever she watches romantic films.
A. cries B. to cry C. crying D. cry
15. My sister promises________ me a new pair of shoes on my next birthday.
A. buying B. to buying C. buy D. to buy
16. He didn't agree_______ me go on leave in March.
A. let B. to let C. to letting D. letting
17. The trainer reminded me_______ in more vegetables for dinner.
A. having taken B. of taking C. to take D. to be taking
18. The coach reminded the team_________ their best effort in the game.
A. give B. giving C. to giving D. to give
19. He promised_______ us the package by 6 P.M.
A. to send B. sending C. send D. to sending
20. Lan hopes______ the United States some day.
A. to visit B. visit C. visiting D. to visting
21. He decided_______ part in a voluntary campaign with his classmates this summer.
A. to take B. take C. to taking D. taking
22. I'd expected_________ some weight when I gave up smoking, but I didn't.
A. to gain B. gain C. gaining D. gained
23. Joan is considering________ her major from pre-med studies to psychology.
A. to changing B. to change C. changing D. change
24. Frederick intends_______ a computer course about artificial intelligence after he graduates
from high school.
A. taking B. to take C. to taking D. take
25. I hope_______ you again in the next meeting.
A. meet B. to meet C. meeting D. to meeting
26. He wants_______a whole bag of chocolate chip cookies before lunch.
A. have eaten B. to eat C. eating D. of eating
27. She wanted ______ French to help with her upcoming trip to Paris.
A. learn B. to learn C. to learning D. learning
28. Jean decided_______ an IT course to improve her computer literacy.
A. to take B. to taking C. taking D. take
29. I am sorry to keep you_______ for so long.
A. to wait B. to waiting C. wait D. waiting
30. All of my students expect ________the coming exam with flying colours.
A. to passing B. to pass C. pass D. passing
31. He promises________ the assignment before 5 p.m.
A. submit B. submitting C. to submitting D. to submit
32. At first, we intended to go out for the walk. We then decided________ to the theater.
35
A. go B. going C. to go D. to going
33. I don't mind ________a little extra for organic produce because it's healthier.
A. paying B. to pay C. paid D. pay
34. Facebook users should set strong passwords to avoid________ into their accounts by strange
people.
A. to be hacked B. being hacked C. hacking D. to hack
35. I don't mind_______ her to carry the heavy suitcase upstairs.
A. helping B. helped C. to help D. help
36. I don’t remember _________ of the decision to
change the company policy on vacations. When was it decided?
A. telling B. to be told C. to tell D. being told
37. Although he knew very little about linguistics, he pretended______ an expert in the field.
A. to be B. be C. being D. been
38. In court, she admitted ______ almost $1 million as "consulting fees".
A. receiving B. receive C. to receive D. received
39. Would you mind_______ me a hand with this bag?
A. to give B. given C. give D. giving
40. Tony forgot _______his name on the exam paper.
A. write B. written C. writing D. to write
41. Lisa recalls ______ a slideshow of old photos from her school days at the reunion.
A. to watch B. to watching C. watching D. watch
42. They planned________ the presentation with a short description of the school.
A. to start B. start C. starting D. started
43. I hope_______ all my courses this term. So far my grades have been
A. to passing B. passing C. pass D. to pass
44. He offered________ me to swim but I said no.
A. to teach B. of teaching C. teaching D. teach
45. Don't forget_______ her my message when you see her.
A. gave B. giving C. to give D. give
46. John volunteered_______ the food supply to the victims in the flood area.
A. to be taken B. take C. taking D. to take
47. All the candidates need __________ about the uniform before being interviewed.
A. inform B. informed C. to be informed D. to inform
48. The job applicant is expected ______ good at time management and communication.
A. to being B. to be C. being D. be
49. The dolphin injured itself while struggling ____________ out of the fisherman’s net.
A. to get B. getting C. to getting D. get
50. I always forget_________ my alarm for Monday mornings.
A. to setting B. set C. to set D. setting
9. TAG QUESTION – CÂU HỎI ĐUÔI
Question 10: They should write an outline for their presentation, ____?
A. should they B. shouldn’t they C. do they D. don’t they
1. Mary traveled to London last month,________?
A. did she B. wasn't she C. was she D. didn’t she
36
2. Someone has just arrived,__________?
A. does he B. haven't they C. have they D. doesn’t he
3. He cheated in the exam two days ago,_______?
A. will he B. didn't he C. did he D. won't he
4. Students often use their calculators to solve tricky questions,_______?
A. don't they B. do they C. do we D. don’t we
5. The situation of COVID-19 worldwide is still serious, ______?
A. isn't it B. hasn't it C. doesn't it D. won't it
6. Bob told you something about my story,_______ ?
A. didn't he B. doesn't he C. hadn't he D. did he
7. John was absent from school for the whole week, ______?
A. wasn't he B. did he C. was he D. didn’t he
8. Your sister is cooking lunch in the kitchen,_______?
A. is she B. was she C. wasn’t she D. isn’t she
9. The shop often closes at 7 p.m.,_______?
A. does it B. isn't it C. is it D. doesn't it
10. He is a good cook,_______ ?
A. does he B. is he C. didn’t he D. isn’t he
11.That is your brother who helped you finish your project,_________?
A. is it B. isn't she C. isn't he D. isn't it
12. His bag is on the table,________?
A. did it B. does it C. wasn’t it D. isn’t it
13. This book is very important for your project, _______?
A. hasn't it B. isn’t it C. wasn’t it D. did it
14. Many teenagers feel bored during their summer holidays,_____?
A. did they B. didn't they C. do they D. don’t they
15. Your school is located in an underdeveloped area,_____?
A. wasn't it B. isn't it C. didn't it D. doesn't it
16. He won't be late for the meeting, ______?
A. won’t he B. isn’t he C. was he D. will he
17. You have visited Ha Long Bay,_______?
A. have you B. haven't you C. don’t you D. do you
18. Your mother hasn't met him before, ______?
A. did she B. has she C. hasn’t she D. didn’t she
19. Digidog is a robotic dog used for policing purposes,______?
A. does it B. isn't it C. didn't it D. wasn’t it
20. He failed to fulfill his promise,_______?
A. did he B. didn't he C. was he D. wouldn’t he
21. Your mother often cooks meals for the whole family, ______?
A. didn't she B. doesn't she C. isn't she D. wasn’t she
22. The number of rare animals in the world is decreasing so rapidly,______?
37
A. doesn't it B. is it C. isn't it D. does it
23. Mary went to the party with her mother,_______?
A. did she B. didn't she C. wasn’t she D. was she
24. We should begin our discussion now,_____?
A. won't we B. don't we C. will we D. shouldn’t we
25. Yesterday, they didn't complete their tasks,_______?
A. didn't they B. do they C. don't they D. did they
26. Your mother is cooking in the kitchen,______?
A. doesn't she B. isn't she C. didn’t she D. won’t she
27. Her new skirt is beautiful, ______?
A. doesn't it B. does it C. isn’t it D. is it
28Michael has attended over 100 meetings so far,______?
A. hasn't he B. does he C. has he D. doesn’t he
29. Your parents will celebrate their wedding anniversary next month,______?
A. will they B. don’t they C. do they D. won’t they
30. Sue is very good at doing the household chores,_________?
A. doesn't she B. was she C. does she D. isn’t she
31. We are having the lesson early tomorrow, ______?
A. don't we B. aren't we C. do we D. are we
32. The garden in which you are planting strawberries is very large and beautiful, ______?
A. are you B. is it C. aren't you D. isn't it
33. Your exam results are satisfactory, ________?
A. is it B. are they C. isn’t it D. aren’t they
34. There are many religions in Malaysia, ______ ?
A. are they B. aren't there C. aren't they D. are there
35. You have a ticket to the water puppetry,_____?
A. do you B. haven't you C. have you D. don’t you
36. Nobody comes there on time,______?
A. don't they B. does he C. do they D. doesn’t he
37. Donald John Trump is the 45th President of the United States,_____?
A. isn't it B. is not he C. is he D. isn’t he
38. Mary traveled to London last month,________?
A. did she B. wasn't she C. was she D. didn’t she
39. Lady Gaga is an American singer, songwriter and actress,_______?
A. is she B. doesn't she C. isn't Lady Gaga D. isn’t she
40. These cats look immensely lovely,______?
A. are these B. are they C. don't they D. don’t these
41. You saw the film "Lat Mat 6" last week,________?
A. were you B. did you C. didn't you D. weren't you
42. Jim rarely returns to his hometown,______?
A. hasn't he B. does he C. has he D. doesn't he
38
43. They never go to the cinema with you, _________?
A. didn't they B. did they C. do they D. don’t they
44. Michael returned to his hometown last summer,_______?
A. didn't he B. doesn't he C. won't he D. hasn’t he
45. You couldn't give me a helping hand, ___________?
A. Do you B. could you C. don't you D. couldn't you
46. Your mother is cooking in the kitchen, ___________?
A. isn't she B. doesn't she C. won’t she D. is she
47. The library is very quiet, ______________?
A. has it B. isn't it C. wasn’t it D. is it
48. He will invite her to the school prom, __________?
A. doesn't he B. won't he C. does he D. will he
49. No amount of money can buy true friendship, ________?
A. can't it B. can it C. does it D. doesn’t it
50. Your brother is going to study abroad, __________?
A. isn't it B. is he C. isn't he D. aren't you
51. No one died in the accident, _____________?
A. did they B. aren't they C. didn't they D. are they
52. He hasn't been running in this weather, _________ ?
A. has he B. does he C. hasn’t he D. doesn’t he
53. There have been some changes in weather patterns, _______?
A. haven’t they B. has it C. haven’t there D. have they
54. She feeds the chickens twice a day,__________?
A. hasn't she B. isn’t she C. doesn’t she D. didn’t she
55. His welcoming speech wasn't good, _______?
A. was he B. wasn't he C. wasn't it D. was it
56. . There are some people having lunch in the cafeteria, _______?
A. aren’t they B. weren’t they C. aren’t there D. weren’t there
57. Those cars are very expensive, ______?
A. are cars B. aren't they C. aren't cars D. are they
58. Mary is writing an essay in her room, _______?
A. isn’t she B. wasn’t she C. didn’t she D. hasn’t she
59. The new office looks spacious and modern,_______?
A. doesn't it B. does it C. has it D. isn’t it
60. You have a ticket to the water puppetry, _____?
A. don’t you B. do you C. have you D. haven’t you
61. She didn't go yesterday,______?
A. does she B. doesn’t she C. did she D. didn’t she
62. Her parents are really strict. They rarely let her stay out late, _________?
A. do they B. does she C. doesn’t she D. don’t they
63. He needs to do the homework before going out with his friends,_______?
A. doesn't he B. does he C. needn't he D. need he
39
64. I don't think Jill would be a good teacher. She's got little patience,_______?
A. hasn't she B. is she C. isn't she D. has she
65. They haven't finished their homework,_________?
A. don't they B. haven't they C. have they D. do they
66. It is very noisy today,________?
A. is it B. isn't it C. hasn’t it D. has it
67. This room isn’t large enough, _______?
A. isn’t it B. hasn’t it C. does it D. is it
68. Nobody phoned while I was out,_________?
A. wasn't I B. was I C. did they D. didn’t they
69. Hana rarely goes to the theatre, _______?
A. didn’t she B. has she C. doesn’t she D. does she
70. The trains are never on time, _________?
A. aren't they B. are they C. do they D. are the trains
71. They can all kinds of fruit, __________?
A. can’t they B. don’t they C. can they D. do they
72. You haven’t applied for the job, ______?
A. haven’t you B. have you C. did you D. didn’t you
73. What they are interested in doesn’t concern me, __________?
A. do they B. aren’t they C. are they D. does it
74. The examination is going to take place at the end of June, _______?
A. isn’t it B. doesn’t it C. wasn’t it D. won’t it
75. They were having lunch at this time yesterday,______?
A. were they B. are they C. don’t they D. weren’t they
76. They were having lunch at this time yesterday,______?
A. were they B. are they C. don’t they D. weren’t they
77. There have been many achievements since the country carried out economic reforms, ______?
A. don't they B. haven't there C. haven't they D. don't there
78. Canned food is not as healthy as fresh food, ______?
A. isn't it B. does it C. doesn't it D. is it
79. You had your house decorated, _______?
A. hadn't you B. weren't you C. didn’t you D. haven’t you
80. Tom invited us to come to his party, __________?
A. hadn't he B. couldn't he C. wasn't he D. didn't he
81. The teacher read the boy’s essay, _______?
A. doesn’t she B. didn’t she C. isn’t she D. hasn’t she
82. I believe that scuba diving is one of the most adventurous sports,_______?
A. don't I B. do I C. isn't it D. is it
83. Let's go to the beach this weekend, ________?
A. do we B. shall we C. will we D. don’t we
84. This exercise is not challenging, ______?
40
A. isn't it B. is it C. doesn't it D. does it
85. Apples are now in season,_______?
A. have they B. aren't they C. won't they D. were they
86. Phong will fly to Cambodia to watch the 32nd SEA Games, ______?
A. doesn’t he B. won’t he C. didn’t he D. isn’t he
87. Hana rarely goes to the theatre,________?
A. didn't she B. doesn’t she C. does she D. has she
88. Singing helps us reduce stress, _______?
A. doesn't it B. isn’t is C. don’t they D. aren’t they
89. Someone has just arrived,________?
A. does he B. have they C. haven't they D. doesn't he
90. Paul used to give away some of his allowance to help others in need, ________?
A. wasn’t he B. didn’t he C. doesn’t he D. usedn’t he

10. (Q. 11-13) - TỪ VỰNG – TỪ CÙNG TRƯỜNG NGHĨA 

Question 11: The results of a recent study greatly __ teenagers’current preferences for
leisure activities.
A. flared B. illuminated C. flashed D. blazed
* flared: bùng lên; illuminate: giải thích rõ; flash: lóe lên; blaze: bùng cháy dữ dội
=> Kết quả của một nghiên cứu gần đây đã giải thích rất rõ ràng sở thích hiện tại của thanh thiếu
niên đối với các hoạt động giải trí.
Question 13: He got a heavy---------- for breaking the traffic regulations yesterday.
A. fine B. rate C. money D. fare
*fine: tiền phạt; rate: tỉ lệ, giá..; money: tiền; fare: giá vé.
=> Hôm qua ông ấy bị phạt nặng vì vi phạm luật giao thông.

41
PRACTICE
1. His eldest brother, John, had a __________ career in the diplomatic service.
A. distinguished B. generous C. respectful D. talented
2. My vegetarian diet seems to be________ because I've lost five pounds since I started this diet.
A. operating B. working C. performing D. processing
3. We should never let misfortune and disabilities get in our_______ to success.
A. way B. road C. path D. route
4. The successful applicant for the_______ will be a well-motivated self-starter who has excellent
communication skills.
A. situation B. position C. location D. placement
5. Books are still a cheap________ to get knowledge and entertainment.
A. way B. means C. measure D. method
6. I got a diploma in business administration, but still sometimes wish I had been able to go to
university and get a _______.
A. diploma B. scholarship C. certificate D. degree
7. Bill _________at his watch and started running. He was late for school.
A. observed B. noticed C. glanced D. watched
8. All the three TV channels provide extensive _______ of sporting events.
A. broadcast B. network C. coverage D. vision
9. The police_______ their attention to the events that led up to the accident.
A. confined B. confirmed C. completed D. contained
10. Paul's ability to________ a challenge made him the perfect candidate to head up the new sales
division.A. handle B. control C. run D. order
11. The museum of modern art we went to yesterday was fantastic, but the entrance_______ was a
bit expensive. A. charge B. cost C. ticket D. fee
12. Applicants will be expected to have a good________ of English.
A. order B. aptitude C. attitude D. command
13. I lost my _______ on the way home and had to call my doctor to get a new one.
A. medicine B. prescription C. treatment D. therapy
14. Known as 'Ha Long Bay on Land', Trang An Scenic Landscape Complex is famous for its
mountains, caves, and archeological_______.
A. excavations B. relics C. heritages D. sites
15. I hope that they can find out a cure for the disease, because they are on the_______ track.
A. correct B. exact C. precise D. right
16. It's reported that Vietnamese________ are more aware of the safety of handmade products.
A. directors B. landlords C. consumers D. judges
17. There are four main blood_______: A, B, AB and 0.
A. types B. forms C. sorts D. kinds
18. She's a bit plump, but she always looks good because she often buys clothes that___ her shape.
A. correct B. flatter C. enhance D. develop
19. This______ can automatically download and install when there are new updates.
A. email B. software C. website D. hardware
42
20. Four Vietnam Airlines flight __ were investigated for smuggling drugs from France last week.
A. waitresses B. voyagers C. assistants D. attendants
21. The ______ of Vietnam is a golden star with five points representing the five classes in
Vietnamese society.
A. emblem B. slogan C. banner D. logo
22. Known as 'Ha Long Bay on Land', Trang An Scenic Landscape Complex is famous for its
mountains, caves, and archeological_______.
A. heritages B. relics C. excavations D. sites
23. Most of this growth had occurred since 1950 and was known as the population_______.
A. surplus B. explosion C. growth D. density
24. She gives the________ of being very busy.
A. attention B. consideration C. opinion D. impression
25. _______ to most of the Vietnamese universities depends on examination results.
A. Admittance B. Admission C. Permission D. Allowance
26. He waited 18 days before going on television to give an accurate_______ of the worst nuclear
disaster in history.
A. account B. speech C. narration D. illustration
27. A group of scientists has conducted a(n)________ research into the effects of the drug.
A. communicable B. successive C. comprehensible D. exhaustive
28. He was sent off for a_________ on the German captain in the final match.
A. fault B. error C. foul D. mistake
29. He won his case on appeal in 1974, in a_______ decision which encouraged other artists to
challenge the terms of their contracts.
A. scenery B. view C. landmark D. landscape
30. In most families, two salaries are essential to cover the______ of raising a family.
A. cost B. fare C. price D. tuition
31. In this job, experience accounts for more than paper_______.
A. quality B. qualifications C. certificates D. background
32. The essay is aimed at discussing the similarities and differences between English pronouns and
Vietnamese_______.
A. fellows B. partners C. twins D. counterparts
33. She finally achieved her________ of visiting the USA.
A. target B. desires C. objective D. ambition
34. The journalist was killed while on_______ in Germany.
A. appointment B. uptake C. assignment D. offer
35. In my country, after completing 4 years of education at university, students will receive a
bachelor's _________and enter the workforce.
A. degree B. diploma C. quality D. certificate
36. We are bombarded every day by______ whose role is to make products attractive enough for
customers to buy them.
A. information B. advertisements C. entertainments D. rumours
37. Books in the library are arranged by different_______.
43
A. ranks B. categories C. classes D. degrees
38The water park we went to yesterday was fantastic, but the entrance_____ was a bit expensive.
A. fee B. charge C. cost D. fare
39. To most Western people, asking about_______, age, and marriage is impolite.
A. profit B. income C. interest D. money
40. If you want antibiotics, you'll have to ask the doctor for a_______.
A. note B. recipe C. prescription D. receipt
50. You can get a mop from the school's_______ downstairs.
A. department store B. warehouse C. changing room D. storeroom
51. An important rule in Tim's family is that they have to pay_______ visits to their grandparents
in the countryside.
A. normal B. customary C. regular D. usual
52. It is important to_______ historical sites and artifacts for future generations to learn from.
A. reserve B. diverse C. conserve D. preserve
53. At the_______ level, you can join thee-year or four-year colleges.
A. postgraduate B. secondary C. primary D. undergraduate
54. After Tom's boss had assessed his performance, he was offered a written______ of
employment.
A. certificate B. licence C. contract D. degree
55. Neil Armstrong was the first person to set foot on the_________ of the moon.
A. exterior B. outside C. surface D. appearance
56. We have decided to_______ our research into environmental waste to ensure high health
standards in this city.
A. exaggerate B. broaden C. underestimate D. outweigh
57. The________ in the stadium all cheered the athletes.
A. onlookers B. spectators C. viewers D. witnesses
58. Candidates applying for this position must have a master's________ in science.
A. degree B. diploma C. license D. certificate
59. We have decided to _____________ our research into environmental waste to ensure high
health standards in this city.
A. exaggerate B. outweigh C. boost D. broaden
60. Much of our knowledge about prehistoric animals comes from the study of
A. artifacts B. ruins C. relics D. fossils
61. During the pandemic, many old people struggled to live on a small ____________
A. fee B. charge C. debt D. pension
62. Having taken good care of the customers, the waitress got a generous _________ from them.
A. fine B. tip C. money D. wage
63. When I agreed to help, I didn't know what I was _________ myself in for.
A. letting B. laying C. putting D. bringing
64. Your store needs a bold sign that will catch the _______ of anyone walking down the street.
That may help to sell more products.
A. peek B. eye C. glimpse D. flash
44
65. Our group ___________ a new method to carry out the survey into teenagers' attitudes
towards online learning.
A. recruited B. employed C. occupied D. engaged
66. Most of this growth had occurred since 1950 and was known as the population __________
A. growth B. explosion C. surplus D. density
67. Immigrants faced __________ and were paid considerably less than their white colleagues.
A. experience B. community C. integration D. discrimination
68. Human activities that release carbon dioxide into the atmosphere have ______ the rate of
global warming.
A. improved B. turned C. accelerated D. made
69. The casserole was delicious! Could you give me the ________ so that I can make it at home?
A. preparation B. recipe C. description D. formula
70. The documentary about life in the slums was really________ and was followed by a lot of
discussion about how to solve this problem.
A. well-prepared B. fast-growing C. thought-provoking D. cost-effective
71. The cat showed her__________ for the stale fish by turning her back on it.
A. disgrace B. despair C. disgust D. distress
72. Some diehard fans of the visiting team got angry at the ______ ’s final decision.
A. umpire B. athlete C. assessor D. examiner
73. Some animals, such as____snakes and spiders, inject venoms to immobilize and kill their prey.
A. venomous B. domesticated C. violent D. amphibious
74. You can seek help from ________ agencies in your area to find work.
A. employment B. career C. profession D. occupation
75. They will elect the opposition into government at the next election, without a______of a doubt.
A. shade B. shadow C. benefit D. hue
76. An effective exercise programme has three key _______ - intensity, frequency and duration.
A. elements B. components C. substances D. ingredients
77. Because fossil fuels are running out, scientists are trying to develop alternative________ of
energy. A. origins B. inputs C. products D. sources
79. Environmental groups try to stop farmers from using harmful_______ on their crops.
A. economy B. agriculture C. investments D. chemicals
80. After years of working together, the partners found themselves_____ linked.
A. permanently B. perpetually C. indelibly D. inextricably
81. The police have every good________ to believe that he is guilty.
A. cause B. excuse C. reason D. ground
82. Food and drinks which strongly_______ the body can cause stress.
A. boost B. stimulate C. motivate D. encourage
83. His boss asked him to take________ of the office for a few days while she was away.
A. responsibility B. advantage C. duty D. charge
84. It is very difficult to_______ the exact meaning of an idiom in a foreign language.
A. convert B. transfer C. convey D. exchange

45
85. It's________ knowledge that eating vegetables and doing exercise regularly do us good.
A. common B. famous C. usual D. normal
86. At the age of 11, Taylor Swift already succeeded in_______ a record deal in Nashville.
A. creating B. writing C. grabbing D. landing
87. In many Western societies, good eye________ is often highly appreciated.
A. looking B. contact C. catching D. attention
88. Many people and organizations have been making every possible effort in order to save _____
species.
A. fearful B. threatening C. endangered D. dangerous
89. Remember to wash all cooking ________ before preparing any dishes.
A. gadgets B. utensils C. devices D. applications
90. Many African and Asian nations have set aside land called_______ to protect the habitats of
elephants and other wild animals.
A. reserves B. wildlife C. species D. forest
91. A new TV show has sparked______ by showing the positive side of dropping out of college.
A. controversy B. argument C. contention D. fight
92. My uncle left his job because he did not have________ of promotion.
A. visions B. prospects C. scenarios D. posts
93. A member of a political ____ is expected to deliver a speech at the national park this evening.
A. staff B. crew C. party D. band
94. A move towards healthy eating could help_______ heart disease.
A. remove B. prevent C. terminate D. reverse
95. Apart from entertaining, art may be used as a _____________for propaganda.
A. means B. vehicle C. method D. way
96. Please turn off all your electronic ______ during takeoff.
A. materials B. applications C. equipments D. devices
97. Robots can act as 24/7 ____ to children with disabilities or the elderly.
A. devices B. contribution C. equipment D. aids
98. During the Covid-19 pandemic, bus ________ were required to wear face masks.
A. guests B. clients C. customers D. passengers
99. The site contains the remains of ancient_______ who came to the area from Asia.
A. citizens B. residents C. inhabitants D. settlers
100. Adults should take control of how children use the Internet to protect them from scams
and__________.
A. cyberbullying B. perseverance C. combustion D. solidarity

11. CHUYÊN ĐỀ: KẾT HỢP THÌ (CÂU 12, 14)


Question 12: When I came to his house, he _________ a football match on TV.
A. is watching B. has watched C. was watching D. watches
Question 14: Jack will join us____.
A. as soon as he had finished his homework B. as soon as he will finish his homework
46
C. as soon as he finished his homework D. as soon as he finishes his homework
1. Hiện tại (Đơn/ Hoàn thành) – Tương Lai Đơn
Ex: a. My son will come back his country after he has finished his courses.
b. When I see him, I will give him your regards.
c. I will wait for you until you come back.

=> WHEN /AFTER


BEFORE/UNTIL + S + Vo/s/es// (or: S + have/ has + V3/ed), S’ + will + Vo..
AS SOON AS
2. Hiện Tại Hoàn Thành – Quá khứ đơn
Ex: a. I started/began to study English when I was 9 years old.
=> I have studied English since I was 9 years old.
b. We started/began to learn English 10 years ago.
=> We have learnt English for 10 years.
3. QK Tiếp diễn – Quá khứ đơn.
Diễn tả hành động đang xảy ra thì có hành động khác xen vào.

*WHEN/While + S + +were/was Ving (quá khứ tiếp diễn), S’ + V2/ed (quá khứ đơn)
* S + were/was +Ving when (khi/thì) S’+ V2/ed.

4. QK đơn – QK hoàn thành:

a. After + S + had + V3/ed, S’ + V2/ed.


b. Before/ By the time/ As soon as + S + V2/ed, S’ + had + V3/ed...

Ex: By the time we got to the cinema, the film had already started
******
PRACTICE 1
1. While Steve Jobs _______ at high school, he met Steve Wozniak, who also loved electronics.
A. studied B. was studying C. is studying D. had studied
2. She_________ French words for hours, but she still doesn't remember all of them.
A. has been learnt B. have been learning C. have learnt D. has been learning
3. They have been friends ever since they______ in grade school.
A. are B. have been C. were D. will be
4. Alice took great photos of butterflies while she_______ in the forest.
A. hiked B. had hiked C. was hiking D. taken
5. His health has improved a lot since he ______ doing exercises regularly.
A. started B. had started C. has started D. was started
6. John________ the latest novel of a famous writer when I saw him.
A. read B. is reading C. was reading D. reading
7. John hasn't met his classmates since he _______ to work in this city.
A. will move B. was moving C. moved D. moves
8. She has been writing to him since she _____ his email.

47
A. will have B. is having C. had D. has
9. Since our last meeting most of the students ______more involved in volunteering.
A. become B. have become C. will become D. became
10. David entered the room while they ________behind his back.
A. talk B. are talking C. were talking D. talked
11. When Peter went into the room, his friends_________ a well-known song.
A. were singing B. sang C. had sung D. have been singing
12. They______ the floor when their mother came home.
A. cleaned B. were cleaning C. clean D. are cleaning
13. Helen and Tom________ at the supermarket when they met their former teacher.
A. shopped B. have shopped C. were shopping D. are shopping
14. Tom________ his homework when suddenly he heard a loud noise outside.
A. was doing B. has done C. did D. is doing
15. The whole family_______ when the thief broke into their house last night.
A. are sleeping B. sleep C. were sleeping D. slept
16. When we ______ through the countryside, we saw so many beautiful landscapes.
A. were driving B. are driving C. drive D. drove
17. When she got home, the children_______ merrily in the front yard.
A. play B. were playing C. played D. are playing
18. I was watering the plants when my brother______ home.
A. comes B. came C. had come D. was coming
19. My mother_______ dinner in the kitchen when my father returned home from work.
A. prepares B. was preparing C. is preparing D. prepared
20. I caught a glimpse of Linda when I_______ to the restaurant.
A. am going B. was going C. went D. have gone
21. Katherine_________ a sweet dream when the alarm clock went off.
A. is having B. was having C. has D. had
22. The children______ soccer happily in the playground when it started to rain.
A. were playing B. play C. are playing D. played
23. We______ interesting movie on TV when the light went out.
A. were watching B. are watching C. watch D. have watched
24. By the time you come home, I_____ the decorating.
A. will have finished B. finished C. have finished D. are finishing
25. The sun________ when I woke up this morning.
A. has shone B. shines C. was shining D. is shining
26. When I came to visit her last night, she______ dinner.
A. has had B. is having C. was having D. had had
27. He_________ TV with his family in the living when the telephone rang.
A. are watching B. was watching C. watched D. watches
28. He's been back to the office a few times since he________
A. has retired B. retires C. had retired D. retired
48
29. Mary and Peter_______ their bedroom when they came across their old wedding photos.
A. are tidying B. tidied C. were tidying D. tidy
30. Sarah_______ too fast when the accident happened.
A. was driving B. drives C. drove D. is driving
31. While we_______ in the mountains, we saw a family of deer.
A. are hiking B. hiked C. had hiked D. were hiking
32. We_______ about the future when the teacher came in.
A. were discussing B. will discuss C. has discussed D. had discussed
33. My best friend has gone to Germany for further education since she______ upper secondary
school.
A. finished B. had finished C. finishes D. is finishing
34. The students______ their test when the fire alarm went off.
A. are taking B. were taking C. took D. take
35. When the boss walked into the office, his secretary______.
A. had typed B. was typing C. has been typing D. is typing
36. My grandmother_________ in the kitchen when she saw a mouse.
A. has cooked B. was cooking C. cooks D. is cooking
37. When a strange man broke into John's house, he_______ like a log.
A. has slept B. was sleeping C. slept D. is sleeping
38. Yesterday I_______ in the park when I saw him playing football.
A. walked B. has been walking C. was walking D. is walking
39. When we came in, they ___________ the meal for us.
A. preparing B. prepare C. prepared D. were preparing
40. Paul noticed a job advertisement while he ____________ along the street.
A. was walking B. would walked C. walked D. had walked
41. When I returned from work, my dad _____________ with his childhood friends.
A. will be playing B. was playing C. have played D. is playing
42. Jack has received lots of friend requests since he ____________________ a talent contest at his
school.
A. is winning B. will win C. won D. wins
43. Since the end of the war, the Government __________ over five thousand of prisoners.
A. have released B. had released C. has released D. released
44. My cousin and I ______________ on the computer when there was a power cut.
A. had played B. played C. were playing D. am playing
45. He fell down when he _____________ towards the market.
A. runs B. run C. was running D. had run
46. By the end of next month, Mary and I _______________ out with each other for three years.
A. will be going B. have been going C. will have been going D. will go
47. Yesterday, when I ________ dinner, my best friend came over.
A. am cooking B. was cooking C. have cooked D. will cook
48. We _____ for over ten hours without a break. Let's call it a day!

49
A. will work B. have been working C. worked D. are working
49. Have they been living in the neighborhood since you ________in?
A. are moving B. have moved C. moved D. has moved
50. Bob has been working as a waiter since he ______ from high school.
A. graduates B. graduated C. had graduated D. would graduate
51. Yesterday, I saw a serious accident while I ____________ on the beach.
A. am walking B. was walking C. walk D. walked
52. She has been sitting at the computer since her husband_______ home.
A. was coming B. will come C. came D. comes
53. By the time he finishes his speech, I ______ for over one hour.
A. will have slept B. sleep C. have slept D. am sleeping
54. When some students _________ the windows, the principal entered the classroom.
A. was cleaning B. have cleaned C. are cleaning D. were cleaning
55. Mary ran into an old friend while she ______ in the park.
A. walked B. was walking C. walks D. has walked
56. When I came to visit my mom last Sunday, she______ in the kitchen.
A. had cooked B. has cooked C. was cooking D. cooked
57. Native species _____a part of a given biological landscape since they adapted to the local
environment.
A. has been B. are C. have been D. is
58. While Linda_______ last night, she got a nightmare.
A. slept B. had slept C. is sleeping D. was sleeping
59. Tom burnt his hand when he_______ dinner.
A. has cooked B. were cooking C. cooked D. was cooking
60. By the time we get there tonight, the film________.
A. will start B. will have started C. started D. starts
61. Mary _____ to catch the bus when she slipped and fell down.
A. has been running B. was running C. has run D. run
62. My mother ________ the clothes to dry outside when it suddenly started to rain.
A. was hanging B. will hang C. hangs D. is hanging
63. Don't tell us that joke again. We______ it so many times!
A. hear B. have heard C. heard D. had heard
64. Lilly________ asleep when she________ her homework yesterday.
A. fell/ was doing B. fell/ did C. was fallen/ was doing D. falls/ does
65. My father was watching TV when my friend suddenly _________to my house.
A. comes B. has come C. came D. was coming
66. Last month the old man ___________ in the hospital for some days.
A. has been B. have been C. had been D. was
67. Since Chat GPT was launched, many university students _________ advantage of it to hone
their writing skills.
A. had taken B. would take C. were taking D. have taken

50
68. While I_______ at the bus stop, three buses went by in the opposite direction.
A. had waited B. was waiting C. were waiting D. waited
69. By the time I went to bed last night, I _____ my homework.
A. were finishing B. have finished C. finished D. had already finished
70. At this time last night she was cooking and he ______ the newspaper.
A. is reading B. was reading C. is reading D. read
71. We were extremely tired at the end of the journey. We_______ for more than 10 hours.
A. had been traveling B. have been traveling C. have traveled D. were traveling
72. Bob _______ along the shady path when he saw an old friend.
A. was cycling B. has cycled C. is cycling D. would cycle
73. We_________ through the forest when we realized we were going in the wrong direction.
A. have walked B. walked C. were walking D. had walked
74. By the time Brown's daughter graduates, he_________.
A. retires B. has retired C. will being retired D. will have retired
75. It_______ heavily when we came back home.
A. is raining B. rain C. was raining D. rained
76. Yuna_______ the plants in the garden when it suddenly started pouring down.
A. is watering B. has watered C. was watering D. waters
77. He was chatting with the locals when he ______ about a secret spot to watch the sunset.
A. is learning B. has learned C. was learning D. learned
78. While they________ the new project, their boss called for a meeting.
A. were discussing B. discuss C. discussed D. are discussing
PRACTICE 2
Mark the letter A, B, C, or D on your answer sheet to indicate the correct answer to each of the
following questions.
Question 1: Emily will arrive ______.
A. after she had completed her assignment B. after she will complete her assignment
C. after she completed her assignment D. after she completes her assignment
Question 2: John will start studying for the exam ______.
A. after he finished his lunch B. when he finishes his lunch
C. before he finished his lunch D. until he is finishing his lunch
Question 3: We'll go on a trip ______.
A. after we saved enough money B. when we save enough money
C. before we saved enough money D. until we saved enough money
Question 4: I will send you some postcards, ______.
A. until I will arrive at the destination B. as soon as I arrive at the destination
C. when I arrived at the destination D. after I had arrived at the destination
Question 5: Tom will return ______.
A. once he had visited his friend B. once he will visit his friend
C. once he visited his friend D. once he visits his friend
Question 6: The party will start ______.
A. after they had set up the decorations B. after they will set up the decorations

51
C. after they set up the decorations D. after they setting up the decorations
Question 7: He will send me the design of my new house ______.
A. after he had finished it B. while he was finishing it
C. as soon as he has finished it D. before he finished it
Question 8: The package will be delivered ______.
A. once it had been processed B. once it will be processed
C. once it was processed D. once it is processed
Question 9: I'll buy the tickets for the concert ______.
A. when I get paid B. after I got paid C. before I got paid D. until I got paid
Question 10: They will give a performance ______.
A. when their school organised the prom B. once their school organises the prom
C. before their school organised the prom D. after their school had organised the prom
Question 11: The film will start ______.
A. once it will be approved by the censor board B. once it is approved by the censor board
C. once it was approved by the censor board D. once it had been approved by the censor board
Question 12: ______, she'll take a break from work.
A. after she finished this project B. when she finishes this project
C. before she had finished this project D. until she is finished this project
Question 13: The event will kick off ______.
A. once the guests had arrived B. once the guests arrive
C. once the guests will arrive D. once the guests arriving
Question 14: Laura will buy a new laptop ______.
A. after she had got her salary B. when she got her salary
C. as soon as she gets her salary D. by the time she got her salary
Question 15: Mark will book his flight ticket ______.
A. after he is saving enough money B. as soon as he saves enough money.
C. as soon as he had saved enough money D. by the he saved enough money
Question 16:______, I will call and tell you something interesting.
A. When I come home after workB. After I had come home after work
C. Before I came home after work D. While I was coming home after work
Question 17: Laura will start writing a full article ______.
A. once she collected all neccessary information
B. as soon as she collects all necessary information
C. when she had collected all necessary information
D. before she collected all necessary information
Question 18: ______, I will give him the report.
A. When he will return B. When he returns
C. Until he will return D. No sooner he returned
Question 19: The report will be submitted ______.
A. after it was reviewed B. after it will be reviewed
C. after it is reviewed D. after it reviewing
Question 20: ______, she will buy her parents a new TV.
A. When she receives her salary B. Once she received her salary
C. Until she received her salary D. After she had received her salary

52
Question 21: I will help you with your homework______.
A. before I did my own B. after I had done my own
C. as soon as I have done my own D. while I was doing my own
Question 22: ______, there will be great joy throughout the land.
A. After the war had been over. B. When the war was over
C. As soon as the war is over D. Once the war will be over
Question 23: ______, she will hold a party to celebrate the event.
A. When Mary moves into her new house B. When Mary moved into her new house
C. When Mary was moving into her new house D. When Mary had moved into hẻ new house
Question 24: The party will end ______.
A. once the music will stop B. once the music stops
C. once the music had stopped D. once the music stopping
Question 25: Hurry up or the film will have ended______.
A. when we will go to the movie B. before we go to the movie
C. by the time we went to the movie D. after we had gone to the movie
Question 26: I’m sorry about the delay, but I will be back ______.
A. after I had checked this B. as soon as I have checked this
C. while I was checking this D. when I checked this
Question 27: ______, we will travel to Quang Ninh.
A. When the epidemic is over B. When the epidemic will be over
C. When the epidemic had been over D. When the epidemic was over
Question 28: The movie will be released ______.
A. once it had been reviewed B. once it will be reviewed
C. once it is reviewed D. once it reviewing
Question 29: ______, he will take the children for a work to a nearby playground.
A. Before he finished dinner B. After he had finished dinner
C. As soon as he has finished dinner D. While he was finishing dinner
Question 30: She'll start exercising _____.
A. when she feels better B. after she is feeling better
C. before she will feel better D. until she felt better
Question 31: Don't say anything ______.
A. when you will be asked B. until you are asked
C. before you were asked D. after you had been asked
Question 32: I will help you with your homework______.
A. before I did my own B. after I had done my own
C. as soon as I have done my own D. while I was doing my own
Question 33: ______, I will reply to you at once.
A. Until I received your message B. When I was receiving your message
C. As soon as I receive your message D. After I had received your message
Question 34: ______, he will tell us about the results of the match.
A. When he arrives B. After he had arrived C. Before he arrived D. Until he arrived
Question 35: _________, we will be able to leave for the airport.
A. Before the taxi arrived B. After the taxi had arrived
C. While the taxi was arriving D. As soon as the taxi arrives

53
Question 36: He won’t understand the responsibilities of a father______.
A. as soon as he had his first child B. after he had had his first child
C. until he has his first child D. once he had his first child
Question 37: She will start her new job ______.
A. when she graduated college B. after she graduates college
C. before she graduated college D. until she graduated college
Question 38: I’m sorry about the delay, but I will be back ______.
A. after I had checked this B. as soon as I have checked this
C. while I was checking this D. when I checked this
Question 39: We’ll leave______.
A. as soon as the other passengers get on the bus.
B. when the other passengers got on the bus.
C. after the other passengers had got on the bus.
D. while the other passengers were getting on the bus.
Question 40: ______, I'll cook dinner.
A. when I get home from work B. after I got home from work
C. before I got home from work D. until I got home from work
Question 41: I'll buy the concert tickets ______.
A. when they go on sale B. after they had gone on sale
C. before they are going on sale D. until they went on sale
Question 42: He will work at his desk ______.
A. before he went to another meeting in the middle of the afternoon.
B. after he had gone to another meeting in the middle of the afternoon.
C. until he goes to another meeting in the middle of the afternoon.
D. till he go to another meeting in the middle of the afternoon.
Question 43: We are going to stay in his flat______.
A. after we had began Scotland B. while we are in Scotland
C. as soon as we were in Scotland D. when we will be in Scotland
Question 44: ______, relief supplies will be loaded on to the waiting aircraft.
A. Until we heard that the airport is open B. After we had heard that the airport is open
C. The moment we hear that the airport is open D. While we were hearing that the airport is open
Question 45: You won’t want to leave ______.
A. when you had been here for a few weeks. B. until you will have been here for a few weeks.
C. once you have been here for a few weeks. D. after you were here for a few weeks.
Question 46. We'll visit the museum ______.
A. after it opened B. when it opens C. before it had opened D. until it opened
Question 47: She will start her new job ______.
A. when she graduated college B. after she graduates college
C. before she graduated college D. until she graduated college
Question 48: ______, we will leave for the conference at the City Hall.
A. Once it stopped raining B. When it had stopped raining
C. While it was stopping raining D. As soon as it stops raining
Question 49: The teacher will give students a written assignment______.
A. until he will give them a series of lectures B. as he had given them as series of lectures.

54
C. when he gave them a series of lectures D. after he gives them a series of lectures.
Question 50: The team will begin the project ______.
A. once they will receive the necessary materials B. once they receive the necessary materials
C. once they received the necessary materials D. once they receiving the necessary materials.
12. CHUYÊN ĐỀ: COLLOCATION – CỤM TỪ CỐ ĐỊNH
Question 15: Whenever Ms. White prepares a meal, her children help her ____ the table.
A. stand B. go C. jump D. lay
*lay the table: bày bàn ăn// dọn cơm

55
56
(to be continued)
PRACTICE
1.________ awareness of environmental issues is key to keeping sustainable living at the
forefront of people's minds, and an opportunity to educate people and spread the world to help
protect nature.
A. Causing B. Raising C. Producing D. Rising
2. It's difficult to_______ accurate predictions about the effects on the environment.
A. make B. do C. go D. take
3. They have_______ some measures to control traffic jam at rush hours.
A. done B. carried C. made D. taken
4. After the interview, don't neglect the thank-you note or_______ letter.
A. follow-up B. turn-up C. start-up D. break-up
5. The university is ______ progress in the ongoing development of educational programs, services
and facilities.
A. doing B. taking C. making D. having
6. If you need to_______ the teacher's attention, just raise your hand.
A. attract B. pay C. pull D. take
7. I had watched my sister ______ an attempt to get out of an arranged marriage.
A. make B. try C. get D. do
8. Mrs. Robinson______ great pride in her cooking.
A. has B. takes C. gets D. finds
9. I get quite depressed when thinking about the damage we are _______to the environment.
57
A. having B. causing C. taking D. giving
10. The scientists introduced new farming methods which resulted in________ crops.
A. hard B. bumper C. wealthy D. successful
11. The players' protests__________ no difference to the referee's decision at all.
A. caused B. created C. made D. did
12. They________ great pride in offering the best service in town.
A. makes B. brings C. takes D. keeps
13. My uncle decided to ______ a business after retiring from his previous job.
A. work B. bring C. make D. run
14. Once again, the government_______ its promise.
A. gave B. put C. took D. broke
15. Language belongs to everyone, so most people feel that they have a right to______ an opinion
about it.
A. hold B. take C. lead D. make
16. If you _______the rules, you will be excluded from the game immediately.
A. make B. miss C. take D. break
17. The team leader______ a firm commitment to increasing spending on new facilities.
A. paid B. did C. caused D. made
18. One of the best ways of _______work experience is through an internship.
A. paying B. making C. doing D. gaining
19. Entering the meeting late, he tried to________ an excuse, still, the strict chairman of the board
told him to leave the room.
A. give B. have C. make D. do
20. I'm going to_______ a word with him about his kids' rude behaviors.
A. let B. set C. put D. have
21. With the development of technology, a lot of traditional trades have______ extinct.
A. done B. made C. given D. become
22. When he was younger, he was poor, but those days are only a________ memory now.
A. weak B. hazy C. feeble D. foggy
23. I decided to______ a complaint to the council about the noise from that factory.
A. set B. make C. take D. do
24. Environmentalists have_____ pressure on the government to finds ways to cut carbon
emissions.
A. given B. gone C. had D. put
25. A lot of generous businessmen have_______ valuable contributions to helping needy people.
A. taken B. done C. given D. made
26. She________ some very helpful suggestions but her boss rejected them all.
A. had B. gave C. made D. expressed
27. My family's rule is that every member has to_______ regular visits to our grandparents in the
countryside.
A. take B. pay C. make D. do

58
28. The player's protests________ no difference to the referee's decision at all.
A. made B. did C. created D. caused
29. The chairman has also______ his approval for an investigation into the case.
A. given B. taken C. needed D. received
30. The talks were meant to break down______ between the two groups.
A. walls B. gates C. fences D. barriers
31. Now that he's met the love of his life, he finally________ a chance at real happiness.
A. makes B. runs C. has D. keeps
32. Whether it rains or not doesn’t_______ any difference to us because we can go out for dinner.
A. take B. go C. do D. make
33. Researchers have __________ to the conclusion that your personality is affected by your genes.
A. reached B. got C. arrived D. come
34. My wife is going on her business next week, so I have to _________ the chores around house.
A. make B. hold C. take D. do
35. We're planning to ______ an offer on a beautiful house which is for sale near the town centre.
A. make B. break C. turn D. catch
36. Your store needs a bold sign that will catch the _______ of anyone walking down the street.
That may help to sell more products.
A. peek B. eye C. glimpse D. flash
37. Whenever you're planning to _______________ a new product, it's essential to provide
excellent customer service.
A. initiate B. start C. declare D. launch
38. The organization ___________ a choice to donate books to the charity last Christmas.
A. made B. did C. took D. had
39. The local clubs are making every _______ to interest more young people.
A. donation B. volunteer C. fund D. effort
40. I still ______ in touch with some of my childhood friends.
A. put B. keep C. make D. earn
41. A musical diva, according to many, should be able to have a(n) ______ on the next generations
of artists. A. reliance B. inspiration C. impact D. decision
42. Can you_______ me a favor?
A. make B. take C. do D. have
43. Don’t trust Lan; she always ________ promises.
A. makes B. keeps C. pays D. breaks
44. On hearing the news of her mother’s death, she ______ into tears.
A. burst B. broke C. ran D. set
45. They all admit that John is more qualified but Sarah _______ the advantage of more hands-on
experience.
A. has B. takes C. makes D. does
46. My class is planning to ___a farewell party before we take the national examination for GCSE.
A. take B. make C. bring D. organize

59
47. We have been working hard. Let's _______a break.
A. make B. find C. do D. take
48. The boy is________ aware that there should be a need to obey the rules of the competition.
A. far B. much C. greatly D. well
49. During the interview, you should try to_________ a good impression on your interviewer.
A. try B. give C. have D. create
50. She may still have a few fans in the world, but she is definitely past her_______.
A. abilities B. prime C. fortune D. fame
51. He caught a_______ of her in the crowd so I can't give you the details.
A. glimpse B. eye C. flash D. glance
52. In many Western societies, good eye________ is often highly appreciated.
A. looking B. contact C. catching D. attention
53. My brother really _______ pride in his achievement at university.
A. keeps B. takes C. makes D. catches
54. He has the habit of _______ notes in English lessons.
A. doing B. staying C. holding D. taking
55. I don't like networking events - spending hours trying to ______ small talk with strangers just
isn't my cup of tea.
A. make B. take C. get D. do
56. The event has ______ a lot of media attention.
A. taken B. won C. attracted D. kept
57. We have _______ significant progress in finding a cure for cancer.
A. kept B. made C. put D. taken
58. The search for a new vaccine will______ priority over all other medical research.
A. cause B. take C. make D. do
59. He always ______ the crossword in the newspaper before breakfast.
A. writes B. makes C. does D. works
60. “How did you know that he was lying?” – “It was just a ________ feeling.”
A. faint B. gut C. flair D. vain
61. The company is able to _______ a competitive advantage over its rivals by launching a new
marketing strategy.
A. make B. stay C. gain D. take
62. The heavyweight champion________ the advantage over his opponent in the last round of the
match.
A. earned B. gained C. took D. won
63. You should_______ more attention to what your teacher explains.
A. make B. pay C. get D. set
64. The expansion plans will face____ opposition from both local people and environmentalists.
A. extreme B. terrible C. fierce D. deep
65. Recent discoveries about corruption have_______ serious damage to the company's reputation.
A. made B. given C. taken D. done

60
66. Governments should ______ action to eliminate gender discrimination.
A. do B. take C. make D. give
67. He has the habit of________ notes in English lessons.
A. doing B. taking C. holding D. staying
68. It is high time we had to ______some measures to protect endangered animals.
A. use B. bring C. take D. hold
69. The government has________ tough measures to stamp out corruption in the country.
A. done B. caused C. taken D. made
70. She complained that her husband never_______ her any compliments anymore.
A. stood B. caught C. paid D. asked
71. The jury______ her compliments on her excellent knowledge of the subject.
A. did B. paid C. made D. gave
72. Analysts expect Turkey’s economic crisis to worsen after President Tayyip Erdoğan ________
top in the first round of the country’s presidential election.
A. came B. took C. had D. made

13. GIỚI TỪ
Question 16: It’s impossible ____ us to sleep because of the noise.
A. for B. on C. at D. to
=> cấu trúc: S + be Adj (for O) to Vo….

PRACTICE
Question 1. She is well- known ______ her intelligence and hard work. (well-known=famous)
A. for B. of C. by D. in
Question 2.The picture is similar ______ the one I saw yesterday.
A. of B. with C. to D. on
Question 3. I'm tired ______ working all day.
A. of B. from C. with D. at
Question 4.The book is different ______ the movie.
A. to B. from C. of D. above
Question 5. You should be fully prepared __________this very important exam.
A. of B. to C. for D. on
Question 6. He's never satisfied ______ what he has.
A. to B. of C. with D. on
Question 7. He is fond ______ jazz music.
A. off B. with C. by D. of
Question 9. I'm interested ______ learning a new language.
A. in B. of C. at D. about
Question 10. Orange juice is rich ______ vitamin C.
A. for B. of C. with D. in
Question 11. She's good ______ singing.
A. with B. in C. at D. for
Question 12. He's angry ______ me.
A. about B. at C. with D. for
61
Question 13. When I got there, the hall was full ___________ teenagers.
A. from B. of C. on D. off
Question 14. I'm not familiar ______ this place.
A. to B. with C. at D. above
Question 15. She's always worried ______ her children.
A. by B. for C. about D. for
Question 16. I'm bored ______ this movie.
A. with B. of C. at D. to
Question 17. This book is really useful______________us.
A. to B. for C. in D. with
Question 18. The cat is afraid ______ water.
A. at B. to C. of D. by
Question 19. He has invested a lot of money________that business.
A. to B. of C. with D. in
Question 20. I'm interested ______ learning a new language.
A. on B. in C. at D. of
Question 21. She is responsible ______ her younger siblings.
A. above B. to C. with D. for
Question 22. I'm keen ______ cats.
A. to B. on C. with D. x
Question 23. The house was built ______ my grandpa.
A. of B. from C. with D. by
Question 24. He is angry ______ his boss for not giving him a raise.
A. with B. at C. to D. for
Question 25. The police stopped him_________running away.
A. by B. from C. with D. of
Question 26. The teacher is proud ______ her students for their hard work.
A. of B. with C. about D. at
Question 27. I'm familiar ______ this neighborhood.
A. within B. of C. at D. with
D. for
Question 28. The book is different ______ the one I read before.
A.to B. to C. from D. with
More exercise
1. While studying, he was financially dependent________ his parents.
A. of B. to C. from D. on
2. Were you aware________the regulations against smoking in the area?
A. in B. with C. of D. about
3. The English language is rich________ vocabulary.
A. at B. in C. on D. for
4. Bob has left home and is independent________ his parents
A. on B. of C. with D. in
5. The manager is directly responsible________ the efficient running of the office
A. about B. for C. at D. in
6. His choice of future career is quite similar ________mine.
A. at B. with C. for D. to
7. English is really necessary____________ your future career.

62
A. with B. by C. for D. on
8. Travelling to the country during the holiday helps me escape ________the noise and pollution in
the city.
A. off B. with C. of D. from
Circle or underline the best answer
1) Cheques are useful (with/to/for/on) travellers.
2) Are you successful (on/in/at/to) your experiment?
3) She got back safe (for/from/with/to) her adventure.
4) Quang Linh is popular (with/for/to/in) folk song lovers.
5) I’m capable (of/with/for/to) speaking two languages.
6) Don’t worry (about/with/to/for) the money! I’ll lend you.
7) They have been waiting (for/with/to/at) the bus for half an hour.
8) I have been looking (after/for/into/at) my dog for two days but I haven’t seen it yet.
9) He spent too much money (with/on/to/in) that car.
10) Do you believe (on/at/to/in) ghosts?
11) Don’t make noise! I’m concentrating (to/on/in/at) the question.
12) She has suffered (from/to/with/about) her heart attack.
14. IDIOMS – THÀNH NGỮ 
Question 17: John can't hang out with his friends at the weekend since he needs to hit____
to prepare for the upcoming exams.
A. the papers B. the roof C. the books D. the dictionary
* hit the book: học 

PRACTICE
1. My daughters are like ___________ these days - one loves baseball and the other loves ballet.
A. chalk and cheese B. part and parcel C. ups and downs D. salt and pepper
chalk and cheese: khác nhau, khác biệt
2. The resignation of the chairman of the board came like a bolt from_______
A. the white B. the red C. the blue D. the sky
a bolt from/out of the blue: tiếng sét ngang tai (đột ngột, bất ngờ)
3. The young actor was a rough________. With some training, she'd become a superstar.
A. deal B. ride C. diamond D. edge
a rough diamond: viên ngọc thô (người có triển vọng)
4. It's so busy in my office. I don't get the chance to________ my breath after my long journey into
the office. I have to get down to work as soon as I get in.
A. trap B. catch C. grab D. keep
catch your breath: lấy hơi, nghỉ xả hơi
5. Mr. Park Hang Seo, a Korean coach, is considered a big_______ in Vietnam football.
A. egg B. bread C. cheese D. sandwich
a big cheese: nhân vật tầm cỡ, quyền lực
6. Rescuers have _______ to pull survivors out from under debris caused by the devastating
earthquake that rocked Turkey and Syria on February, 2nd.
A. taken their time B. raced against time C. made good time D. kept up with the times

63
- make good time: hoàn thành nhanh chóng (hành trình, chuyến đi,…)
7. It was the case of any port in a_______ when the supermodel went to work in my center after
she had been unemployed for 10 months due to corona pandemic.
A. typhoon B. storm C. hurricane D. tornado
any port in a storm (idiom): méo mó có hơn không
8. Even if you are rich, you should save some money for a _____ day.
A. snow B. foggy C. windy D. rainy
- for a rainy day: những ngày khó khăn
9. Sarah quit smoking two months ago. She broke a___ she had had for more than half of her life.
A. sweat B. habit C. fire D. favour
10. Anne always does whatever Frank tells her to do. She's like_______ in his hands.
A. butter B. glue C. putty D. clay
(like) putty in one’s hands: rất dễ bị kiểm soát bởi ai, ngoan ngoãn làm theo lời ai
11. wish my flat-mate wouldn’t play the guitar all day and night. It’s driving me up the ______.
A. ceiling B. sky C. wall D. ground
drive somebody up the wall: làm ai khó chịu
12. When he was asked about his girlfriend, he'd go as red as a_______.
A. chili B. tomato C. beetroot D. strawberry
- go as red as a beetroot: ngượng ngùng
13. I hope that they can find out a cure for the disease, because they are on the_______ track.
A. correct B. exact C. precise D. right
- on the right track: đi đúng hương
14. I know you are angry with your employer, but don’t burn your___. You may need their help.
A. clothes B. bridges C. forests D. houses
- burn your bridges: làm điều gì đó khiến sau này không thể quay lại tình trạng trước đó.
(Mình biết bạn rất giận ông chủ của mình, nhưng đừng nên đứt đoạn như vậy. Có thể bạn sẽ cần sự
giúp đỡ của họ)
15. In the next few days, we shall see if the ambassador can weather the political_______ caused
by his ill-advised remarks.
A. gale B. cyclone C. thunder D. storm
- weather the storm: vượt qua giai đoạn khó khăn
16. Jose had a hard time comparing the iPhone to the Samsung phone because to him they
were_______ and oranges.
A. tomatoes B. apples C. peaches D. lemons
be apples and oranges: quá khác biệt (không thể so sánh)
17. He hit the nail on the ______ when he suggested that the company needed to improve its
customer service.
A. muscle B. weight C. head D. teeth
hit the nail on the head: nói chính xác
18. When he was asked about his girlfriend, he'd go as red as a______.
A. strawberry B. chili C. beetroot D. tomato
be as red as a beetroot: đỏ tía mặt (vì xấu hổ)
64
19. I've got a busy day tomorrow, so I think I'll hit the______.
A. bag B. bed C. bedroom D. sack
- hit the sack: đi ngủ
20. The man had a face like_______ when some boys broke his car window.
A. tornado B. snowstorm C. lightning D. thunder
- somebody has a face like thunder: ai đó trông rất tức giận
21. The drugstore owner was arrested for selling liquor under the_______.
A. counter B. covers C. weather D. hammer
- under the counter: bí mật và bất hợp pháp
22. His finding broke fresh_______ in the treatment of cancer. It is now giving many cancer
victims hope of complete recovery.
A. path B. road C. ground D. course
- break new/fresh ground: có đột phá
23. The hurricane laid______ to the entire village and left the villagers homeless.
A. trash B. rubbish C. garbage D. waste
- lay waste (to) something: phá hủy, tàn phá
24. After my negative prediction for the season, I certainly________ my words when the team
started out undefeated.
A. taste B. digest C. eat D. smell
- eat your words: thừa nhận về những gì mình đã nói sai
25. After congratulating his team, the coach left, allowing the players to let their_____ down for a
while.
A. souls B. heads C. hair D. hearts
- let your hair down: thư giãn và tận hưởng
26. I poured my________ out to him and then he told all his friends what I'd said!
A. head B. mouth C. tongue D. heart
- pour your heart out to somebody: trút bầu tâm sự

27. I managed to sleep on the plane and arrived feeling as fresh as a______.
A. fruit B. maiden C. daisy D. kitten
- as fresh as a daisy: đầy năng lượng, tươi như hoa
28. The firstborn often gets the most attention and is put on a______.
A. podium B. lectern C. base D. pedestal
- put somebody on a pedestal: cưng chiều ai, xem ai là nhất và rất hoàn hảo
Tạm dịch: Con đầu lòng thường được chú ý và cưng chiều nhất.
29. The thought of jumping out of a plane makes his_______ stand on end.
A. leg B. nose C. hair D. neck
- make somebody’s hair stand on end: làm ai dựng tóc gáy
Tạm dịch: Ý nghĩ nhảy ra khỏi máy bay khiến anh dựng tóc gáy.
30. Environmentalists have__ pressure on the government to finds ways to cut carbon emissions.
A. given B. gone C. had D. put

65
- put pressure on somebody (to do something): gây áp lực lên ai
31. There are a lot of crooked people in big cities. If you don't want to be cheated, you'll need to
keep your _______ about you.
A. mind B. wits C. brain D. head
- have/keep your wits about you: cảnh giác, tỉnh táo
Tạm dịch: Có rất nhiều kẻ nham hiểm ở các thành phố lớn. Nếu bạn không muốn bị lừa, bạn sẽ
cần phải cảnh giác lên.
32. We_________ our brains but we couldn't come up with a solution.
A. tore B. racked C. hit D. picked
- rack your brain(s) (idiom): vắt óc suy nghĩ
33. I know you have been unlucky lately, Tony, but keep your_______ up; the sky will be clear
after the storm.
A. socks B. chin C. mind D. head
- keep your chin up: hãy cứ lạc quan lên (dùng để an ủi ai đó)
34. Of course, he didn't win the lottery! He was only________ your leg.
A. taking B. pulling C. making D. getting
- pull somebody’s legs: nói đùa
35. I've never really enjoyed going to the ballet or opera; they're not really my_______.
A. sweet and candy B. biscuit C. piece of cake D. cup of tea
- not somebody’s cup of tea: không phải những gì ai đó thích hoặc quan tâm
36. The five members of that group wanted to go for a trip to the Himalayas, but Jones, the sixth
member, upset the_______ cart by refusing to go.
A. carrot B. apple C. tomato D. orange
- upset the apple cart: phá hỏng hoàn toàn kế hoạch của người khác, phá đám
37. There must be something wrong with the puppy; it’s as quiet as a(n) _____ today.
A. peacock B. fly C. mouse D. ant
- as quiet as a mouse: rất im lặng
38. After the bitter divorce, she worked her........... to the bone to feed and clothe her three children.
A. depth B. eyes C. weight D. fingers
- work someone’s fingers to the bone: làm việc rất vất vả.
39. Despite all of the criticism and negative feedback, he decided to_______ to his guns and pursue
his dream of becoming an artist.
A. grab B. tick C. stick D. close
- Stick to one’s gun: giữ vững quan điểm, kiên định với lập trường của mình
40. Sometimes in a bad situation, there may still be some good things. Try not to "throw out
the______ with the bathwater".
A. child B. duck C. baby D. fish
- throw out the baby with the bathwater: vứt bỏ một cái gì quý báu cùng với một cái gì mình
không thích
41. When hearing the news, Tom tried his best to keep a________ on his surprise.
A. roof B. lid C. hat D. hood
keep a lid on something: kiềm chế cái gì

66
42. He's really got the bit between his________ so that he can pass the exam with flying colours.
A. teeth B. eyes C. hands D. legs
Get the bit between one’s teeth: kiên quyết, quyết tâm làm gì
43. Winning the cup in 1998 was just a_______ in the pan - they haven't won the match since then.
A. flash B. light C. blaze D. flame
a flash in the pan: thành công nhất thời
44. We were quite lucky because our hotel was only a______ throw away from the beach.
A. lane's B. stone’s C. gravel's D. paver’s
a stone’s throw from something: rất gần với đâu
45. There have been a few________ in the wind suggesting the economic situation might be
getting a little better.
A. birds B. straws C. roses D. flowers
- a straw in the wind: dấu hiệu của việc có thể xảy ra trong tương lai
46. Winning the cup in 1998 was just a ______ in the pan - they haven't won the match since then.
A. blaze B. flash C. flame D. light
- a flash in the pan: thành công đột ngột nhưng chỉ là nhất thời, chớp nhoáng
47. She didn't do much, but she got the lion's ___________ of the attention from the teachers.
A. part B. bite C. share D. meal
- the lion’s share: phần lớn nhất
48. He's really got the bit between his ______ so that he can pass the exam with flying colours.
A. hands B. teeth C. neck D. eyes
get the bit between one's teeth: kiên quyết, quyết tâm làm gì.
49. Mike used to be an exemplary student, but he went off the _________ after getting into drugs
A. routes B. rails C. lanes D. tracks
go off the rails: (người) mất kiểm soát, hành động không đúng mực, không chấp nhận được.
50. Don't let my mother watch any of those sad movies. She cries at the drop of a ______
A. hat B. bag C. cat D. rag
-at the drop of a hat: ngay lập tức
51. She is said to be a woman of her ________ If she says something, she'll do it.
A. letter B. promise C. word D. speech
a man/woman of his/her word: người giữ lời hứa, đáng tin cậy
52. A _____ of glory, a fury of passion that left them weak and trembling in the arms of each other.
A. fire B. burn C. blaze D. flame
a blaze of glory: vinh quang, thành công
53. Because of the impacts of the COVID-19 pandemic, the aviation industry has been in for a
_______ ride for 6 months.
A. hard B. fast C. rough D. tough
a rough ride: thời điểm khó khăn
54. Winning the cup in 1998 was just a______ in the pan; they haven’t won the match since then.
A. flame B. light C. flash D. blaze
a flash in the pan: thành công nhất thời

67
55. Few people can do creative work unless they are in the right_____ of mind.
A. trend B. tendenc C. frame D. attitude
be in the right frame of mind: có tâm trạng thích hợp
56. She's too busy to prepare for the job interview, so she'll just play it by_______.
A. nose B. mouth C. hand D. ear
-play something by ear: tùy cơ ứng biến
57. They were rooted to the ______ with utter amazement when their son returned home after 5
years living abroad.
A. path B. spot C. eye D. nest
- rooted to the spot: lặng người, đúng yên một chỗ vì quá xúc động hoặc sợ hãi
58. They will elect the opposition into government at the next election, without a______ of a
doubt.
A. shade B. shadow C. benefit D. hue
without a shadow of a doubt: không hề nghi ngờ
59. They’re hoping against _______ that the boy is still alive.
A. fact B. wish C. hope D. luck
- hope against hope (that…): tiếp tục hy vọng cho một việc rất khó xảy ra
60. Some teenagers just simply run___to their parents’ expectations as a way to express their identity.
A. against B. opposite C. counter D. contrary
be/run/go counter to something: trái ngược với cái gì
61. We were so looking forward to stretching out on the beach in the sunshine, but it________
whole time we were there.
A. poured with rain B. rained dogs and cats
C. dropped in the bucket ( như muối bỏ biển) D. made hay while the sun shined
- it poured with rain: mưa như trút nước
62. Binh's parents will certainly go_______ when he buys them a present with his first salary.
A. apples B. oranges C. cucumbers D. bananas
go bananas: nổi điên lên hoặc rất phấn khích, vui mừng
63. I won't buy that car because it has too much_______ on it.
A. wear and tear B. ups and downs C. odds and ends D. white lie
- wear and tear: hư (vì dùng lâu)
64. If you don't finish your homework early, I will breathe down your_________ all evening long
until it is done.
A. shoulder B. mouth C. neck D. heart
breathe down one’s neck: theo ai sát nút
65. She may still have a few fans in the world, but she is definitely past her_______.
A. abilities B. prime C. fortune D. fame
past your prime: qua giai đoạn thành công
66. There's been a white van sitting_______ for the past ten miles.
A. in my stomach B. on my tail C. on the move D. on the head
- on somebody’s tail: theo sau ai rất gần

68
67. You're putting the cart before the_______ of your work on Project A because the former is a
sequel to the latter.
A. ox B. dog C. buffalo D. horse
- put the cart before the horse: làm sai thứ tự
68. I think the main problem in this area is the lack of a good bus service.
- You're right. You've hit the _____ on the head.
A. nail B. wall C. lips D. hand
- hit the nail on the head: nói một điều gì cực kỳ chính xác
69. A lot of fans are disappointed to find that their idols have _____ of clay when they are caught
doing something embarrassing.
A. mouth B. weight C. teeth D. feet
have feet of clay: có khuyết điểm (nhưng đã không nhận ra)
70. How much longer do we have to wait? This is starting to get on my __________.
A. mind B. back C. nerves D. noses
get on somebody’s nerves: làm ai bực mình
71. During the first 10 minutes of the presentation, her lack of confidence ______ out a mile.
A. lay B. stuck C. kept D. met
stick out/stand out a mile: dễ thấy, nổi bật
72. y neighbors' loud music every night is driving me up the________.
A. ground B. ceiling C. roof D. wall
- drive sb up the wall (idiom): khiến ai tức giận, phát điên
73. Experts say that another outbreak of flu epidemic is on the _____.
A. cards B. boards C. papers D. days
- be on the cards: sắp xảy ra
74. He’s between ______ at the moment, so he is using the time to do some voluntary work.
A. careers B. unemployment C. unemployed D. jobs
be between jobs: tạm thời thất nghiệp
75. Just ____, Bill. Let’s think about it for a moment.
A. hold your horses B. bell the cat C. hurt a fly D. dog tired
- hold your horses: đừng vội, bình tĩnh
76. You’d better leave him to his own ________; he’s already a grown-up man.
A. devices B. instruments C. appliances D. gadgets
leave someone to their own devices: để ai tự đưa ra quyết định
77. He couldn't sleep well last night and he's not really on the______ today.
A. ball B. net C. bat D. stick
be on the ball: phản ứng nhanh nhạy
78. You need to stop sweeping your problems under the ____. Nothing will get resolved like that!
A. curtain B. pillow C. blanket D. carpet
sweep something under the carpet/rug: che đậy, che giấu cái gì
79. Hearing about people who mistreat animals makes me go hot under the ______.
A. chin B. collar C. sleeves D. vest

69
get/go hot under the collar: tức giận
80. When we arrived at this house, it was a jungle in the garden but Peter always________ and
had it looking like the gardens of a palace within six months.
A. had green fingers B. caught red handed
C. had the black sheep D. saw through rose colored spectacles.
- have green fingers: có tay chăm sóc cây (mát tay)
81. Mary was left to carry the _______ when the project ended in failure although she didn’t have
to take any responsibility for this project.
A. bucket B. tray C. can D. bag
carry the can: gánh trách nhiệm
82. The journalist really put the singer on the _______ by asking him about the scandal last year.
A. corner B. kick C. spot D. field
put somebody on the spot: làm ai bối rối, khó xử
83. Reading the article about damages caused by the devastating earthquake really brought a lump
to my_______.
A. teeth B. eye C. throat D. lip
a lump in/to somebody’s throat: cổ họng nghẹn lại (cảm giác muốn bật khóc)
84. Just a moment! I've got the answer on the______ of my tongue.
A. top B. tap C. tip D. back
- on the tip of your tongue: chắc chắn là biết thứ gì nhưng không thể nhớ ra
85. We should always learn to_______ on our own two feet and not depend on others.
A. walk B. stand C. knee D. jump
- stand on your own (two) feet: độc lập và có thể tự chăm sóc bản thân
86. That Peter was born and brought up in a rich family is as plain as the______.
A. nose on his face B. tip of his tongue C. back of his hand D. hair on his head
- as plain as the nose on your face: rõ ràng
87. The others ran off, leaving me to face the______.
A. music B. scene C. wall D. desk
- face the music: chấp nhận những lời chỉ trích, trừng phạt
88. John used a very old idiom which I can't quite bring to________.
A. home B. head C. town D. mind
Bring someone/something to mind: nhớ ra điều gì
89. My mother and I are cut from the same ______, both having a strong sense of style and
creativity when it comes to fashion.
A. shirt B. cloth C. pant D. page
- be cut from the same cloth: giống nhau, như đúc từ cùng một khuôn
90. During the first 10 minutes of the presentation, her lack of confidence______ out a mile.
A. kept B. lay C. stuck D. met
- stick out/stand out a mile: dễ thấy, rõ ràng
91. Falsely arrested and charged, in a strange country and all alone, Mark was truly at his
________ end.
A. knacks B. legs C. wits D. arms
70
- at one’s wits’ end: không còn biết làm thế nào nữa, vô phương kế
92. Amanda was refusing to give me the week off so I went over her_______ and spoke the boss.
A. head B. arm C. shoulder D. ear
- go over somebody’s head: đến gặp một người có vị trí quyền lực cao hơn bạn
93. He's really got the bit between his_______ so that he can pass the exam with flying colours.
A. teeth B. hands C. neck D. eyes
- get the bit between your teeth: kiên quyết, quyết tâm làm gì
94. I know you are upset about breaking up with Tom, but there are plenty more _________
A. horses in the stable B. tigers in the jungle C. cows in the shed D. fish in the sea
*There are plenty more fish in the sea: có nhiều lựa chọn khác nữa.
95. When Jack won the lottery, he was______.
A. over the clouds B. over the moon C. over the stars D. over the sky
96. Lisa’s new job is a______
A. piece of cake B. piece of pie C. piece of bread D. piece of muffin
97. Sam lost his keys again! He’s always losing them. He’s really______
A. on the ball B. on the fence C. off the hook D. in hot water
98. The kids were so excited about the trip to Disneyland, they were______
A. jumping for joy B. jumping for sorrow C. jumping for anger D. jumping for fear
99. Sarah has been feeling______lately. I hope she feels better soon.
A. under the rainbow B. under the weather C. under the water D. under the climate
100. When I told my friend about my problem, she told me to______
A. keep my chin down B. keep my chin up C. keep my chin left D. keep my chin
right
101. We’ve been waiting for the bus for ages. It feels like we’re waiting______
A. for ages B. for hours C. for eternity D. for days
102. Peter is always daydreaming. He’s often ______.
A. in the clouds B. in the sky C. in the ground D. in the sea
103. Don’t worry about Tom. He can take care of himself. He’s as tough as______.
A. nails B. wood C. steel D. iron
104. It’s raining cats and dogs outside! I forgot my umbrella, and now I’m completely ______.
A. dry B. soaked C. sunny D. wet
105. Anna won the race fair and square. She’s the______ of the pack.
A. top dog B. underdog C. middle dog D. stray dog
106. I can’t believe Sarah said that to me! She really hit ______.
A. below the belt B. below the waist C. below the shoulder D. below the head
107. I’ve been feeling a bit under the weather lately. I think I’m coming ____.
A. down with a cold B. down with a fever C. down with a headache D. down with a flu
108. Don’t worry, Tim. Everything will be okay. You just need to keep your ______up.
A. head B. chin C. eyes D. mouth
109. I was so nervous about the interview, but I’m glad to say I came out of it ______.
A. on top B. on bottom C. in between D. sideways
110. Sarah has been feeling a bit down ______ lately. I think she misses her family.
A. in the dumps B. in the valley C. in the hole D. in the pond
111. I was so tired last night, I slept ______.
A. like a baby B. like a cat C. like a dog D. like a bird
112. When I saw the final exam, I nearly had a ______!
71
A. heart jump B. heart attack C. heart break D. heart stop
113. Jake always talks about his big plans, but he never follows through. He’s ___ and no cattle.
A. all talk B. all show C. all bark D. all bite
114. It’s so cold outside! I feel like I’m freezing ______.
A. my legs off B. my arms off C. my fingers off D. my toes off
115. Don’t worry about the presentation, Jane. Just go for it and break a ______!
A. bone B. leg C. arm D. foot

15. (CÂU 18, 19) PHÁT ÂM – PHỤ ÂM + NGUYÊN ÂM


Question 18: A. comic B. comfort C. city D. cancer
Question 19: A. wife B. give C. mine D. smile

PRACTICE 1
PHỤ ÂM
Câu 1: A. president B. signal C. surprising D. similar
Câu 2: A. second B. become C.office D. couple
Câu 3: A. student B. school C. resort D. status
Câu 4: A. armchair B. cheerful C. machine D. children
Câu 5: A. question B. nation C. situation D. production
Câu 6: A. charity B. chemistry C. channel D. voucher
Câu 7: A. ocean B. official C. sincere D. ancient
Câu 8: A. central B. scatter C. medical D. increase
Câu 9: A. honest B. home C. honey D. hockey
Câu 10: A. dominant B. essential C. instant D. resistance
Câu 11: A. choice B. achieve C. each D. Chemistry
Câu 12: A. scholarship B. mechanic C. technology D. championship
Câu 13: A. complete B. center C. cooker D. consumer
Câu 14: A. busy B. answer C. clumsy D. noisy
Câu 15: A. chemical B. approach C. achieve D. challenge
Câu 16: A. helpful B. hour C. high D. humor
Câu 17: A. Think B. Thank C. Theater D. Therefore
Câu 18: A. accident B. contrast C. discover D. aircraft
NGUYÊN ÂM
Câu 1: A. before B. report C. repaint D. access
Câu 2: A. energy B. remain C. design D. behind
Câu 3: A. account B. against C. angry D. apply
Câu 4: A. teaspoon B. heavy C. ready D. headache
Câu 5: A. nation B. affect C. accept D. attack
Câu 6: A. state B. cave C. stand D. train
Câu 7: A. floor B. good C. stood D. foot
Câu 8: A. eleven B. elephant C. examine D. exact
Câu 9: A. species B. invent C. medicine D. tennis
Câu 10: A. deal B. teach C. break D. clean
Câu 11: A. status B. future C. during D. reduce
Câu 12: A. date B. face C. page D. map
Câu 13: A. walk B. call C. take D. talk
72
Câu 14: A. find B. think C. drive D. mind
Câu 15: A. collect B. operate C. hobby D. volume
Câu 16: A. climate B. nothing C. city D. discuss
Câu 17: A. pink B. mill C. wide D. sick
Câu 18: A. music B. union C. human D. equal
Câu 19: A. meat B. beat C. bread D. cheat
Câu 20: A. miss B. nick C. link D. shirt
Câu 21: A. village B. luggage C. engage D. damage
Câu 22: A. wife B. give C. mine D. smile
Practice 2. PHỤ ÂM
Question 1. A. thank B. thick C. that D. thing
Question 2. A. chaos B. church C. check D. child
Question 3. A. thirteen B. thanks C. think D. father
Question 4. A. water B. whale C. who D. window
Question 5. A. thumb B. themselves C. think D. thorough
Question 6. A. think B. this C. those D. there
Question 7. A. thank B. threat C. thin D. those
Question 8. A. sun B. sure C. success D. sort
Question 9. A. white B. while C. who D. which
Question 10. A. gas B. gain C. germ D. good
Question 11. A. win B. wine C. wrong D. wet
Question 12. A. switch B. stomach C. match D. catch
Question 13. A. enough B. cough C. though D. rough
Question 14. A. high B. horn C. home D. hour
Question 15. A. thing B. thought C. though D. thumb
Question 16. A. suit B. seven C. sugar D. sun
Question 17. A. hour B. honest C. heir D. hospital
Question 18. A. chorus B. cherish C. chaos D. scholar
Question 19. A. country B. cover C. carbon D. ceiling
Question 20. A. both B. tenth C. myth D. with
Question 21. A. call B. cage C. couch D. cent
Question 22. A. cast B. cite C. cold D. cup
Question 23. A. chicken B. chemist C. chocolate D. champion
Question 24: A. chat B. change C. ache D. cheer
Question 25: A. machine B. chemistry C. mechanic D. headache
Question 26: A. honour B. honest C. hour D. hello
Question 27 A. center B. clutter C. calcium D. castle
Question 28 A. sick B. second C. service D. sure
Question 29: A. thoughtful B. threaten C. therefore D. thin
Question 30 A. honest B. holiday C. home D. happiness
Question 31: A. honor B. harbor C. honey D. heavy
Question 32: A. think B. thank C. theater D. therefore

73
Question 33 A. cast B.center C. cold D. cup
Question 34: A. thickly B.theft C. theme D. though
Question 35: A. genius B.gorilla C. global D. gases
Question 36: A. pollution B.poaching C. physical D. pesticide
Question 37: A. sugar B.consume C. transfer D. obesity
Question 38: A. vegetable B.hygiene C. sugary D. longevity
NGUYÊN ÂM
Question 1. A. captain B. table C. lazy D. base
Question 2. A. vegetation B. destination C. adventure D. detective
Question 3. A. risky B. imprison C. wildlife D. victory
Question 4. A. ghost B. lost C. post D. mostly
Question 5. A. globe B. emotion C. official D. lonely
Question 6. A. discover B. awesome C. tongue D. won
Question 7. A. sense B. develop C. enlarge D. reserved
Question 8. A. criticism B. arrival C. interrupt D. primitive
Question 9. A. oxygen B. typist C. syllable D. typical
Question 10. A. fund B. unusual C. volunteer D. muddy
Question 11. A. fake B. amaze C. frame D. have
Question 12. A. none B. phone C. stone D. zone
Question 13. A. life B. like C. live D. lively
Question 14. A. ancient B. educate C. stranger D. transfer
Question 15. A. international B. nation C. national D. nationality
Question 16. A. species B. nest C. special D. helpful
Question 17. A. park B. share C. dare D. bare
Question 18. A. polite B. idea C. police D. oblige
Question 19. A. family B. father C. happy D. frankly
Question 20. A. hospital B. confidence C. biologist D. home
Question 21. A. night B. children C. shift D. quit
Question 22. A. pollute B. busy C. solution D. conclusion
Question 23. A. sacrifice B. determine C. involve D. different
Question 24. A. attitude B. wisdom C. determine D. unwise
Question 25. A. hand B. demand C. happy D. hat
Question 26. A. equal B. arrest C. reject D. envelope
Question 27. A. culture B. under C. conduct D. bushes
Question 28. A. wedding B. exchange C. guest D. ancestor
Question 29. A. most B. cost C. post D. host
Question 30. A. woman B. push C. pull D. women
Question 31. A. prizes B. since C. spirit D. conical
Question 32. A. bribe B. child C. children D. wife
Question 33. A. some B. women C. love D. money
Question 34. A. workshop B. worm C. worry D. wordless
74
Question 35. A. idiot B. idea C. idol D. ideal
Question 36. A. fame B. baby C. many D. plane
Question 37. A. hide B. pick C. shine D. like
Question 38. A. candy B. sandy C. many D. handy
Question 39. A. distribute B. tribe C. triangle D. trial
Question 40. A. with B. library C. willing D. if
Question 41. A. ask B. angry C. manager D. damage
Question 42. A. pull B. sugar C. plural D. study

2. Câu 21: TRỌNG ÂM TỪ CÓ 2 ÂM TIẾT

Question 21: A. offer B. describe C. threaten D. finish

Câu 1: A. apply B. reduce C. invent D. listen


Câu 2: A. fossil B. Cover C. Perform D. Father
Câu 3: A. Morning B. arrive C. college D. Famous
Câu 4: A. Effort B. Effect C. Deafness D. Speaker
Câu 5: A. punish B. Reason C. Promise D. combine
Câu 6: A. Appear B. Orphan C. Friendly D. Beauty
Câu 7: A. Effect B. Event C. enough D. even
Câu 8: A. Desert B. Common C. Hotel D. Temple
Câu 9: A. Express B. Effect C. Office D. regard
Câu 10: A. enjoy B. Answer C. Autumn D. Campsite
Câu 11: A. Flourish B. Season C. Product D. Today
Câu 12: A. Enough B. Early C. Entrance D. Engine
Câu 13: A. Polite B. Normal C. Number D. speaker
Câu 14: A. Garden B. Handsome C. Happen D. Explain
Câu 15: A. modern B. College C. Study D. Locate
Câu 16: A. Cartoon B. Western C. rocket D. Leader
Câu 17: A. Raincoat B. Relax C. Request D. Refer
Câu 18: A. police B. system C. woman D. novel
Câu 19: A. Although B. Agree C. Almost D. Address
Câu 20: A. Prepare B. Repeat C. Purpose D. Police
Câu 21: A. Provide B. Product C. Promote D. Produce
Câu 22: A. Include B. Instead C. Invite D. Island
Câu 23: A. Award B. Enroll C. Music D. Below
Câu 24: A. belong B. indeed C. private D. emit
Câu 25: A. apply B. obtain C. disease D. desert
Câu 26: A. contain B. victim C. panic D. nature
Câu 27: A. expand B. maintain C. approach D. comment
Câu 28: A. horror B. frighten C. study D. compare
Câu 29: A. apply B. visit C. appear D. attend
Câu 30: A. weather B. confirm C. highland D. entrance

TRỌNG ÂM CỦA TỪ CÓ 3 ÂM TIẾT


Question 20: A. popular B. dangerous C. accurate D. effective
75
Question 1: A. president B. physicist C. inventor D. property
Question 2: A. elephant B. dinosaur C. buffalo D. mosquito
Question 3: A. conference B. lecturer C. researcher D. reference
Question 4: A. represent B. intensive C. domestic D. employment
Question 5: A. minister B. dependent C. encourage D. agreement
Question 6: A. hospital B. inflation C. policy D. constantly
Question 7: A. different B. achievement C. educate D. primary
Question 8: A. certainty B. activity C. organize D. compliment
Question 9: A. delicate B. promotion C. volcanic D. resources
Question 10: A. gorilla B. interesting C. September D. opponent
Question 11: A. exciting B. impolite C. attention D. attractive
Question 20: A. popular B. dangerous C. accurate D. effective
Question 20: A. exciting B. impolite C. attention D. attractive
Question 20: A. meaningful B. suitable C. attentive D. courteous
Question 20: A. generous B. suspicious C. beautiful D. national
Question 20: A. essential B. unselfish C. attractive D. humorous
Question 20: A. terrific B. chemical C. general D. beautiful
Question 20: A. different B. polluted C. educated D. possible
Question 20. A. national B. vertical C. amateur D. athletic
Question 20. A. difficult B. impressive C. different D. exciting
Question 20. A. meaningful B. suitable C. attentive D. courteous
Question 20. A. dangerous B. various C. cultural D. important
Question 20. A. essential B. disabled C. powerful D. effective
Question 20. A. humorous B. expensive C. elegant D. national
Question 20. A. numerous B. popular C. negative D. successful
Question 20: A. difficult B. popular C. effective D. national
Question 20: A. financial B. fortunate C. marvelous D. physical
Question 20: A. possible B. cultural C. confident D. supportive
Question 20: A. medical B. essential C. dangerous D. regular
Question 20. A. familiar B. impatient C. uncertain D. arrogant
Question 20. A. natural B. capable C. magnetic D. possible
Question 20. A. addicted B. distinguished C. inficient D. talented
Question 20. A. organic B. symbolic C. popular D. refusal
Question 20. A. national B. endangered C. convenient D. dramatic
Question 20. A. outdated B. explosive C. disable D. relevant
Question 20. A. domestic B. initial C. impressive D. confident
Question 20. A. previous B. proficient C. personal D. marvelous
Question 20. A. tedious B. temporary C. suitable D. impolite

Q. 22-23. TỪ ĐỒNG NGHĨA


Question 22: The salary you receive will vary according to your qualifications and experience.
A. change B. take C. stand D. hold
Question 23: The house looks quite different now because they have made major changes to it.

76
A. small B. big C. normal D. tiny

Exercise 1. Mark the letter A, B, C or D on your answer sheet to indicate the word of phrase that
is CLOSEST in meaning to the underlined part in each of the following questions.
1. But for your support, our band couldn’t have won the Grand Music competition.
A. show B. quiz C. contest D. tour
2. In a school year in Vietnam, there are two terms called the first term and the second term.
A. grades B. levels C. semesters D. systems
3. It is advisable that everyone in the family share the household duties.
A. views B. ideas C. jobs D. chores
4. Despite her cries, no one came to provide assistance.
A. help B. suggestion C. requirement D. belief
5. Salt has been used for centuries as a method of preserving food.
A. displaying B. cooking C. conserving D. seasoning
6. When the game comes a tie, there are two overtime periods of three minutes each.
A. penalty B. draw C. score D. goal
7. School uniforms are compulsory in most of Vietnamese schools.
A. divided B. paid C. depended D. required
8. Setting and clearing the table or making bed are suitable chores for 8-to 10-years-old children.
A. appropriate B. improper C. unacceptable D. unreasonable
9. Men should share household chores with women in their families.
A. cover B. finish C. take D. split
10. Hunting for meat and burning forests for soil cause destruction to wildlife.
A. organization B. damage C. protection D. contamination
11. Many women sacrifice their career for their family.
A. work B. unemployment C. occupation D. interest
12. Mother Teresa devoted herself to caring for the sick and the poor.
A. spent B. contributed C. sacrificed D. dedicated
13. The old blood cells are broken down by the spleen and eliminated from the body.
A. cut out B. exhaled C. removed D. held
14. Ha Long Bay is well-known for its beauty. That’s why it receives many visitors every year.
A. famous B. responsible C. unknown D. ideal
15. The medical community continues to make progress in the fight against cancer.
A. improvement B. speed C. expectation D. treatment
16. That competition was sponsored by the host country.
A. limited B. finished C. financed D. planned
17. Computers are recent accomplishments in our time.
A. structures B. achievements C. calculations D. inventions
18. Those people who have enough courage are likely to be successful.
A. energy B. motivation C. bravery D. desire
19. It’s vital that our children’s handwriting should be clear.
A. required B. compulsory C. essential D. obliged
20. Young people are now getting more and more concerned about environmental problems.
A. worried B. nervous C. hopeless D. uneasy
21. In Western culture, it is polite to maintain eye contact during a conversation.
A. irresponsible B. courteous C. secure D. informal
22. The most important thing is to keep yourself occupied during the social distancing time.

77
A. relaxed B. comfortable C. free D. busy
23. The repeated commercials on TV distract many viewers from watching their favorite films.
A. advertisements B. game shows C. contests D. news
24. The differences between British and American English are comparatively small.
A. relatively B. extremely C. surprisingly D. straightly
25. Every month, the volunteer group go to remote and mountainous areas to help those in need.
A. empty B. faraway C. crowded D. poor
26. Whatever his shortcomings as a husband, he was a good father to his children.
A. strengths B. weaknesses C. abilities D. good sides
27. The government needs to seek a solution to prevent domestic violence as soon as possible.
A. recreation B. remedy C. decision D. technique
28. All of these treatments are supposed to cleanse your body and stimulate your immune system.
A. encourage B. generate C. increase D. expand
29. She has shown herself to be a highly competent manager.
A. ambitious B. unqualified C. proficient D. decisive
30. A popular way to relieve stress in today's busy lifestyles is practising meditation.
A. reduce B. relax C. repeat D. require
31. We're surprised to hear that his musical talent was nurtured by their loving parents when he
was a child.
A. supported B. limited C. fostered D. restricted
32. This is the instance where big, obvious non-verbal signals are appropriate.
A. place B. situation C. time D. issue
33. Today’s scientists have overcome many of the challenges of the depth by using more
sophisticated tools.
A. common B. modern C. simple D. complicated
34. This game show has attracted many participants since it was aired in 2015.
A. stopped B. designed C. introduced D. broadcasted
35. In the 20th century, most of the traditional attitudes to remarriage are changing.
A. conventional B. contemporary C. latest D. new
36. The organization was established to better conditions for the disableD.
A. accept B. improve C. change D. prepare
37. They all encouraged my creativity and inspired me in many ways.
A. overlooked B. motivated C. rejected D. disappointed
38. Land erosion is mainly caused by widespread deforestation.
A. afforestation B. logging C. lawn mowing D. reforestation
39. Just by coincidence, I met my old classmate after 20 years.
A. sight B. surprise C. chance D. luck
40. I only applied for this business with a view to accumulating first-hand experience.
A. describing B. sharing C. lacking D. gaining

TỪ TRÁI NGHĨA- TỪ ĐƠN – CÂU 24 ĐỀ MH


Mark the letter A, B, C, or D on your answer sheet to indicate the word(s) OPPOSITE in
meaning to the underlined word(s) in each of the following questions.
Question 24: New laws to conserve the wildlife in this area have been passed.
A. protect B. maintain C. preserve D. destroy

78
Mark the letter A, B, C, or D on your answer sheet to indicate the word(s) OPPOSITE in
meaning to the underlined word(s) in each of the following questions.
Question 1. School uniform is compulsory in most of Vietnamese school.
A. depended B. optional C. obligatory D. required
Question 2. Names of people in the book were changed to preserve anonymity.
A. cover B. conserve C. presume D. reveal
Question 3. A large city such as Chicago would be called an urban area.
A. metropolitan B. rustic C. suburban D. sophisticated
Question 4. We arrived home safe and sound.
A. healthy B. unsound C. insound D. dissound.
Question 5. He revealed his intentions of leaving the company to the manager during the office
dinner party.
A. disclosed B. concealed C. misled D. influenced
Question 6. Slang can be defined as a set of lexical, grammatical, and phonological regularities
used in informal speech.
A. informative B. official C. situational D. casual
Question 7. Adverse weather conditions made it difficult to play the game.
A. favorable B. bad C. comfortable D. severe
Question 8. The Red Cross is an international humanitarian agency dedicated to reducing the
sufferings of wounded soldiers, civilians and prisoners of war.
A. sadness B. loss C. happiness D. pain and sorrow
Question 9. They are launching a campaign to promote awareness of environmental issues.
A. encourage B. publicize C. hinder D. strengthen
Question 10. We have to employ extra staff to deal with the increased workload.
A. approach B. dismiss C. interview D. meet
Question 11. The only means of access to the station is through a dark subway.
A. arrival B. admission C. outlet D. output
Question 12. Heavy rain makes driving on the road very difficult.
A. storm B. torrential C. storm D. light
Question 13. The government is being widely criticized in the media for falling to limit air
pollution.
A. praised B. attracted C. blamed D. approved
Question 14. Never punish your children by hitting them. This might teach them to become
hitters.
A. bring B. accept C. give D.reward
Question 15. The problem is due to discipline, or, more precisely, the lack of discipline, in school.
A. informally B. flexible C. casually D. wrongly
Question 16. The council has spent an enormous amount of money on this project.
A. tiny B. thin C. loose D. gigantic
Question 17. The loss of his journals had caused him even more sorrow than his retirement from
the military six years earlier.
A. grief B. joy C. comfort D. sympathy
Question 18. If we use robots instead of humans, many people may be out of work.
A. employed B. jobless C. inemployed D. unemployed
Question 19. This speedy and secure service of transferring money can be useful.
A. careful B. rapid C. slow D. hurried

79
Question 20. Our well- trained staff are always courteous to customers.
A. helpful B. friendly C. polite D. rude
Question 21. I think it’s impossible to abolish school examinations. They are necessary to
evaluate students’ progress.
A. stop B. extinguish C. continue D. organize
Question 22. We managed to get to school in time despite the heavy rain.
A. earlier than a particular moment B. later than expected
C. early enough to do something D. as long as expected
Question 23. The palace was badly damaged by fire, but was eventually restored to its original
splendor.
A. refurbished B. devastated C. strengthened D. renovated
Question 24. After the marriage, Ruth decided to settle permanently in New York.
A. temporarily B. constantly C. sustainably D. regularly
Question 25. The first year at university was probably the best and most challenging year of my
life. It caused me plenty of troubles.
A. tricky B. tough C. easy D. difficult
Question 26. Ships crossing the oceans can receive signals from satellites that enable them to
calculate their position accurately.
A. carelessly B. imprecisely C. uneasily D. untruthfully
Question 27. He is very absent – minded. He is likely to forget things or to think about something
different from what he should be thinking about.
A. retentive B. unforgettable C. old – fashioned D. easy – going
Question 28. We ought to keep these proposals secret from the chairman for the time being.
A. lively B. frequented C. accessible D. revealed
Question 29. When being interviewed, You should focus on what the interviewer is saying or
asking you.
A. to pay no attention to B. be interested in
C. be related D. express interested in
Question 30. They've always encouraged me in everything I've wanted to do.
A. unpardoned B. misconstrue C. discouraged D. impaired
Question 31. Punctuality is imperative in your new job.
A. Being efficient B. Being courteous C. Being cheerful D. Being late
Question 32. Travel insurance is sometimes mistaken for temporary health insurance, but the two
are
actually different.
A. mutable B. permanent C. passing D. transitory
Question 33. All children can attend without paying fees at state schools.
A. primary schools B. secondary schools C. high schools D. independent schools
Question 34. That is a well-behaved boy whose behaviour has nothing to complain about.
A. behaving improperly B. behaving nice C. good behavior D. behaving cleverly
Question 35. “Mary, I think these clothes are inappropriate for this important celebration.”
A. suitable B. improper C. attractive D. available
Question 36. Despite having a bigger and cheaper choice of healthy foods, many Americans have
lost a lot of weight.
A. acquire B. win C. gained D. obtain
Question 37. No one knew precisely what would happen to human being in space.
A. informally B. Flexibly C. wrongly D. casually

80
Question 38. A lot of people think that Angelina Jolie is really hot.
A. cool B. unattractive C. memorable D. beautiful
Question 39. Nonfat milk has slightly less fat than low fat.
A. a little B. a few C. much D. small
Question 40. I must have a watch since punctuality is imperative in my new job.
A. being on time B. being cheerful C. being sufficient D. being late

TỪ TRÁI NGHĨA – THÀNH NGỮ - CÂU 25 ĐỀ MH

Mark the letter A, B, C, or D on your answer sheet to indicate the word(s) OPPOSITE in meaning
to the underlined word(s) in each of the following questions.
Question 25: His resignation came as a bolt from the blue. We were all very surprised.
A. accidentally B. suspiciously C. predictably D. incredibly
* as a bolt from the blue: rất bất ngờ >< predictably: có thể đoán trước.
Mark the letter A, B, C, or D on your answer sheet to indicate the word(s) OPPOSITE in
meaning to the underlined word(s) in each of the following questions.
1. I don’t know what they are going to ask in the job interview. I’ll just play it by ear.
A. plan well in advance B. be careful about it
C. listen to others saying D. do not plan beforehand
2. It’s quite disappointing that many bystanders still turn a blind eye to acts of injustice they
witness in the street.
A. take no notice of B. show respect for C. pay attention to D. have no feeling for
3. When you’re new in a school, it’s easiest to just go with the flow for a while, and see what
people are like.
A. follow the common rules B. do what other people are doing
C. follow the current trends D. do something different from other people
4. Julie’s slimming attempts have been going by fits and starts. She needs to do it consistently.A.
slowly B. poorly C. steadily D. flexibly
5. I should keep a stiff upper lip and take the high road and all that, so I will.
A. be talkative B. be calm C. be quiet D. be nervous
6. Tom may get into hot water when driving at full speed after drinking wine.
A. get into trouble B. stay safe C. fall into disuse D. keep calm
7. Most families in my village have been on the breadline since the covid-19 pandemic broke
out.
A. healthy B. exhausted C. poor D. wealthy
8. John has been unemployed for the past three years – he’s really in dire straits.
A. in a serious situation B. in a bad situation
C. in a difficult situation D. in a pleasant situation
9. We had a whale of time as everything was quite fantastic.
A. felt disappointed B. had little time to play
C. felt happy D. had a lot time to play

81
10. He's a very good worker but he's sometimes a bit slow on the uptake. You have to explain
everything twice.
A. hear things easily B. understand things easily
C. understand things with difficulty D. hard of hearing
11. As a newspaper reporter, she always wanted to get information at first hand.
A. directly B. indirectly C. slowly D. easily
12. If you need me, just call me. I can come at the drop of a hat.
A. immediately B. reluctantly C. instantly D. occasionally
13. Our new house is located only a stone’s throw from the beach, allowing us to easily enjoy the
sun and surf whenever we want.
A. a short distance away B. within close proximity C. a long way D. near
14. I’m sure it won’t rain, but I’ll take an umbrella just to be on the safe side.
A. careful B. easy C. careless D. difficult
15. When I suggested he was mistaken, John got hot under the collar and stormed out of the
room. A. got emotional B. became furious C. remained calm D. felt anxious
16. The Internet is a treasure trove of information, which offers knowledge on any given topic
under the sun.
A. densely B. scarcely C. abundantly D. undoubtedly
17. You’d think he would have blown his lid when he realized the chef was downstairs.
A. become tired B. felt happy C. stayed calm D. been furious
18. At first no one believed she was a pilot, but her documents lent colour to her statements.
A. provided evidence for B. got information from
C. borrowed colour from D. gave no proof of
19. Julie’s slimming attempts have been going by fits and starts. She needs to do it consistently.
A. slowly B. poorly C. steadily D. flexibly
20. The new manager put the cat among the pigeons by suggesting that the staff might have to
come to work earlier than usual.
A. made a lot of people satisfied B. made a lot of people annoyed
C. made other people disappointed D. made other people nervous
21. Don't tell Jane anything about the surprise party for Jack. She has got a big mouth.
A. can't eat a lot B. hates parties C. talks too much D. can keep secrets
22. He didn’t bat an eye when he realized he failed the exam again.
A.didn’t want to see B. showed surprise C. wasn’t happy D. didn’t care
23. Once you choose to be in showbiz, you must learn to shut your ears to all rumours.
A. pay attention B. be indifferent C. refuse to listen D. be in reaction
24. Though I persuaded my boss to solve a very serious problem in the new management system,
he just made light of it.
A. completely ignored B. treated as important
C. disagreed with D.discovered by chance
25. His resignation came as a bolt from the blue, we were all very surprised.
A. accidentally B. suspiciously C. predictably D. incredibly
26. They are always optimistic although they don’t have a penny to their name.
82
A. are very poor B. are very rich C. are very mean D. are very healthy
27. The teacher will see red when he finds out you haven't done your homework again!
A. be maddened B. annoy C. make things difficult D. be calm
28. She was extremely pleased that her new books were selling like hot cakes.
A. money burns a hole in one’s pocket B. like a streak of lightning
C. go out of business D. go into business
29. He was too wet behind the ears to be in charge of such a difficult task.
A. full of sincerity B. full of experience
C. without money D. lack of responsibility
30. My brother has become a bit of couch potato since he stopped playing football.
A. a nice person B. a kind person C. a live wire D. a lazy person
31. A final agreement hasn’t been reached yet because they are at loggerheads over the cost. A.
think in a creative way B. are in disagreement
C. share the viewpoint D. become totally exhausted
32. When you're new in a school, it's easiest to just go with the flow for a while, and see what
people are like.
A. follow the common rules B. do what other people are doing
C. follow the current trends D. do something different from other people
33. He is very stubborn but flattery usually works like a charm on him. .
A. is completely successful in B. takes effect
C. turned out to be a disaster D. has no effect on
34. The best hope of avoiding a down-market tabloid TV future lies in the pressure currently
being put on the networks to clean up their act.
A. expensive B. cheap C. inferior D. economical
35. He bent over backwards to please his new girlfriend but she never seemed satisfied.
A. gave someone a nice surprise B. tried to do something which is easy
C. did some physical exercise D. tried to do something impossible
36. I’m sure it won’t rain, but I’ll take an umbrella just to be on the safe side.
A. careful B. easy C. careless D. difficult
37. Sorry, I can't come to your party. I am snowed under with work at the moment.
A. free from B. interested in C. relaxed about D. busy with
38. Tony has been under the gun in the past few days because he’s going to have a job interview
tomorrow.
A. having bought a gun B. feeling extremely calm
C. having a lot of pressure D. feeling badly upset
39. A final agreement hasn’t been reached yet because they are at loggerheads over the cost. A.
think in a creative way B. are in disagreement
C. share the viewpoint D. become totally exhausted
40. I think we cannot purchase this device this time as it costs an arm and a leg.
A. is cheap B. is painful C. is confusing D. is expensive
41. We managed to get to school in time despite the heavy rain.

83
A. earlier than a particular moment B. later than expected
C. early enough to do something D. as long as expected
42. James may get into hot water when driving at full speed after drinking.
A. get into trouble B. fall into disuse C. remain calm D. stay safe
43. My brother has become a bit of couch potato since he stopped playing football.
A. a nice person B. a kind person C. a live wire D. a lazy person
44. He was too wet behind the ears to be in charge of such a difficult task.
A. full of sincerity B. full of experience
C. without money D. lack of responsibility
45. The new manager put the cat among the pigeons by suggesting that the staff might have to
come to work earlier than usual.
A. made a lot of people satisfied B. made a lot of people annoyed
C. made other people disappointed D. made other people nervous
46. His life hung in the balance when he had a massive stroke.
A. was certain B. worsened a lot C. was unsuccessful D. changed quickly
47. Thomas was like a bear with a sore head when he wasn’t chosen for the football team.
A. totally disappointed B. in a bad mood
C. in an angry state D. extremely delighted
48. Faced with the economic crisis, many businesses were tightening their belts and cutting jobs.
A. be generous with money B. be careful with money
C. put on tighter belts D. dress in baggy clothes
49. If you are at a loose end this weekend, I will show you around the city.
A. free B. confident C. occupied D. reluctant
50. Wendy is on the horns of a dilemma: she just wonders whether to go for a picnic with her
friends or to stay at home with her family.
A. unwilling to make a decision B. able to make a choice
C. eager to make a plan D. unready to make up her mind
51. The nominating committee always meet behind closed doors, lest its deliberations become
known prematurely.
A. privately B. safely C. publicly D. dangerously
52. The year-end party was out of this world. We had never tasted such delicious food.
A. enormous B. terrific C. strange D. awful

CÂU ĐIỀU KIỆN, CÂU ƯỚC = CÂU 43 ĐỀ MH

Question 43: He didn’t try his best. He didn’t make much progress.
A. If he had tried his best, he can have made a lot of progress.
B. If he had tried his best, he can't have made much progress.
C. If he had tried his best, he couldn’t have made much progress.
D. If he had tried his best, he could have made a lot of progress.

Mark the letter A, B, C, or D on your answer Sheet to indicate the sentence that best combines
each pair of sentences in the following questions.
1. Mary forgot her homework. She got a detention.
84
A. If Mary forgets her homework, she will get a detention.
B. If Mary had forgotten her homework, she wouldn't have got a detention.
C. If Mary forgets her homework, she wouldn't get a detention.
D. If Mary forgot her homework, she gets a detention.
2. Tom didn't listen to his mother's advice. He got into trouble.
A. If Tom listens to his mother's advice, he will get into trouble.
B. If Tom didn't listen to his mother's advice, he gets into trouble.
C. If Tom had listened to his mother's advice, he wouldn't have got into trouble.
D. If Tom didn't listen to his mother's advice, he would get into trouble.
3. The students didn't study for the exam. They failed it.
A. If the students study for the exam, they fail it.
B. If the students had studied for the exam, they wouldn't have failed it.
C. If the students didn't study for the exam, they fail it.
D. If the students didn't study for the exam, they failed it.
4. Emily didn't wear her seatbelt. She got a ticket.
A. If Emily wears her seatbelt, she gets a ticket.
B. If Emily had worn her seatbelt, she wouldn't have got a ticket.
C. If Emily doesn't wear her seatbelt, she gets a ticket.
D. If Emily didn't wear her seatbelt, she will get a ticket.
5. John didn't turn off the lights. The electricity bill was high.
A. If John turns off the lights, the electricity bill is high.
B. If John had turned off the lights, the electricity bill wouldn't have been high.
C. If John didn't turn off the lights, the electricity bill would be high.
D. If John didn't turn off the lights, the electricity bill was high.
6. Laura didn't bring an umbrella. She got soaked in the rain.
A. If Laura brings an umbrella, she won't get soaked in the rain.
B. If Laura had brought an umbrella, she wouldn’t have got soaked in the rain.
C. If Laura doesn't bring an umbrella, she gets soaked in the rain.
D. If Laura didn't bring an umbrella, she will get soaked in the rain.
7. Michael didn't wear sunscreen. He got sunburnt.
A. If Michael wears sunscreen, he gets sunburnt.
B. If Michael had worn sunscreen, he wouldn't have got sunburnt.
C. If Michael doesn't wear sunscreen, he will get sunburnt.
D. If Michael didn't wear sunscreen, he would get sunburnt.
8. Sarah didn't charge her phone. It died during the trip.
A. If Sarah charges her phone, it dies during the trip.
B. If Sarah had charged her phone, it wouldn't have died during the trip.
C. If Sarah doesn't charge her phone, it will die during the trip.
D. If Sarah didn't charge her phone, it would die during the trip.
9. David didn't follow the recipe. The dish didn't turn out well.
A. If David follows the recipe, the dish doesn't turn out well.
B. If David had followed the recipe, the dish would have turned out well.
C. If David didn't follow the recipe, the dish wouldn't turn out well.
D. If David didn't follow the recipe, the dish doesn't turn out well.
10. Lisa didn't lock the door. The house was burgled.
A. If Lisa locks the door, the house is burgled.
B. If Lisa had locked the door, the house wouldn't have been burgled.

85
C. If Lisa doesn't lock the door, the house will be burgled.
D. If Lisa didn't lock the door, the house would be burgled.
11. Sarah forgot her umbrella. She got drenched in the rain.
A. If Sarah forgets her umbrella, she gets drenched in the rain.
B. If Sarah had forgotten her umbrella, she wouldn't have got drenched in the rain.
C. If Sarah forgets her umbrella, she got drenched in the rain.
D. If Sarah forgot her umbrella, she gets drenched in the rain.
12. Tom didn't lock his bike. It got stolen.
A. If Tom locks his bike, it wouldn't have got stolen.
B. If Tom had locked his bike, it wouldn't have got stolen.
C. If Tom doesn't lock his bike, it will get stolen.
D. If Tom didn't lock his bike, it will get stolen.
13. The students didn't study for the exam. They failed it.
A. If the students study for the exam, they fail it.
B. If the students had studied for the exam, they wouldn't have failed it.
C. If the students didn't study for the exam, they fail it.
D. If the students didn't study for the exam, they failed it.
14. Emily didn't set her alarm. She overslept.
A. If Emily sets her alarm, she wouldn't have overslept.
B. If Emily hadn't set her alarm, she wouldn't have overslept.
C. If Emily doesn't set her alarm, she will oversleep.
D. If Emily didn't set her alarm, she will oversleep.
Question 43. John didn't wear his helmet. He sustained a head injury.
A. If John wears his helmet, he sustains a head injury.
B. If John had worn his helmet, he wouldn't have sustained a head injury.
C. If John doesn't wear his helmet, he will sustain a head injury.
D. If John didn't wear his helmet, he would sustain a head injury.
15. Maria didn't save her work. Her computer crashed.
A. If Maria saves her work, her computer wouldn't have crashed.
B. If Maria hadn't saved her work, her computer wouldn't have crashed.
C. If Maria doesn't save her work, her computer will crash.
D. If Maria didn't save her work, her computer will crash.
16. They didn't bring a map. They got lost.
A. If they bring a map, they won't get lost.
B. If they hadn't brought a map, they wouldn't have got lost.
C. If they don't bring a map, they will get lost.
D. If they didn't bring a map, they would get lost.
17. David didn't follow the recipe. The dish didn't turn out well.
A. If David follows the recipe, the dish doesn't turn out well.
B. If David had followed the recipe, the dish would have turned out well.
C. If David didn't follow the recipe, the dish wouldn't turn out well.
D. If David didn't follow the recipe, the dish doesn't turn out well.
18. Lisa didn't charge her phone. It died during the trip.
A. If Lisa charges her phone, it wouldn't have died during the trip.
B. If Lisa hadn't charged her phone, it wouldn't have died during the trip.
C. If Lisa doesn't charge her phone, it will die during the trip.
D. If Lisa didn't charge her phone, it will die during the trip.

86
19. Jason didn't wear his seatbelt. He got fined.
A. If Jason wears his seatbelt, he gets fined.
B. If Jason had worn his seatbelt, he wouldn't have got fined.
C. If Jason doesn't wear his seatbelt, he will get fined.
D. If Jason didn't wear his seatbelt, he will get fined.
20. Sarah forgot her umbrella. She got soaked in the rain.
A. If Sarah forgets her umbrella, she gets soaked in the rain.
B. If Sarah had forgotten her umbrella, she wouldn't have got soaked in the rain.
C. If Sarah forgets her umbrella, she got soaked in the rain.
D. If Sarah forgot her umbrella, she gets soaked in the rain.
21. Tom didn't lock his bike. It was stolen.
A. If Tom locks his bike, it wouldn't have been stolen.
B. If Tom had locked his bike, it wouldn't have been stolen.
C. If Tom doesn't lock his bike, it will be stolen.
D. If Tom didn't lock his bike, it will be stolen.
22. The students didn't study for the exam. They failed it.
A. If the students study for the exam, they fail it.
B. If the students had studied for the exam, they wouldn't have failed it.
C. If the students didn't study for the exam, they fail it.
D. If the students didn't study for the exam, they failed it.
23. Emily didn't set her alarm. She overslept.
A. If Emily sets her alarm, she wouldn't have overslept.
B. If Emily hadn't set her alarm, she wouldn't have overslept.
C. If Emily doesn't set her alarm, she will oversleep.
D. If Emily didn't set her alarm, she will oversleep.
24. John didn't wear his helmet. He sustained a head injury.
A. If John wears his helmet, he sustains a head injury.
B. If John had worn his helmet, he wouldn't have sustained a head injury.
C. If John doesn't wear his helmet, he will sustain a head injury.
D. If John didn't wear his helmet, he would sustain a head injury.
25. Maria didn't save her work. Her computer crashed.
A. If Maria saves her work, her computer wouldn't have crashed.
B. If Maria hadn't saved her work, her computer wouldn't have crashed.
C. If Maria doesn't save her work, her computer will crash.
D. If Maria didn't save her work, her computer will crash.
26. They didn't bring a map. They got lost.
A. If they bring a map, they won't get lost.
B. If they hadn't brought a map, they wouldn't have got lost.
C. If they don't bring a map, they will get lost.
D. If they didn't bring a map, they would get lost.
27. David didn't follow the recipe. The dish didn't turn out well.
A. If David follows the recipe, the dish doesn't turn out well.
B. If David had followed the recipe, the dish would have turned out well.
C. If David didn't follow the recipe, the dish wouldn't turn out well.
D. If David didn't follow the recipe, the dish doesn't turn out well.
28. Lisa didn't charge her phone. It died during the trip.
A. If Lisa charges her phone, it wouldn't have died during the trip.

87
B. If Lisa hadn't charged her phone, it wouldn't have died during the trip.
C. If Lisa doesn't charge her phone, it will die during the trip.
D. If Lisa didn't charge her phone, it will die during the trip.
29. Jason didn't wear his seatbelt. He got fined.
A. If Jason wears his seatbelt, he gets fined.
B. If Jason had worn his seatbelt, he wouldn't have got fined.
C. If Jason doesn't wear his seatbelt, he will get fined.
D. If Jason didn't wear his seatbelt, he will get fined.
30. Sarah forgot her umbrella. She got caught in the rain.
A. If Sarah forgets her umbrella, she gets caught in the rain.
B. If Sarah had forgotten her umbrella, she wouldn't have got caught in the rain.
C. If Sarah forgets her umbrella, she got caught in the rain.
D. If Sarah forgot her umbrella, she gets caught in the rain.

OTHER TYPES OF CONDITIONAL SENTENCES


Mark the letter A, B, C, or D to indicate the sentence that is closest in meaning to each of the
following questions.
1. I didn't have an umbrella with me, so I got wet.
A. Since I got wet, I didn't have an umbrella with me.
B. My umbrella helped me to get wet.
C. I wouldn’t have got wet if I had had an umbrella with me.
D. I got wet, so I didn't have an umbrella.
2. Unless you leave me alone, I'll call the police.
A. I'll call the police because you leave me alone.
B. I'll call the police if you don't leave me alone.
C. If you leave me alone, I'll call the police.
D. You leave me alone, so I'll call the police.
3. If I had known the reason why she was absent from class, I would have told you.
A. I knew the reason why she was absent from class, but I didn't tell you.
B. Unless I knew the reason why she was absent from class, I wouldn't tell you.
C. I didn't know the reason why she was absent from class, so I didn't tell you.
D. Although I knew the reason why she was absent from class, I didn't tell you.
4. But for your carelessness, you could have been a partner in the firm.
A. If it hadn't been your carelessness, you could have been a partner in the firm.
B. Your carelessness was only thing to prevent being a partner in the firm.
C. It was your carelessness that made you impossible to be a partner in the firm
D. You could have been a partner in the firm, but you were so careless.
5. She is so busy that she can't come to the party.
A. If she were not so busy, she must come to the party.
B. If she were not so busy, she need to come to the party.
C. If she were not so busy, she could come to the party.
D. If she were not so busy, she should come to the party.
6. We could not handle the situation without you.
A. You didn't help us handle the situation.
B. If you had not helped us, we could not have handled the situation.
C. If you did not help us, we could not handle the situation.
D. We will handle the situation if you help.

88
7. Unless you have tickets you can't come in.
A. You can't come in provided that you have tickets.
B. You can come in provided that you have tickets.
C. If you didn't have tickets, you couldn't come in.
D. Unless you don't have tickets, you can come in.
8. He stepped on the mine, and it exploded.
A. If he doesn’t step on the mine, it doesn’t explode.
B. If he doesn’t step on the mine, it won’t explode.
C. If he didn’t step on the mine, it wouldn’t explode.
D. If he hadn’t stepped on the mine, it wouldn’t have exploded.
9. It may rain this afternoon. I hope it doesn’t because I don’t want the match to be cancelled.
A. If it rains, the match is cancelled.
B. If it rains, the match will be cancelled.
C. if it rained, the match would be cancelled.
D. If it had rained, the match would have been cancelled.
10. Unfortunately, I don’t know philosophy, so I can’t answer your question.
A. If I know Philosophy, I can answer your question.
B. If I know Philosophy, I will be able to answer your question.
C. If I knew Philosophy, I would be able to answer your question.
D. If I had known Philosophy, I would have been able to answer your question.
11. We survived that accident because we were wearing our seat belts.
A. But for our seat belts, we would have survived that accident.
B. Had we not been wearing our seat belts, we wouldn't have survived that accident.
C. Without our seat belts, we could have survived that accident.
D. If we weren't wearing our seat belts, we couldn't have survived that accident.
12. They were late for the meeting because of the heavy snow.
A. If it snowed heavily, they would be late for the meeting.
B. Had it not snowed heavily, they would have been late for the meeting.
C. But for the heavy snow, they wouldn't have been late for the meeting.
D. If it didn't snow heavily, they wouldn't be late for the meeting.
13. He was successful in his career thanks to his parents' support.
A. Had it not been for his parents' support, he wouldn't be successful in his career.
B. If his parents hadn't supported him, he wouldn't have been successful in his career.
C. But for his parents' support, he wouldn't be successful in his career.
D. Without his parents' support, he would have been successful in his career.
14. They cancelled all the sporting events because of the heavy rain.
A. Without the heavy rain, they wouldn't cancel all the sporting events.
B. If it hadn't rained heavily, they would have cancelled all the sporting events.
C. If it didn't rain heavily, they wouldn't cancel all the sporting events.
D. Had it not rained heavily, they wouldn't have cancelled all the sporting events.
15. If you had arrived, you would have had the chance of meeting the chairman.
A. You hadn't arrived so you hadn't had the chance of meeting the chairman.
B. You didn't have the chance of meeting the chairman because you didn't arrive.
C. If you arrived, you would the chance of meeting the chairman.
D. You don't arrive so you don't have the chance of meeting the chairman.
16. Get in touch with me as soon as possible if you change your mind about the trip.
A. Should you change your mind about the trip, contact me as soon as possible.

89
B. If you changed your mind about the trip, get in touch with me as soon as possible.
C. You would call me whether you changed your mind about the trip.
D. Having changed your mind about the trip, you should get in touch with me soon.
17. If it hadn't been for the goalkeeper, our team would have lost.
A. Our team didn't lose the game thanks to the goalkeeper.
B. Our team lost the match because of the goalkeeper.
C. Without the goalkeeper, our team could have won.
D. If the goalkeeper didn't play well, our team would have lost.
18. I didn't know you were coming, so I didn't wait for you.
A. If I had known you were coming, I would wait for you.
B. I would have waited for you if I knew you were coming.
C. Had I known you were coming, I would have waited for you.
D. If you had known you had been coming, I would have waited for you.
19. My friends rang me at the office because they didn't know that I was away.
A. If my friends rang me at the office, they knew that I was away.
B. My friends didn't ring me because they knew that I was away.
C. Had my friends known that I was away, they wouldn't have rung me at the office.
D. Were my friends not to ring me, they knew that I was away.
20. Please let me know if you wish to keep the books any longer.
A. If you should wish to keep the books any longer, please let me know.
B. Should you wish to keep the books any longer, please let me know.
C. Should you wish keep the books any longer, please let me know.
D. Wish you to keep the books any longer, please let me know.
21. It's my opinion that you should take more exercises.
A. If I were you, you would take more exercises.
B. If I were you, I would take more exercises.
C. Had I been you, I would take more exercises.
D. If I were you, I will take more exercises.
22. Had the advertisement for our product been better, more people would have bought it.
A. Not many people bought our product because it was so bad.
B. Our product was of better quality so that more people would buy it.
C. Fewer people bought our product due to its bad quality.
D. Since our advertisement for our product was so bad, fewer people bought it.
23. If it hadn't been for his carelessness, we would have finished the work.
A. He was careless because he hadn't finished the work.
B. If he was careful, we would finish the work.
C. If he had been more careful, we would have completed the work.
D. Because he wasn't careless, we didn't finish the work.
24. This conference wouldn't have been possible without your organization.
A. Had you not organized this conference, it wouldn't be possible.
B. Your organization made it possible for this conference to take place.
C. But for your organization, this conference would have been possible.
D. If you didn't organize, this conference wouldn't have taken place.
25. She is so busy that she can't come to the party.
A. If she were not so busy, she must come to the party.
B. If she were not so busy, she need to come to the party.
C. If she were not so busy, she could come to the party.

90
D. If she were not so busy, she should come to the party.

CHUYÊN ĐỀ ĐẢO NGỮ - CÂU 44 ĐỀ MH

Question 44: Hoa had just graduated from university. She was offered a job in a prestigious
multinational corporation.
A. Hardly had Hoa graduated from university when she was offered a job in a prestigious
multinational corporation.
B. Only after Hoa was offered a job in a prestigious multinational corporation did she graduate
from university.
C. Not until Hoa was offered a job in a prestigious multinational corporation did she graduate
from university.
D. Had it not been for Hoa’s graduation from university, she would have been offered a job
in a prestigious multinational corporation.

a. NO SOONER + had + S + V3,ed + THAN + S + V2,ed


b. HARDLY / BARELY / SCARCELY + had + S + V3,ed +WHEN + S + V2,ed
(Ngay sau khi….thì….// Vừa mới…..thì….)
c. NOT UNTIL + CLAUSE / TIME + AUXILIARY + S + V ………….
d. ONLY WHEN / AFTER + CLAUSE + AUXILIARY + S + V ……

PRACTICE 1
1. The basketball team knew they lost the match. They soon started to blame each other.
A. Hardly had the basketball team known they lost the match when they started to blame each
other.
B. Not only did the basketball team lose the match but they blamed each other as well.
C. No sooner had the basketball team started to blame each other than they knew they lost the
match.
D. As soon as they blamed each other, the basketball team knew they lost the match.
2. Sue and Brian met. Shortly after that, he announced they were getting married.
A. As soon as Sue and Brian met, they announced they were getting married.
B. Right at the time Brian met Sue, he announced they were getting married.
C. Scarcely had Sue and Brian met when he announced they were getting married.
D. Until Sue and Brian met, they had announced they were getting married.
3. She had only begun to speak. People started interrupting.
A. She hardly had begun to speak when people started interrupting.
B. Hardly she had begun to speak when people started interrupting.
C. Hardly had she begun to speak when people started interrupting.
D. She hadn’t begun to speak when people started interrupting.
4. I had only just put the phone down. The boss rang back.
A. I put the phone down when the boss rang back.
B. Hardly had I put the phone down when the boss rang back.
C. No sooner had I put the phone down when the boss rang back.
D. Scarcely had I put the phone down than the boss rang back.

91
5. The car turned the corner. The wheel came off.
A. No sooner has the car turned the corner than the wheel came off.
B. No sooner the car had turned the corner than the wheel came off.
C. No sooner had the car turned the corner, the wheel came off.
D. No sooner had the car turned the corner than the wheel came off.
6. Maria just passed her driving test. Maria received a car right then.
A. Hardly had Maria received a car when she passed her driving test.
B. It was not until Maria got a car that she quickly passed her driving test.
C. Maria was not able to get a car until she passed her driving test.
D. No sooner had Maria passed her driving test than she was given a car.
7. The date of the conference was chosen. Invitations were sent out.
A. After choosing the date of the conference, invitations were sent out.
B. Before sending out invitations, the date of the conference been chosen.
C. Hardly had the date of the conference been chosen when invitations were sent out.
D. Choose the date of the conference before sending out invitations.
8. Han told us about his investing in the company. He did it on his arrival at the meeting.
A. Only after investing in the company did Han inform us of his arrival at the meeting.
B. Not until Han told us that he would invest in the company did he arrive at the meeting.
C. Hardly had he informed us about his investing in the company when Han arrived at the
meeting.
D. No sooner had Han arrived at the meeting than he told us about his investing in the company.
9. We started out for Seoul. It started to rain right after that.
A. No sooner did we start out for Seoul than it started to rain.
B. No sooner had we started out for Seoul when it started to rain.
C. No sooner had it started to rain than we started out for Seoul.
D. No sooner had we started out for Seoul than it started to rain.
10. He left school. He soon started computer programming.
A. No sooner had he started computer programming than he left school.
B. Hardly had he started computer programming when he left school.
C. No sooner had he left school than he started computer programming.
D. After he left school, he had started computer programming.
11. I had turned on my new iPad Pro. There was a strange noise right after that.
A. As soon as there was a strange noise, I turned on my new iPad Pro.
B. Scarcely had I turned on my new iPad Pro when there was a strange noise.
C. Hardly I had turned on my new iPad Pro, there was a strange noise.
D. I had hardly turned on my new iPad Pro than there was a strange noise.
12. Maria sings well. She writes good songs, too.
A. Maria can either sing well or write good songs.
B. Maria can neither sing well or write good songs.
C. Not only does Maria sing well but she also writes good songs.
D. Well as Maria sings, she writes good songs.
13. The boy got out of his house. It soon started to rain heavily.
A. It had rained heavily before the boy got out of his house.
B. Hardly had it started to rain heavily when the boy got out of his house.
C. No sooner had the boy got out of his house than it started to rain heavily.
D. Not until it started to rain heavily did the boy got out of his house.
14. He died in 1960. He received the bravery award in 1970.

92
A. Only after his death did he receive the bravery award in 1970.
B. Because he died in 1960, he received the bravery award in 1970.
C. He died in 1960 so he received the bravery award in 1970.
D. Before his death, he received the bravery award in 1970.
15. The teacher had only just come in the room. Then the fire alarm rang.
A. While the teacher was coming in the room, she heard the fire alarm rang.
B. As soon as the fire alarm had rung, the teacher came in the room.
C. Before the teacher came in the room, the fire alarm rang.
D. Hardly had the teacher come in the room when the fire alarm rang.
16. Ms. Diem had turned up at the party. She soon called home.
A. Calling home, Ms. Diem said that she had turned up at the party.
B. No sooner had Ms. Diem arrived at the party than she called home.
C. Ms. Diem turned up at the party sooner than he had expected.
D. Ms. Diem arrived at the party and called me to take her home.
17. He got down to writing the letter as soon as he returned from his walk.
A. No sooner had he returned from his walk when he got down to writing the letter.
B. Not until he returned from his walk did he get down to writing the letter.
C. Only after he had returned from his walk did he get down to writing the letter.
D. Hardly had he returned from his walk when he got down to writing the letter.
18. She just had time to put up her umbrella before the rain came down in torrents.
A. No sooner had she put up her umbrella than the rain came down in torrents.
B. The rain came down in torrents as soon as she just has time to put up her umbrella.
C. Only when she had put up her umbrella did the rain come down in torrents.
D. If she hadn’t time to put up her umbrella, the rain wouldn’t have come down in torrents.
PRACTICE 2.
1. He started working in the World Bank. That was right after he had graduated from university.
A. No sooner had he graduated from university than he started working in the World
Bank.
B. Hardly had he started working in the World Bank when he graduated from university.
C. If you had lent me the dictionary last night, I could have translated the article today.
D. No sooner had he started working in the World Bank than he graduated from university.
2. He started computer programming as soon as he left school.
A. No sooner had he started computer programming than he left school.
B. Hardly had he started computer programming when he left school.
C. No sooner had he left school than he started computer programming.
D. After he left school, he had started computer programming.
3. The teacher spoke to Mary. Mary then realized her mistake.
A. Only after the teacher had spoken to Mary did she realize her mistake.
B. Were the teacher to speak to Mary, she would realize her mistake.
C. Had the teacher spoken to Mary, she wouldn't have realized her mistake.
D. Not until the teacher realized Mary's mistake did he speak to her.
4. My eldest sister graduated from university. She soon started working as a freelance journalist.
A. No sooner had my eldest sister started working as a freelance journalist than she graduated
from university.
93
B. Not until my eldest sister started working as a freelance journalist did she graduate from
university.
C. No sooner had my eldest sister graduated from university than she started working as a
freelance journalist.
D. Hardly had my eldest sister started working as a freelance journalist when she graduated from
university.
5. I arrived at work. The assistant knocked at the door.
A. No sooner had I arrived at work than the assistant knocked at the door.
B. I hardly knew the assistant knocked at the door as I just arrived at work.
C. I was arriving at work as the assistant knocked at the door.
D. Hardly had I arrived at work then the assistant knocked at the door.
6. She read all the instructions of the exam. She started to do it.
A. Only by reading all the instructions of the exam did she start to do it.
B. Hardly had she read all the instructions of the exam when she started to do it.
C. Not until she started to do the exam did she read all of its instructions.
D. Only after reading all the instructions of the exam that she started to do it.
7. I only realised how dangerous the situation had been when I got home.
A. Only when I got home did I realise how dangerous the situation had been.
B. When I got home, I was realizing how dangerous the situation had been.
C. Only when did I get home that I realise how dangerous the situation had been.
D. Just after I got home, I only realised how dangerous the situation had been.
8. We had reached the top. Then we realized how far we had come.
A. Not until we had reached the top did we realize how far we had come.
B. Having realized how far we had come, we reached the top.
C. Only after we realized how far we had come had we reached the top.
D. We had reached the top until we realized how far we had come.
9. Ngoc’s personal information was leaked. She was aware of the danger of the Internet.
A. No sooner had Ngoc realised that the Internet was dangerous than her personal information
was leaked.
B. Only after Ngoc’s personal information had been leaked did she realise how dangerous
the Internet was.
C. Not until Ngoc’s personal information had been leaked was she aware of the benefit of the
Internet.
D. Hardly had Ngoc’s personal information been leaked when she realised how beneficial the
Internet was.
10. Emily had just finished saving all the documents. The computer crashed then.
A. Hardly had Emily finished saving all the documents when the computer crashed.
B. Had it not been for the computer crash, Emily could have saved all the documents.
C. No sooner had the computer crashed than Emily finished saving all the documents.
D. The moment Emily started to save all the documents, the computer crashed.
11. They arrived at the market. The announcement then started.

94
A. No sooner had they arrived at the market than the announcement started.
B. Having arrived at the market, the announcement then started.
C. Hardly had they arrived at the market than the announcement started.
D. Only after the announcement started did they arrive at the market.
12. The marketing team knew they couldn't sign the contract. They soon started to blame each
other.
A. No sooner had the marketing team started to blame each other than they knew they couldn't
sign the contract.
B. As soon as they blamed each other, the marketing team knew they couldn't sign the contract.
C. Hardly had the marketing team known they couldn't sign the contract when they
started to blame each other.
D. Not only could the marketing team sign the contract but they also blamed each other.
13. July told her parents about her result in the final exam. She did it on her arrival home.
A. Only after getting the result in the final exam July informed her parents of it on her arrival
home.
B. Not until July told her parents that she would get good results in the final exam did she arrive
at the meeting.
C. Hardly had she informed her parents about her result in the final exam when July arrived at
the meeting.
D. No sooner had July arrived home than she told her parents about her result in the final
exam.
14. Jane got off the bus. She was robbed of the handbag right after that.
A. As soon as Jane was robbed of the handbag, she got off the bus.
B. Hardly had Jane got off the bus when she was robbed of the handbag.
C. Not until Jane got off the bus, she was robbed of the handbag.
D. Only after did Jane get off the bus, she was robbed of the handbag.
15. We arrived at the cinema. Then we realized that our tickets were still at home.
A. Not until we arrived at the cinema that we realized that our tickets were still at home.
B. Hardly had we arrived at the cinema than we realized that our tickets were still at home.
C. No sooner had we realized that our tickets were still at home than we arrived at the cinema.
D. Only after we had arrived at the cinema did we realize that our tickets were still at
home.
16. They were being followed by a group of strangers. They then realized that.
A. Only later they realized that were they being followed by a group of strangers.
B. Only later did they realize that they were being followed by a group of strangers.
C. Not until they realized that were they being followed by a group of strangers.
D. Not until were they being followed by a group of strangers, they realized that.
17. Mike became a father. He felt a strong sense of responsibility towards his parents.
A. Only after Mike had become a father himself did he feel a strong sense of responsibility
towards his parents.
B. Not until he felt a strong sense of responsibility towards his parents did Mike become a father

95
himself.
C. Had Mike become a father himself, he would have felt a strong sense of responsibility
towards his parents.
D. Were Mike to become a father himself, he would feel a strong sense of responsibility towards
his parents.
18. The train arrived at the station. Shortly after that the passengers rushed towards it.
A. Only after the passengers rushed towards the station did the train arrive.
B. Had the train arrived at the station, the passengers could have rushed towards it.
C. Scarcely had the train arrived at the station when the passengers rushed towards it.
D. Not until the passengers had rushed towards the station did the train arrive.
19. Olga handed in her exam paper. She then realised that she had missed one question.
A. Had Olga realised that she had missed one question, she wouldn’t hand in her exam paper.
B. Having realised that she had missed one question, Olga handed in her exam paper.
C. Only after Olga realised that she had missed one question did she hand in her exam paper.
D. Not until Olga had handed in her exam paper did she realise that she had missed one
question.
20. James started working. He then realized that his decision had not been a good one.
A. Just before James took up his new post, he realized that he was not suited for it.
B. Had James not begun his new job, he would have gone looking for a better one.
C. Since James did not like his new job, he began looking for a better one.
D. No sooner had James begun his new job than he knew his decision was wrong.
21. We had no sooner got to know our neighbors than they moved away.
A. Once we had got used to our new neighbors, they moved somewhere else.
B. Soon after we got to know our new neighbors, we stopped having contact with them.
C. If our new neighbors had stayed longer, we would have got to know them better.
D. Hardly had we become acquainted with our new neighbors when they went somewhere
else to live.
22. He participated in the environmental campaign. He realised how much damage humans had
caused to the environment only then.
A. Only when he realised how much damage humans had caused to the environment did he take
part in the environmental campaign.
B. Only after he had been involved in the environmental campaign did he realise how
much damage humans had caused to the environment.
C. Not until he had withdrawn from the environmental campaign did he realise how much
damage humans had caused to the environment.
D. No sooner had he joined the environmental campaign than he failed to understand how much
damage humans had caused to the environment.
24. Phuc Lam focused on studying a couple of months. Right after that, he won a medal in the
International Olympic Examination.
A. Not until Phuc Lam had won a medal in the International Olympic Examination did he focus
on studying a couple of months.

96
B. Only after had Phuc Lam focused on studying a couple of months did he win a medal in the
International Olympic Examination.
C. But for a medal in the International Olympic Examination, Phuc Lam wouldn’t focus on
studying a couple of months.
D. Hardly had Phuc Lam focused on studying a couple of months when he won a medal in the
International Olympic Examination.
25. They got lost in the jungle for almost two days. Also, they had nothing to eat.
A. Having got lost in the jungle for nearly two days, they eventually had something to eat.
B. Neither did they get lost in the jungle for almost two days nor did they have anything to eat.
C. Not only did they get lost in the jungle for almost two days but they also had nothing to eat.
D. They had got lost in the jungle for nearly two days until they had something to eat.

LỖI PHÉP QUY CHIẾU – (Q.45)

Question 45: All office workers are required to wear her name tags in the workplace.
A B C => their D

Đại từ chủ Đại từ phản


Đại từ tân ngữ Tính từ sở hữu Đại từ sở hữu
ngữ thân
myself (chính mine (là của
I (tôi) me (là tôi) my (của tôi)
tôi) tôi/thuộc về tôi)
yourself, yours(là của
yourselves your (của bạn/các bạn/
You (bạn) you (là bạn)
(chính bạn/các bạn/các bạn) thuộc về các
bạn) bạn)
himself, herself,
He, she, it him, her, it (là his, her, its (của his, hers (là của
itself (chính
(anh/cô ta, nó) anh/ cô ấy, nó) anh/cô ấy, nó) anh/cô ấy)
anh/cô ấy, nó)
ours (là của
ourselves (chính our (của chúng
We (chúng tôi) us (là chúng tôi) chúng tôi/thuộc
chúng tôi) tôi/chúng ta)
về chúng tôi)
themselves theirs (là của
They (Họ/ them (họ/ chúng their (của họ/
(chính họ/ họ/thuộc về
chúng nó) nó) của chúng)
chúng nó) họ/..)
PRACTICE
Question 1: Many living organisms depend largely on the environment for the satisfaction
A B C
of its needs.
D
Question 2: All the judges paid the dancer compliments on their excellent performance
A B C

97
in the competition.
D
Question 3: Unlike the old one, this new car can perform their functions in half the time
A B C D
Question 4: All the candidates for the scholarship will be equally treated regardless of
A B C
her age, sex, or nationality.
D
Question 5: A beaver uses the strong front teeth to cut down trees and peel off its bark.
A B C D
Question 6: Research has proved that a baby can distinguish their mother's voice from the womb.
A B C D
Question 7: The Joshua tree is marked by their sword shaped leaves and greenish white flowers.
A B C D
Question 8: An octopus has three hearts to pump blood throughout their body.
A B C D
Question 9: Kiwi birds mainly eat insects, worms, and snails and search for food by
A B
probing the ground with its long bills.
C D
Question 10: The library at the university is new and has taken her name from the wife
A B C
of the first president of the university.
D
Question 11: Deciduous trees are ones that lose its leaves in the autumn and grow new ones in the spring.
A B C D
Question 12: Some of us have to study their lessons more carefully if we expect
A B C
to pass this examination.
D
Question 13: Every scientist knows that gravity is the force that maintains the earth and
A B
the planets in its orbits around the Sun.
C D
Question 14: The Gray Wolf, a species reintroduced into their native habitat in
A B
Yellowstone National Park, has begun to breed naturally there.
C D
Question 15: The international Red Cross, which has helped so many nations, won
A B
the Nobel Peace Prize three times for their efforts to reduce human suffering.
C D
Question 16: Genetic engineering is helping researchers unravel the mysteries of
A
previously incurable diseases so that they can get to its root causes and find cures.
B C D
Question 17: Each of the students in the accounting class has to type their own research

98
A B C
paper this (D) semester.
Question 18: Animals like frogs have waterproof skin that prevents it from drying out
A B C
quickly in air, sun, or wind.
D
Question 19: Children learn primarily by directly experiencing the world around it.
A B C D
Question 20: Since poaching is becoming more serious, the government has imposed
A B
stricter laws to prevent them.
C D
Question 21: Those of us who have a family history of heart disease should make yearly
A B C
appointment with their doctors
D
Question 22: The sea horse is unique among fish because the female deposits their eggs in
A B C
a pouch that the male carries until the small sea horses are hatched.
D
Question 23: Sloths spend most of its time hanging upside down from trees and feeding on
A B C
leaves and fruit.
D
Question 24: Before the report is finalized, the information in their notes and our must be
A B C D
proofed.

Question 25: The weather in the mountains this weekend will be extremely cold, so please
A B C
take yours heavy jackets.
D
Question 26: Helicopters are being used more and more in emergency situations because of
A B
its ability to reach out-of –the-way places.
C D
Question 27: Many different kinds of aspirin are on the market, but theirs effectiveness
A B C
seems to be equal.
D
Question 28: When children experience too much frustration, its behavior ceases to be integrated.
A B C D
Question 29: Almost half of the Pilgrims did not survive theirs first winter in the New World.
A B C D
Question 30: The latest medical report indicated that the patient’s temperature was near
A B
normal and their lungs were partially cleared.

99
C D
Question 31: Our company offered them a job but they turned it down claiming that their is better.
A B C D
Question 32: He’s got into trouble at work on several occasions because of their
A B C
unconventional way of dealing with clients.
D
Question 33: Life is getting more and more expensive, so women’s salaries are becoming
A B
important to her household budgets.
C D
Question 34: While the children were playing on the beach, Peter trod on a broken bottle
A B C
and cut their foot rather badly.
D
Question 35: Passengers were left stranded for three hours when its train broke down.
A B C D

Q. 46. CONFUSING WORDS – TỪ HAY NHẦM LẪN


(SỬA LỖI)

appreciable (a)

100
nâng cao

Question 46: The local government and the advisable committee agreed that the construction of
A
the new residential area should play a substantial role in the long-term goals.
B C D
* advisable (adj) thích hợp => advisory: tư vấn, cố vấn
=>Chính quyền địa phương và ủy ban tư vấn đã đồng ý rằng việc xây dựng khu dân cư mới sẽ đóng
một vai trò quan trọng trong các mục tiêu dài hạn.

PRACTICE:
1. The building was formally used as a bank, but it has been turned into a church recently.
A B C D
2. A considerate amount of time and effort has gone into this exhibition.
A B C D
3. Emotional problems, such as stress, anxiety, or depression, can make a person more forgettable.
A B C D
4. You should be more respectable of other people’s points of view. Don’t embarrass
A B C D
someone even if they are wrong.
5. The novel is regarded as one of the classical works. I really love reading it in my free time.
A B C D
6. I think the sensitive thing to do is call and ask for directions.
A B C D
7. We must develop more rapid, responsible systems for dealing with online messages.
A B C D

101
8. With rents so high, it wasn’t economic to continue to live in the city.
A B C D
9. You’ll need to be a little more imaginable if you want to hold their attention.
A B C D
10. Most of these women are very poorly paid and work in terrific conditions.
A B C D
11. An area with a favourite climate will inevitably be richer than one without.
A B C D
12. Our team won the trophy for the second successful season.
A B C D
13. He was arrested at the airport with a kilo of heroine secreted in his clothing.
A B C D
14. We need someone really effective who can organize the office and make it smoothly.
A B C D
15. The organization works on the principal that all members have the same rights.
A B C D
16. We will offer you a comprehensible training in all aspects of the business if you take this
course. A B C D
17. The plans for the new development have risen angry protests from local residents.
A B C D
18. She is always polite and considerable towards her employees.
A B C D
19. His continuous demands for sympathy became quite a strain on his friends.
A B C D
20. He’s the author of several hugely successive children’s books.
A B C D
21. The equator is an imaginative line around the middle of the earth.
A B C D
22. Certain chemicals have been banned because of their damaging affect on the
A B C
environment. However, many farmers are still using them on their farm.
D
23. TikTok, a very entertained application, has become popular with the young recently.
A B C D
24. It is essential to provide high school students with adequate career orientation, so
A B
they can make more informative decisions about their future major.
C D
25. There is no doubt that the cost of living in many megacities around the world is raising.
A B C D
26. Some people believe animal behaviour could offer a viable alternative means of
earthquake detective. A B C
D
27. Producing variable clones of many species is a lengthy and difficult problem and,
A B
despite questionable claims of success, has yet to be proven in humans.
C D
102
28. For those who suffered from irreversible damage caused by the unexpected fire, the
A B
relief agencies tried to provide food and other basic essence.
C D
29. The woman relentlessly raised concerns about neglectful parental skills, heavy
A B C
drinking and severe domestic violence.
D
30. The results of an exhausting study into masculinity were published this week and
A B C
they make for illuminating reading.
D
31. The unprecedented pandemic has caused massive disruptions to the country and the
A B
government is waiting until economic conditions are more favourite.
C D
32. To everyone’s surprising, it wasn’t in Da Nang City that he made his fortune,
A B C D
although that’s where he was born.
33. Until the invention of the telephone, skyscrapers were not considered very practicable.
A B C D
34. It is advisable to wait for the domestic economic conditions to become more
A B
favorite before making any massive investment in stocks.
C D
35. She is a confidential and practiced speaker who always impresses her audience.
A B C D
36. Food prices have raised so rapidly in the past few months that many families have
A B C D
to change their eating habit.
37. William James, the general manager, started with some complementary remarks
A B C
about the organisers of the conference.
D
38. I felt annoyed by his continuous interruptions at the meeting this morning.
A B C D
39.It’s economic to buy good shoes. They cost more, but they last much longer than cheaper ones.
A B C D
40. He was sensitive enough to see that Jake was the best candidate for the job.
A B C D
41. I find it impossible to sleep because of the continual noise from the party in the flat above.
A B C D
42. Human prehistory is divided into three successful periods: The Stone Age, the
A B C
Bronze Age and the Iron Age.
103
D
43. We spent about three weeks in Boston, after leaving New York, and I didn’t
A B
need to tell you we had a most delighted time.
C D
44. Teacher says it was a day-dream, and she thinks you would be delightful to hear it.
A B C D
45. Because the electric device’s signals could interfere with traffic control, Sheila
A B
was asked to place her cellphone on airplane mode.
C D
46. I am not fond of reading fiction books as they are imaginable ones which are not real.
A B C D
47. The world is becoming more industrial and the number of animal species that have
A B
become extinct has increased.
C D
48. Dr Wee is a human physician. He treats the poor for free.
A B C D
49. Her father used to be a distinguishable professor at the university. Many students
A B C
worshipped him.
D
50. The whole matter is farther complicated by the fact that Amanda and Jo refuse
A B C
to speak to each other.
D
51. The building was formally used as a bank, but it has been turned into a church recently.
A B C D
52. He's such a bored guy because he only ever talks about himself.
A B C D
53. Germany, in companion with France, has now lifted the ban on the export of live
A B C
animals to Europe.
D
54. Many successful film directions are former actors who desire to expand their
A B C D
experience in the film industry.
55. Hot soup is very comfortable on a cold winter's day.
A B C D
56. It is said that these good life skills will make young people become more
A B C
confidential. (D)
57. They have carried exhausting research into the effects of smartphone on
A B
104
schoolchildren’s behaviour and their academic performance
C D
58. Food prices have raised so rapidly in the past few months that some families have
A B C
been forced to alter their eating habits
D
59. Our new products are being sold very well due to favourite condition in the market.
A B C D
60. The sign says that we should read the constructions carefully before proceeding.
A B C D
61. Computers treat time as a series of discreet moments rather than a continuous flow.
A B C D
62. People should be encouraged to buy smaller, more economic cars with fewer toxic
emissions. A B C
D
63. I am deeply impressive by the beauty of Ancient capital of Hue
A B C D
64. This is an exhausting list of grammar rules you will need to know for the exam.
A B C D

LỖI VỀ THÌ (Q.47)

Question 47: Many doctors in our hospital volunteer to help people in remote areas last year.
A B C D

Mark the letter A, B, C, or D on your answer sheet to indicate the underlined part that needs
correction in each of the following questions.
1. He is arrested at the airport last Sunday with a kilo of heroin secreted in his clothes.
A B C D
2. The national soccer team make every effort to win the match yesterday.
A B C D
3. Animals like frogs had waterproof skin that prevents them from drying out quickly in air, sun
or wind. A B C D
4. I think artificial intelligence would replace jobs in the future.
A B C D
5. Barack Obama, along with his wife and two daughters enter the White House in 2008.
A B C D
6. People supposed that the adventure of ChatGPT will have negative impacts on how
A B C
college students write essays.
D
7. The company developed more rapidly and reliant systems for handling customers’
105
A B C
complaints for 2 years.
D
8. My brother leaves his job last week because he did not have any chance to travel.
A B C D
9. Commercial airliners did not fly in the vicinity of volcanic eruptions because even a
A B C
small amount of volcanic ash can damage their engines.
D
10. Air pollution became very serious in recent years due to human activity.
A B C D
11. The president do not sound unduly concerned that the new policy would be
A B
disadvantaged to the future development of the company.
C D
12. If the essay is written in a simpler way, it would be rated as good.
A B C D
13. The poor old man is unable to explain what was going on to his family.
A B C D
14. Though the exact death toll was not known, it is considered the dead tropical cyclone
on record. (D) A B C
15. My friends get lost on the campus of their university a few days ago.
A B C D
16. Yesterday, for the first time in his life, Mike refuses to follow his parents' advice
A B C D
17. ASEAN aimed at promoting economic growth, regional peace as well as providing
A B C
opportunities for its members.
D
18. Last year, Tim works as a shipper and he earned twice as much as his brother.
A B C D
19. Now some children still faced discrimination at school because of its sexual
A B C D
orientation.
20. I have worked on an English assignment yesterday morning.
A B C D
21. Last week Mark tells me that he got very bored with his job and was looking for a new one.
A B C D
22. She has disappeared three days ago, and they are still looking for her now.
A B C D
23. My mother is watching a game show on TV at 9 p.m yesterday.
106
A B C D
24. Yesterday, I leave for work late because I had forgot to set my alarm.
A B C D
25. Do you remember the man who you meet at the party last night?
A B C D
26. There were so many books on the shelves that I don’t know which one to choose.
A B C D
27. Last year, the story of The Three Little Pigs is adapted from different sources to print in books
for children. (D) A B C
28. The man, a poor man, is invited to the Clambake last night.
A B C D
29. Since he started primary school, he helped his mother prepare meals,
A B C
do the washing up and take care of his younger sister.
D
30. He didn’t meet her even one time since they said good bye together .
A B C D
31. Many countries were now concerned by the low level of economic activity.
A B C D
32. In the past, people don’t tend to pay much attention to leisure time activities, which is very
A B C D
different from now.
33. Last night, Susan doesn’t come to the party with her boyfriend because of bad weather.
A B C D
34. Next week, when there will be an English club held here, I will give you more information
A B C about (D) it.
35. My brother usually asked me for help when he has difficulty with his homework.
A B C D
36. Peter’s sister gets married to a rich businessman two years ago.
A B C D
37. I’m becoming increasingly forgetful. Last week I lock myself out of the house twice.
A B C D
38. His uncle only gets home from France yesterday.
A B C D
39. I am studying hard for a month and now I feel exhausted.
A B C D
40. Not Scotland again! It rains every day last time. I want to go somewhere sunny this summer.
A B C D
41. Because it did not have a blood supply, the cornea takes its oxygen directly from the air.
A B C D
42. This is an exhaustive list of grammar rules you would need to know for the exam.

107
A B C D
43. In South Korea, the national government built English immersion schools all over
A B C D
the country so far.
44. On April 15, 2012 the Health Ministry of Viet Nam issue vaccine passports that the
A B
general public can use to travel in line with their schedule.
C D
45. The current restructuring of the advisory service provided a timely opportunity to
A B C D
address some of these issues.
46. Last Wednesday, they cancel all the sporting events because of the heavy rain.
A B C D
47. Passengers are left stranded for three hours when their train broke down.
A B C D
48. She has disappeared three days ago, and they are still looking for her now.
A B C D
49. Yesterday afternoon, Vanessa had submitted the research paper early for the first time.
A B C D
50. Scarecrows help farmers save their crops from hungry birds in ancient times.
A B C D

CHỌN CÂU ĐỒNG NGHĨA - PASSIVE MODELS/ REPORTED/ TENSES - (Q 48)


Question 48: Visitors are not permitted to enter this area.
A. Visitors won’t enter this area. B. Visitors needn’t enter this area.
C. Visitors wouldn’t enter this area. D. Visitors mustn’t enter this area.

A. MODEL VERBS

108
Mark the letter A, B, c, or D on your answer sheet to indicate the sentence that is closest in
meaning to each of the following questions.
1. It's necessary for you to drink enough water every day.
A. You should drink enough water every day. B. You may drink enough water every day.
C. You needn't drink enough water every day. D. You mustn't drink enough water every day.
2. It is unnecessary for you to finish the report until tomorrow afternoon
A. You needn’t finish the report until tomorrow afternoon.
B. You have to finish the report until tomorrow afternoon.
C. You may finish the report after tomorrow afternoon.
D. You should finish the report until tomorrow afternoon
3. Cheating is not allowed in the GCSE.
A. You may cheat in the GCSE. B. You mustn’t cheat in the GC.SE
C. You don’t have to cheat in the GCSE D. You should cheat in the GCSE
4. You are able to go out with your friend this evening
A. You mustn’t go out with your friend this evening.
B. You should go out with your friend this evening.
C. You needn’t go out with your friend this evening.
D. You can go out with your friend this evening.
5. It isn’t necessary for us to get a visa for Singapore.
A. We mustn’t get a visa for Singapore. B. We needn’t get a visa for Singapore.
C. We mayn’t get a visa for Singapore. D. We shouldn’t get a visa for Singapore.
6. I’m sure that they had practiced hard for the games as they won a lot of medals.
A. They couldn’t have practiced hard for the games as they won a lot of medals
B. They must have practiced hard for the games as they won a lot of medals
C. They shouldn’t have practiced hard for the games as they won a lot of medals
D. They might have practiced hard for the games as they won a lot of medals.
7. Children are not allowed to play football in the streets.
A. Children may play football in the streets. B. Children mustn’t play football in the streets.
109
C. Children should play football in the streets. D. Children needn’t play football in the streets.
8. It’s very likely that the company will accept his application.
A. The company needs accept his application.
B. The company might accept his application.
C. The company must accept his application.
D. The company should accept his application.
9. She will probably buy this house next week.
A. She doesn't have to buy this house next week.B. She might buy this house next week.
C. She should buy this house next week. D. She must buy this house next week.
10. In most developed countries, it is not necessary for people to boil water before they drink it.
A. In most developed countries, people ought to boil water before they drink it.
B. In most developed countries, people must boil water before they drink it.
C. In most developed countries, people needn’t boil water before they drink it.
D. In most developed countries, people mustn’t boil water before they drink it.
11. It is necessary for you to finish this work today.
A. You can’t finish this work today. B. You mustn’t finish this work today.
C. You may finish this work today. D. You need finish this work today.
12. It isn’t necessary for Tony to buy that house.
A. Tony shouldn’t buy that house. B. Tony needn’t buy that house.
C. Tony mustn’t buy that house. D. Tony can’t buy that house.
13. I am not very sure that Max will accept the job offer.
A. Max should accept the job offer. B. Max may not accept the job offer.
C. Max can’t accept the job offer. D. Max mustn’t accept the job offer.
14. I don’t think Max broke your vase because he wasn’t here then.
A. Max wouldn’t have broken your vase because he wasn’t here then.
B. Max was likely to break your vase because he wasn’t here then.
C. Max can’t have broken your vase because he wasn’t here then.
D. Max wasn’t able to break your vase because he wasn’t here then.
15. It will be a mistake if you buy that old house.
A. You couldn’t buy that old house. B. You may not buy that old house.
C. You needn’t buy that old house. D. You shouldn’t buy that old house.
16. There was no need for you to have left the house in such weather.
A. You shouldn’t leave the house in such weather.
B. You didn’t have to leave the house in such weather.
C. You didn’t need to leave the house in such weather.
D. You needn’t have left the house in such weather.
17. There’s now no possibility that he will pass the driving test.
A. He might pass the driving test easily. B. He needn’t pass the driving test.
C. He can pass the driving test easily. D. He may not pass the driving test.
18. It is unnecessary for you to finish the report until tomorrow afternoon
A. You needn’t finish the report until tomorrow afternoon.
110
B. You have to finish the report until tomorrow afternoon.
C. You may finish the report after tomorrow afternoon.
D. You should finish the report until tomorrow afternoon
19. My car keys are possibly in the kitchen.
A. My car keys should be put in the kitchen.
B. My car keys cannot be in the kitchen.
C. I do not know whether my car keys are in the kitchen.
D. My car keys might be in the kitchen.
20. It is necessary for you to get a visa for Singapore.
A. We needn’t get a visa for Singapore. B. We mustn’t get a visa for Singapore.
C. We may get a visa for Singapore. D. You should get a visa for Singapore.
21. It’s impossible that John is at school right now.
A. John can’t be at school right now. B. John might be at school right now.
C. John need be at school right now. D. John should be at school right now.
22. It isn't necessary to submit my assignment today.
A. I must submit my assignment today. B. I needn't submit my assignment today.
C. I could submit my assignment today. D. I shouldn’t submit my assignment today.
23. I am required to prepare the meals for my family every day.
A. I must prepare the meals for my family every day.
B. I will prepare the meals for my family every day.
C. I can prepare the meals for my family every day.
D. I don’t have to prepare the meals for my family every day.
24. My children are possibly in the living room.
A. My children will be playing in the living room.
B. My children cannot be in the kitchen.
C. I do not know whether my children are in the living room.
D. My children might be in the living room.
25. It's possible that Newton has got lost.
A. There's possibility that Newton will get lost. B. Newton may well have got lost
C. Newton was highly likely to get lost. D. Newton must have gone lost.
26. There is no need for you to redecorate your bedroom.
A. You needn’t redecorate your bedroom B. You may have to redecorate your bedroom.
C. You should need to redecorate your bedroom.D. You mustn’t redecorate your bedroom.
27. It’s possible that Joanna didn’t receive my message.
A. Joanna shouldn’t have received my message. B. Joanna needn’t have received my message.
C. Joanna mightn’t have received my message. D. Joanna can’t have received my message.
28. Every student is required to write an essay on the topic.
A. Every student might write an essay on the topic.
B. Every student must write an essay on the topic.
C. They require every student can write an essay on the topic.
D. Every student should write an essay on the topic.
111
29. Walking on the grass in the park is not permitted.
A. You can walk on the grass in the park if you want to.
B. People like walking on the grass in the park.
C. We must not walk on the grass in the park.
D. We do not have to walk on the grass in the park.
30. You are allowed to take some photos at the park.
A. You can take some photos at the park. B. You mustn’t take some photos at the park.
C. You may have taken some photos at the park. D. You need to take some photos at the park.
31. It is necessary for you to eat more vegetables.
A. You should eat more vegetables. B. You mustn’t eat more vegetables.
C. You needn’t eat more vegetables. D. You may eat more vegetables.
32. I’m not sure that she will arrive at the party on time.
A. She mustn’t arrive at the party on time. B. She may not arrive at the party on time.
C. She should arrive at the party on time. D. She doesn’t have to arrive at the party on time.
33. It’s possible that John misses my party.
A. John should miss my party B. John needn’t miss my party.
C. John might miss my party D. John can’t miss my party.
34. There’s now no possibility that he will pass the driving test.
A. He might pass the driving test easily. B. He needn’t pass the driving test.
C. He can pass the driving test easily. D. He may not pass the driving test.
35. Perhaps Susan knew the address.
A. Susan must have known the dress. B. Susan should know the dress.
C. Susan may have known the dress. D. Susan needn’t know the dress.
36. It’s not necessary for you to do homework today.
A. You shouldn’t do homework today. B. You needn’t do homework today.
C. You mustn’t do homework today. D. You can’t do homework today.
37. It is essential for you to do morning exercise regularly.
A. You should do morning exercise regularly. B. You may do morning exercise regularly.
C. You needn’t do morning exercise regularly. D. You mustn’t do morning exercise regularly.
38. It is against the school rules to cheat in the test.
A. You don't have to cheat in the test. B. You must cheat in the test.
C. You must not cheat in the test. D. You have to cheat in the test.
39. My grandfather was able to work on the field all day without eating.
A. My grandfather would work on the field all day without eating.
B. My grandfather can work on the field all day without eating.
C. My grandfather may work on the field all day without eating.
D. My grandfather could work on the field all day without eating.
40. You are not allowed to take the exam paper out of the classroom.
A. You mustn’t take the exam paper out of the classroom.
B. You may take the exam paper out of the classroom.
C. You needn’t take the exam paper out of the classroom.

112
D. You should take the exam paper out of the classroom.
41. It’s very likely that the company will accept his application.
A. The company needs accept his application. B. The company might accept his application.
C. The company must accept his application. D. The company should accept his application.
42. Sally paid for her travel in advance, but it wasn’t necessary.
A. Sally needn't have paid for her travel in advance.
B. Sally might not have paid for her travel in advance.
C. Sally may not have paid for her travel in advance.
D. Sally couldn’t have paid for her travel in advance.
43. It’s not obligatory for me to submit my assignment today.
A. I must submit my assignment today. B. I needn't submit my assignment today.
C. I could submit my assignment today. D. I shouldn’t submit my assignment today.
44. Smoking is not allowed in the museum.
A. You mustn’t smoke in the museum. B. You can smoke in the museum
C. You don’t have to smoke in the museum D. You may smoke in the museum

C. CHUYỂN ĐỔI THÌ – HTHT – QKĐ (Q.49)


Question 49: The last time Tim played basketball with his brother was five months ago.
A. Tim hasn’t played basketball with his brother for five months.
B. Tim didn't play basketball with his brother for five months.
C. Tim has played basketball with his brother for five months.
D. Tim started playing basketball with his brother five months ago.

a. S + last + V2/ed + O + khoảng thời gian + ago.


=> S + has/have + not + V3/ed + ……. + for + khoảng thời gian.
b. It is + (Time) + since + S + (last) + V2/ed + …….
=> The last time S + V2/ed ……… was + (Time) + ago

113
1. She last visited her home country ten years ago.
A. She hasn't visited her home country for ten years.
B. She didn’t visit her home country ten years ago.
C. She has visited her home country for ten years.
D. She was in her home country for ten years.
2. The last time she came back to her hometown was 4 years ago.
A. It's 4 years since she last lived in her hometown.
B. She didn't come back to her hometown 4 years ago.
C. She started coming back to her hometown 4 years ago.
D. She hasn’t come back to her hometown for 4 years.
3. He started working as a bank clerk 3 months ago.
A. He last worked as a bank clerk 3 months ago.
B. He didn't work as a bank clerk 3 months ago.
C. He has been working as a bank clerk for 3 months.
D. The last time she worked as a bank clerk was 3 months ago.
4. It is a long time since we last met each other.
A. We haven't met each other for a long time.
B. The last time we met each other is a long time ago.
C. We last met each other for a long time.
D. We started meeting each other a long time ago.
5. I last had my hair cut in November.
A. The last time I had my hair cut was since November.
B. I haven't had my hair cut since November.
C. I didn't have my hair cut in November.
D. I started having my hair cut in November.
6. This is the first time I have had such a delicious meal.
A. I haven't had such a delicious meal before.
B. This is the most delicious meal I have ever had.
C. I didn't have such a delicious meal. D. I didn’t have such a delicious meal before.
7. She has been working as a teacher for 5 years.
114
A. The last time she worked as a teacher was 5 years ago.
B. It is 5 years since she last worked as a teacher.
C. She began working as a teacher 5 years ago.
D. She hasn’t been working as a teacher for 5 years.
8. I haven't met my grandparents for five years.
A. I often met my grandparents five years ago.
B. I last met my grandparents five years ago.
C. I have met my grandparents for five years.
D. I didn't meet my grandparents five years ago.
9. He last visited London three years ago.
A. He has been in London for three years. B. He hasn't visited London for three years.
C. He didn't visit London three years ago. D. He was in London for three years.
10. I haven't visited my hometown for a few years.
A. I have been in my hometown for a few years.
B. I last visited my hometown a few years ago.
C. I didn't visit my hometown a few years ago.
D. I was in my hometown for a few years.
11. I haven't heard from Susan for several months.
A. I last heard from Susan several months ago.
B. Susan didn't hear from me several months ago.
C. Susan heard from me several months ago.
D. I didn't hear from Susan several months ago.
12. It is over twenty years since I last got in touch with them.
A. I can’t help keeping getting in touch with them for over 20 years.
B. I haven't gotten in touch with them for over 20 years.
C. I used to get in touch with them for over 20 years.
D. I have been getting in touch with them for over 20 years.
13. He last had his eyes tested ten months ago.
A. He had tested his eyes ten months before.
B. He didn’t have any test on his eyes ten months before.
C. He had not tested his eyes for ten months then.
D. He hasn’t had his eyes tested for ten months.
14. The last time I went to the museum was a year ago.
A. I have not been to the museum for a year. B. A year ago, I often went to the museum.
C. My going to the museum lasted a year. D. At last I went to the museum after a year.
15. The last time I saw Peter was when I ran into him at the station on my way to Glasgow.
A. I haven't seen Peter since a chance meeting with him at the station when I was setting off
for Glasgow
B. The last time I went to Glasgow, I happened to meet Peter at the station.
C. When I last saw Peter at the station when I was on my way to Glasgow, I ran after him.
D. I finally saw Peter at the station when I was on my way to Glasgow.
16. We started working here three years ago.
A. We have been working here for three years. B. We worked here for three years.
C. We will work here for three years. D. We have no longer worked here for three years.
17. It started to rain at 2 o’clock and it is still raining.
A. It has been raining at 2 o’clock. B. It has been raining in 2 o’clock.
C. It has been raining for 2 o’clock. D. It has been raining since 2 o'clock

115
18. The last time I saw Rose was three years ago.
A. I haven't seen Rose since three years. B. I haven’t seen Rose three years ago.
C. I didn’t see Rose for three years. D. I haven't seen Rose for three years.
19. Eight years ago, we started writing to each other.
A. We have rarely written to each other for eight years.
B. Eight years is a long time for us to write to each other.
C. We have been writing to each other for eight years.
D. We wrote to each other eight year ago.
20. I last heard this song 10 years ago.
A. It has been 10 years when I have heard this song B. It was 10 years since I last heard this song.
C. It is 10 years since I last heard this song. D. It is 10 years when I have heard this song.

B. REPORTED SPEECH
Question 50: “My sister has just finished her English course,” said Tony.
A. Tony said that my sister has just finished her English course.
B. Tony said that his sister had just finished her English course.
C. Tony said that my sister had just finished her English course.
D. Tony said that his sister just finished her English course.

PRACTICE 1
1. He said, “My wife has just bought a diamond ring.”
A. He said that his wife had just bought a diamond ring.
B. He said that my wife had just bought a diamond ring.
C. He said that his wife has just bought a diamond ring.
D. he said that his wife just bought a diamond ring.
2. “I will come with you as soon as I am ready”, she said to Philip.
A. She said to Philip he will come to see you as soon as he I am ready.
B. She told Philip she will come to see her as soon as she was ready.
C. She told Philip she would come to see you as soon as she was ready.
D. She told Philip she would come to see him as soon as she was ready.
3. “I wrote to him yesterday”
A. She said to me I wrote to him the day before.
B. She told me she wrote to him yesterday.
C. She told me she had written to him yesterday.
D. She told me she had written to him the day before.
4. “He is talking to your sister”, She said to me.
A. She told me he was talking to your sister. B. She told me she was talking to my sister.
C. She told me he was talking to my sister. D. She told me he was talking to her sister.
5. Miss White said to him, “ Why are you so late? Did your car have a flat fire?”
A. Miss White told him why was he so late and did your car have a flat fire
B. Miss White asked him why was he so late and did your car have a flat fire.
C. Miss White asked him why was he so late and if your car have a flat fire.
D. Miss White asked him why he was so late and whether his car had a flat fire.
6. He said "If I had enough money, I could buy that dictionary."
A. He said if he had enough money, he could buy that dictionary
B. He said if he had had enough money, he could buy that dictionary.
C. He said if he had enough money, he could have bought that dictionary.
116
D. He said if he had had enough money, he could have bought that dictionary.
7. “I’ll tell you about this tomorrow, Mary.” said Tom.
A. Tom said to Mary that he will tell her about that the next day.
B. Tom told Mary that I would tell you about that the next day.
C. Tom told Mary that he would tell her about that the next day.
D. Tom told Mary that she would tell him about that the next day.
8. “I have something to tell you” Kerry said to Cheryl.
A. Kerry told Cheryl I had something to tell her.
B. Kerry told Chery he had something to tell her.
C. Kerry told Cheryl she had had something to tell him.
D. Kerry told Cheryl he had had something to tell her.
9. “It is the time to check what you have done”, the father said to the boys.
A. The father said to the boys it was time to check what they had done.
B. The father told the boys it was time to check what they had done.
C. The father told the boys it was time to check what they have done.
D. The father told the boys it is time to check what they had done.
10. “You will like my sister when you meet her”
A. He told me you will like her sister when you meet her.
B. He told me I will like his sister when I met her.
C. He told me I would like his sister when I met her
D. He told me I would be liked his sister when I met her.
11. “I didn’t meet Susan last week”
A. He said he didn’t meet Susan the week before.B. He said he hasn’t met Susan last week.
C. He said he hadn’t met Susan last week. D. He said he hadn’t met Susan the week before.
12. “I didn’t break your watch”
A. The boy told the girl he hadn’t broken her watch.
B. The boy asked the girl he hadn’t broken her watch.
C. The boy told the girl he didn’t break her watch.
D. The boy told the girl he hadn’t broken your watch.
13. “I found a lot of mistakes in your plan”. Catherine said to Kevin.
A. Catherine told Kevin she found a lot of mistakes in his plan.
B. Catherine told Kevin she has found a lot of mistakes in his plan.
C. Catherine told Kevin she had found a lot of mistakes in his plan.
D. Catherine told Kevin she had found a lot of mistakes in her plan.
14. “I have just seen your mother this morning”.
A. Laura told Lewis I have just seen your mother this morning.
B. Laura told Lewis she had just seen his mother that morning.
C. Laura told Lewis she has just seen his mother that morning.
D. Laura told Lewis he had just seen her mother that morning.
15. “We are ready to come with our friends”
A. They told us they are ready to come with their friends.
B. They told us they were ready to come with our friends.
C. They told us we were ready to come with our friends.
D. They told us they were ready to come with their friends.

PRACTICE 2

117
1. '' Where did you go last night''? she said to her boyfriend.
A. She asked her boyfriend where did he go last night.
B. She asked her boyfriend where he went the night before.
C. She asked her boyfriend where had he gone the night before.
D. She asked her boyfriend where he had gone the night before.
2. '' Remember to write to your aunt''. I said to Miss Linh.
A. I said to Miss Linh remember to write to her aunt.
B. I said to Miss Linh to remember to write to her aunt.
C. I told Miss Linh remember to write to her aunt.
D. I reminded Miss Linh to remember to write to her aunt.
3. '' How long have you lived in Ha Noi''? said my friend.
A. My friend asked me how long have I lived in HaNoi.
B. My friend asked me how long had I lived in HaNoi.
C. My friend asked me how long I had lived in HaNoi.
D. My friend asked me how long I have lived in HaNoi.
4. ''Close the books, please'' said our teacher.
A. Close your book said by our teacher. B. Our teacher asked us close our book.
C. Our teacher said us close our book. D. Our teacher asked us to close our book.
5. “I didn’t break your watch” the boy said.
A. The boy told the girl that he hadn’t broken her watch.
B. The boy asked the girl that he hadn’t broken her watch.
C. The boy told the girl that he didn’t break her watch.
D. The boy told the girl that he hadn’t broken your watch.
6. “Don’t make noise because I am listening music now” he said to me.
A. He asked me not to make noise because I am listening music now.
B. He asked me not to make noise because I was listening music then.
C. He asked me not to make noise because he was listening music then.
D. He asked me to make noise because I was listening music then.
7. “I have just seen your mother this morning”. Laura said to Lewis.
A. Laura told Lewis I have just seen your mother this morning.
B. Laura told Lewis she had just seen his mother that morning.
C. Laura told Lewis she has just seen his mother that morning.
D. Laura told Lewis he had just seen her mother that morning.
8. “We are ready to come with our friends” they said
A. They told us they are ready to come with their friends.
B. They told us they were ready to come with our friends.
C. They told us we were ready to come with our friends.
D. They told us they were ready to come with their friends.
9. “I was intending to meet you tomorrow” she said.
A. She told me she was intending to meet me tomorrow.
B. She told me she had intending to meet me the next day.
C. She told me she had been intending to meet me tomorrow.
D. She told me she had been intending to meet me the next day.
10. Mrs Smith: “ Don’t play in front of my windows”
A. Mrs Smith told us not to play in front of her windows.
B. Mrs Smith told us not to play in front of my windows.
C. Mrs Smith told us to not play in front of her windows.

118
D. Mrs Smith said us not to play in front of her windows.
11. “I didn’t witness that accident. ”
A. He denied not having witnessed that accident. B. He denied having witnessed that accident.
C. He denied not having witnessing that accident. D. He denied not had witnessed that accident.
12. “You cheated in the exam. ” The teacher said to his students
A. The teacher insisted his students on cheating in the exam.
B. The teacher prevented his students from cheating in the exam.
C. The teacher advised his students to cheat in the exam.
D. The teacher accused his students of cheating in the exam.
13. " Don't forget to give the book back to Mary," he said to me.
A. He reminded me to give the book back to Mary.
B. He reminded me to forget to give the book back to Mary.
C. He advised me to give the book back to Mary.
D. He advised me to forget to give the book back to Mary.
14. "Would you like to go to the cinema with me tonight?" he said.
A. He invited me to go to the cinema with him that night.
B. He offered me to go to the cinema with him tonight.
C. He asked me if I'd like to go to the cinema with him tonight.
D. He would like me to go to the cinema with him this night.
15. 'Remember to pick me up at 6 o'clock tomorrow afternoon," she said.
A. She told me to remember to pick her up at 6 o'clock tomorrow afternoon.
B. She reminded me to pick her up at 6 o'clock the following afternoon.
C. She reminded me to remember to pick her up at 6 o'clock the next afternoon.
D. She told me to pick her up at 6 o'clock the next day afternoon.
16. " Let's have a picnic next Saturday," Julia said.
A. Julia said that let's have a picnic the next Saturday.
B. Julia suggested having a picnic the following Saturday.
C. Julia advised how about having a picnic the next Saturday.
D. Julia told that why they didn't have a picnic next Saturday.
17. "If I were you, I'd tell him the truth," she said to me.
A. She said to me that if I were you, I'd tell him the truth.
B. She will tell him the truth if she is me.
C. She suggested to tell him the truth if she were me.
D. She advised me to tell him the truth.
18. “ Why don't you have your room repainted?" said Viet to Nam.
A. Viet suggested that Nam should have his room repainted.
B. Viet suggested having Nam's room repainted.
C. Viet asked Nam why you didn't have your room repainted.
D. Viet wanted to know why Nam doesn't have his room repainted.
19. "If I were you. Bill, I'd buy the house, " Stephen said.
A. Stephen suggested Bill to buy the house. B. Stephen advised Bill to buy the house.
C. Stephen promised Bill that he would buy the house. D. Stephen forced Bill to buy the house.
20. “Don’t forget to feed the chicken twice a day. ”
A. He said don’t forget to feed the chicken twice a day.
B. He told not to forget to feed the chicken twice a day.
C. He reminded me to feed the chicken twice a day.
D. He suggested me to feed the chicken twice a day

119
PRACTICE 3
1."Do you want to come to the party with me?" asked Sarah.
A. Sarah asked me if I wanted to come to the party with her.
B. Sarah asked me if I want to come to the party with her.
C. Sarah asked me if I wanted to come to the party with me.
D. Sarah asked me if I want to come to the party with me.
2."Where did you put my keys?" asked my mom.
A. My mom asked me where I put her keys. B. My mom asked me where did I put her keys.
C. My mom asked me where I had put her keys. D. My mom asked me where had I put her keys.
3"Why aren't you at work today?" asked my boss.
A. My boss asked me why I wasn't at work that day.
B. My boss asked me why aren't I at work that day.
C. My boss asked me why I wasn't at work today.
D. My boss asked me why wasn't I at work today.
4."Can you lend me some money?" asked my friend.
A. My friend asked me if I could lend him some money.
B. My friend asked me if can I lend him some money.
C. My friend asked me if I could lend me some money.
D. My friend asked me if can I lend me some money.
5."Did you see the news last night?" asked my dad.
A. My dad asked me if I saw the news the previous night.
B. My dad asked me if I see the news the previous night.
C. My dad asked me if I had seen the news the previous night.
D. My dad asked me if had I seen the news the previous night.
6. "What did you eat for breakfast?" asked my roommate.
A. My roommate asked me what did I eat for breakfast.
B. My roommate asked me what I have eaten for breakfast.
C. My roommate asked me what I had eaten for breakfast.
D. My roommate asked me what have I eaten for breakfast.
7."How do you get to the library from here?" asked the tourist.
A. The tourist asked me how I got to the library from there.
B. The tourist asked me how do I get to the library from there.
C. The tourist asked me how I get to the library from here.
D. The tourist asked me how do I get to the
8. Can you help me with this math problem?" asked my little sister.
A. My little sister asked me if can I help her with the math problem.
B. My little sister asked me if I could help her with the math problem.
C. My little sister asked me if I could help me with the math problem.
D. My little sister asked me if can I help me with the math problem.
9."What time does the train leave?" asked the passenger.
A. The passenger asked me what time the train left.
B. The passenger asked me what time does the train leave.
C. The passenger asked me what time the train was leaving.
D. The passenger asked me what time was the train leaving.
10."Have you seen my phone charger?" asked my roommate.
A. My roommate asked me if have I seen her phone charger.

120
B. My roommate asked me if I had seen her phone charger.
C. My roommate asked me if I had seen my phone charger.
D. My roommate asked me if have I seen my phone charger.
11."How many siblings do you have?" asked my new friend.
A. My new friend asked me how many siblings I had.
B. My new friend asked me how many siblings do I have.
C. My new friend asked me how many siblings I have had.
D. My new friend asked me how many siblings had I have.
12."Are you coming to the party tonight?" asked my neighbor.
A. My neighbor asked me if am I coming to the party that night.
B. My neighbor asked me if I was coming to the party that night.
C. My neighbor asked me if I was coming to the party tonight.
D. My neighbor asked me if was I coming to the party tonight.1
13."What do you want for dinner?" asked my mom.
A. My mom asked me what I wanted for dinner.
B. My mom asked me what did I want for dinner.
C. My mom asked me what I have wanted for dinner.
D. My mom asked me what have I wanted for dinner.
14."Do you have any plans for the weekend?" asked my coworker.
A. My coworker asked me if I had any plans for the weekend.
B. My coworker asked me if have I any plans for the weekend.
C. My coworker asked me if I had any plans on the weekend.
D. My coworker asked me if have I any plans on the weekend.
15. "Did you enjoy the concert?" asked my friend.
A. My friend asked me if did I enjoy the concert.
B. My friend asked me if I had enjoyed the concert.
C. My friend asked me if had I enjoyed the concert.
D. My friend asked me if I enjoyed the concert.
16."How long have you been studying English?" asked the teacher.
A. The teacher asked me how long have I been studying English.
B. The teacher asked me how long I had been studying English.
C. The teacher asked me how long I have been studying English.
D. The teacher asked me how long had I been studying English.
17."Did you finish your homework?" asked my teacher.
A. My teacher asked me if did I finish my homework.
B. My teacher asked me if I had finished my homework.
C. My teacher asked me if I have finished my homework.
D. My teacher asked me if have I finished my homework.
18. "What's your favorite type of music?" asked the DJ.
A. The DJ asked me what's my favorite type of music.
B. The DJ asked me what my favorite type of music was.
C. The DJ asked me what was my favorite type of music.
D. The DJ asked me what favorite type of music I had.
19.Why did you quit your job?" asked the interviewer.
A. The interviewer asked me why did I quit my job.
B. The interviewer asked me why I have quit my job.
C. The interviewer asked me why have I quit my job.

121
D. The interviewer asked me why I had quit my job.
20. "Can you lend me some money?" asked my friend.
A. My friend asked me if I could lend her some money.
B. My friend asked me if can I lend her some money.
C. My friend asked me if I can lend me some money.
D. My friend asked me if can I lend me some money.

  READING 
A. GAP – FILLING (CÂU 26-30 ĐỀ MH)
Read the following passage and mark the letter A, B, C, or D to indicate the correct word that
best fits each of the numbered blanks.
1. Millions of people of all ages enjoy a hobby which is both interesting and fun. And (1)
……… year, more and more people start a stamp (2)…………….of their own and discover an
interest which can last a lifetime. Starting your collection is easy (3)…….stamps can be found
everywhere. Holiday postcards from friends, birthday cards from relatives and letters from pen pals
can all (4)………you with stamps from all over the world. But once you have started collecting
seriously, you will probably want to join the Stamp Collectors’ Club (5)………….. exists to
provide collectors with new British stamps.
1. A. some B. every C. a few D. many
2. A. collect B. collector C. collection D. collecting
3. A. moreover B. although C. furthermore D. because
4. A. provide B. consider C. export D. select
5. A. where B. which C. when D. who
2. Keeping fit and healthy may seem difficult, but there are a few easy-to-follow guidelines.
Firstly, a balanced diet (1) ______ means selecting food that is low in salt and sugar is absolutely
essential. Experts recommend reducing the amount of fat in our diet, as too much can (2) ______
heart problems.
Secondly, it is important to (3) ______ exercise into our daily routine. This can be done by
simply walking as much as possible and climbing stairs instead of taking the lift. Exercise is
necessary to maintain a healthy body, as well as increasing energy levels and making you feel
generally fitter and happier.
Finally, staying relaxed is (4) ______ reason for good health. Too much stress can lead to a
variety of illnesses, from headaches to high blood pressure. Whenever possible, do thing you enjoy
and treat yourself occasionally. (5) ______, the message is simple – enjoy yourself but learn to
respect your body too. It’s all a question of getting a balance right. (Adapted from FCE
Use of English)
Question 1. A. who B. when C. whose D. which
Question 2. A. solve B. improve C. cause D. prevent
Question 3. A. fit B. match C. use D. search
Question 4. A. few B. another C. many D. much
Question 5. A. So B. Or C. Although D. Since
3. Ecotourism is booming and many tour operators say this is helpful to nature. (1)_______ year,
millions of people visit protected natural areas to observe rare species. (2) ________, a new
report casts doubt on this form of tourism. The report, published in the journal "Trends in
Ecology and Evolution", suggests that ecotourism damages more than (3) nature.
Researchers believe tourists disrupt animals in their natural (4) . They point to a recent
122
event in Costa Rica where turtles had problems laying their eggs because of the many tourists
(5)_______ had gathered on the beach to watch them.
1. A. Every B. Some C. Many D. Few
2. A. Therefore B. Moreover C. However D. Although
3. A. helps B. causes C. takes D. sees
4. A. habitable B. habitation C. habit D. habitat
5. A. which B. who C. where D. whose
4. Sailing or boating may seem to be very interesting with the high level of technical elements
involved in the sport. Whether you choose a small boat or a catamaran, whether you choose lake or
ocean sailing, there are (1) _____ number of things you need to master before you can start
enjoying the pure pleasure of being on the water.
Getting the sailing skills (2)_________ you need to start sailing your own boat is easy if you
just sign up with a sailing school. You can learn to sail at any age. There are no restrictions. The
best schools and instructors will teach you all you need to know about sailing in a friendly and
pleasant manner. As with all (3)_________, sailing will get you into contact with new friends.
There is a closeness that develops out at sea, however, which not many other sports have. Being
on the water in any capacity demands a healthy outlook and way of life. Sailing and boating are
two sports that (4)_____ all our senses to be at their peak. Sailing calls upon physical strength,
intelligence and intuition. It is one sport that allows you to leave all your daily concerns
(5)_______ habits back on the land and to be off with only your bare self.
1. A. a B. an C. some D. Ø
2. A. where B. which C. whom D. why
3. A. programs B. actions C. sports D. movements
4. A. demand B. make C. take D. suggest
5. A. also B. so C. though D. and

5. The attitude against women had its (1) _______ from the beginning of mankind’s history when men
lived in caves and went hunting for food. The task of hunting needed great strength of body. (2) _______,
the best place for women was not in forests, but at home where they could satisfactorily do their job, such
as doing household chores and looking after children. In our modern time, there are more and more jobs
(3)______ do not need the strength of muscles. It is a consequence that women have played a/an (4)
_______ important role in the society. However, there are still many backward places, especially in Asian
countries. Most illiterates are females. A man often expects his wife and daughters to stay at home, serve
him, and obey him in (5)_______ things. Women are considered as inferior to men. Young girls are
educated to tolerate and accept intimate partner violence as a part of tradition.
*Sexism: thành kiến về giới tính.
1. A. evolution B. origin C. organization D. effect
2. A. Therefore B. But C. Nevertheless D. And
3. A. whom B. whose C. this D. that
4. A. repeatedly B. increasingly C. tiringly D. extensively
5. A. all B. much C. a lot D. little
6. Tourism is now one of the largest industries in the world, and is the main source of income in
(1) ____ countries. It may bring in earnings, create jobs for local people and improve their life as
a whole. Unfortunately, the construction of essential facilities to (2) ____ the increasing need of
123
visitors has destroyed natural habitats, thus threatening wildlife and spoiling some of the world’s
most beautiful regions. Air travel to distant places results in huge amounts of carbon emissions (3)
____ contribute to global warming.
The majority of holidaymakers are unaware of such issues, (4) ____ fortunately there are people
who would rather travel with environmental responsibility. For these travellers, there is now an
alternative to conventional tourism, known as ecotourism. They choose their travelling carefully,
and try to make sure that their visit will have only a (5) ____ effect on the environment.
(Adapted from Master Mind)
Question 1. A. little B. another C. every D. many
Question 2. A. bring B. take C. meet D. look
Question 3. A. who B. which C. where D. when
Question 4. A. but B. even C. because D. moreover
Question 5. A. harmful B. significant C. destructive D. minimal

B. CHOOSE THE CORRECT ANSWER. (5 CÂU HỎI) (= CÂU 31 35 ĐỀ MH)


1. Read the following passage and mark the letter A, B, C, or D on your answer sheet to
indicate the correct answer to each of the questions.
Sweden can be one of the first countries to stop using cash. In most cities, buses do not take
any cash. Instead, you pay for your tickets by card or mobile phone before you get on. Some
businesses only take cards and there are banks that do not deal with any notes or coins.
In a cashless society, you do not need to worry about having enough change for the bus.
You will feel much more secure as well. Statistics show that since the decrease in the cash
economy, there is not as much crime in Sweden.
However, not all Swedes like the idea. Elderly people complain much about the problems of
paying by card. Small business are also unhappy because they have to pay a little money to the
bank each time somebody pay them using a card.
It is not just richer countries that are becoming cashless. Many countries in the developing
world have found that mobile phones are perfect for transferring money. For example, in Kenya,
only a few people have a bank account, but lots of Kenyan have mobile phone. Now people can
pay for things in shops, pay their bills and their children’s school fees, and receive their salary with
their phones. Kenya has led a revolution in mobile banking. (Adapted from
Navigate)
1. Which of the following can be the best title for the passage?
A. Bank Notes Then Cash? B. Why Once Again Sweden?
C. The End of Cash? D. A New Currency Unit For Kenya?
2. According to the passage, in a cashless society ______.
A. people will travel more by bus B. elderly people will become more active
C. you will feel much safer D. businesses will earn more money
3. The word ‘they’ in paragraph 3 refers to ______.
A. the problems B. all Swedes C. small businesses D. elderly people
4. The word ‘revolution’ in paragraph 4 mostly means ______.
A. an important change B. an unsuccessful attempt
C. a frequent movement D. a violent struggle
5. Which of the following is NOT true, according to the passage?
A. People in Sweden can pay for bus tickets by mobile phone.
B. Few people in Kenya own a mobile phone.
C. People in Kenya can pay school fees with their phones.
124
D. Sweden may be among the first countries to stop using cash.
2. Sheena Southall, a 22-year-old student, was keen to test her limit when she agreed to appear on
a TV show. It has been described as one of the most fascinating shows in the world.
She saw an advert which said: “Do you want to make some quick cash and do you want to
travel?” She talked to herself: “Why don’t I have a try? I’m always looking for an opportunity to
test myself.” So, she applied for the job, went for an interview, and got it. At the interview, she
explained how she was prepared for any challenges. Then, just as she was leaving for Singapore
she found out that she was going to become a skyscraper window cleaner.
The job was challenging. She had to pull herself up and down carrying heavy water buckets. If
she didn’t clean the windows properly, they would tell her to go back and do it again. But she
admits the experience has changed her. As soon as she got back, she finished her studies and
graduated. She started hunting for a job that would be both exciting and tough. Sheena says:
“Nothing will ever be as exciting and scary as that first day of cleaning windows up in the air. But
if you don’t face your fears and accept what life offers you, how will you know what you are
capable of? It has been the best experience of my life so far.” (Adapted from Gateway)
1. The passage is mainly about….
A. a TV show about young people in Singapore
B. a challenging experience of a young student
C. a young girl's difficulties in earning money
D. an experience of skyscraper window cleaners
2. The word It in paragraph 1 refers to____.
A. the world B. an advert C. a TV show D. her limit
3. According to the passage, Sheena wanted to appear on the show in order to____.
A. complete her study B. become a cleaner C. explore her limit D. attend an interview
4. The word tough in paragraph 3 is closest in meaning to____.
A. tiring B. boring C. negative D. difficult
5. According to paragraph 3, Sheena did all of the following EXCEPT____.
A. dropping out of school B. hunting for a job
C. cleaning the windows D. carrying heavy water buckets

C. CHOOSE THE CORRECT ANSWER. (7 CÂU HỎI = CÂU 36  42 ĐỀ MH)


Read the following passage and mark the letter A, B, C, or D on your answer sheet to indicate
the correct answer to each of the questions.
1. It is often held that technology in general does little to help family relationships as, in many
cases, family members are more interested in their smartphones and the TV than they are in each
other. Before smartphones the TV was blamed the most for breakdowns in communication
between parents and children. It was argued that “the box” stop them from talking to each other, and
clashes between family members over what programs to watch create barriers and cause resentment.
However, TV programs can be booth informative and entertaining. They often provide
topics for family members to discuss, bring them closer together rather than driving them further
away. What is more, twenty-first century families do not need to watch the same program at the
same time. The fact that people can watch what they want results in more time for families to be
together rather than less.
The smartphone has taken over from the TV as a major reason for the lack of
communication between family members. Parents often do not see why their children spend so
much time online and fail to see the importance of having an online identity while most teens feel
125
their online image is part of who they are. They often criticize the amount of time their children
spend online rather than attempting to understand why they do it. In fact, adults are often not very
good examples themselves. Who has not witnessed a table at a restaurant where all the family
members are silently focused on their mobile devices?
Broadly speaking, smartphones do have some negative effects on family relationships. This
technology is still relatively new, so many families have not yet learned how to deal with it.
Parents and children should discuss how they use technology to help them all to make the most of
technology instead of blaming it for their problems. (Adapted from High Note)
1. Which of the following can be the title for the passage?
A. How to Deal with Generation Gap in the Family
B. How Technology Affects Family Relationships
C. Negative Impacts of Technology on Schoolchildren
D. TV and Smartphones as Means of Communication
2. The word ‘clashes’ in paragraph 1 is the closest in meaning to ______.
A. chances B. benefits C. arguments D. worries
3. According to paragraph 1, the TV ______.
A. makes family members become more interested in each other
B. is often believed to be greatly helpful in fostering family bonds
C. encouraged more communication among family members
D. was claimed to prevent parents from conversing with children
4. The word ‘them’ in paragraph 2 refers to ______.
A. family members B. barriers C. topics D. TV programs
5. The word ‘negative’ in paragraph 4 is the closest in meaning to ______.
A. positive B. minimal C. interesting D. harmful
6. Which of the following is NOT true, according to the passage?
A. The lack of communication in the family is now mostly blamed on the TV.
B. The majority of teenagers regard their online image as important to their identity.
C. The TV can serve as a source of both information and entertainment.
D. Parents and children should work on how to utilize technology to their advantage.
7. Which of the following can be inferred from the passage?
A. Most families prefer talking to one another to using smartphones in restaurants.
B. Twenty-first century families are generally not in favor of watching television together.
C. Parents in the past did not criticize their children for watching too much television.
D. A lot of families are still incapable of making god uses of smartphones.
2. Do you find yourself yawning all day at school? You probably need more sleep. However, it is
not just how much sleep you get that is important. What you do before bedtime also has a huge
effect on how you feel the next day. If you spend time chatting to someone on social media, or
reading a book on your e-reader, you are not alone.
All devices’ screens give out a large amount of blue light, which affects the body’s production
of the sleep hormone, melatonin, which can change your sleep patterns. So it is not just getting to
sleep that becomes hard, but the quality of your sleep also suffers. The result is that you can wake
up several times during the night, and have difficulty getting up in the morning.
Sleep is food for the brain, so not sleeping well can be a severe problem for anyone, especially
for teenagers, because their brains are still developing. When you do not sleep well, it can be
difficult to concentrate at school, which makes it harder to learn and get good grades. It can also
affect your mood, making you feel sad or anxious.
So, it is clear you should not use your phone before you go to sleep. Why not leave it in another
126
room? Nothing on social media is so important that it cannot wait a few hours! You should
rediscover paper books and enjoy feeling tired and sleepy before bedtime rather than wondering
why your best friend has not liked your photo! How will you wake up? You do not have to use the
alarm on your phone. Go and buy yourself an old-fashioned alarm clock! The negative effects that
screen time is having on sleep have been in the news a lot recently, so manufacturers of
smartphones and tablets have added a feature to the mobile devices that can help reduce these.
Most new phones have a night-time setting that changes the colour of the screen so that it gives
out less blue light. If you really cannot avoid going onto social media at bedtime, think about
turning on this setting on your phone. (Adapted from Prepare)
Question 36: Which of the following can be the best title of the passage?
A. New Devices - Old Functions B. Inventions before Social Media
C. Screen Time before Bedtime D. Screen Time - Best Time
Question 37: Blue light from digital screens affects…….
A. the way people use their devices B. people’s reading habits
C. how people communicate D. the body’s production of melatonin
Question 38: The word suffers in paragraph 2 is closest in meaning to.
A. gets longer B. improves C. maintains D. becomes worse
Question 39: The word severe in paragraph 3 is closest in meaning to.
A. potential B. serious C. basic D. obvious
Question 40: The word it in paragraph 4 refers to.
A. alarm B. bedtime C. your phone D. your photo
Question 41: Which of the following is NOT TRUE according to the passage?
A. Sleeping well is very important to the development of the brain.
B. Reading printed books makes it difficult to fall asleep.
C. Exposure to blue light before bedtime does harm to sleep quality.
D. Poor sleep quality is likely to result in sadness and anxiety.
Question 42: Which of the following can be inferred from the reading passage?
A. The effects of blue light on sleep are still controversial.
B. Tech companies have taken users' health into consideration.
C. An old-fashioned alarm clock is more popular than a new phone.
D. Chatting to others on social media should be avoided, especially for teenagers.
_____HẾT_____

CÁC ĐỀ THAM KHẢO THEO FORM ĐỀ MINH HỌA

ĐỀ 1
Mark the letter A. B. C. or D on your answer sheet to indicate the word whose underlined
part differs from the other three in pronunciation in each of the following questions.
Câu 1: A. identify B. practical C. habitat D. gorilla
Câu 2: A. focus B. impact C. pesticide D. discuss
Mark the letter A. B. C. or D on your answer sheet to indicate the word that differs from the
other three in the position of stress in each of the following questions.
Câu 3: A. struggle B. answer C. confide D. comfort
Câu 4: A. inflation B. maximum C. applicant D. character

127
Mark the letter A. B. C. or D on your answer sheet to indicate the correct answer to each of the
following questions.
Câu 5: The Sears Tower is ______ building in Chicago.
A. the most tall B. the more tall C. the tallest D. the taller
Câu 6: We have to suffer a lot of floods due to our serious ______ of forests.
A. destroy B. destruction C. destructive D. destructor
Câu 7: _____ bikes on the street, they saw too much litter.
A. Riding B. Ride C. To ride D. Is riden
Câu 8: During the flood, Army helicopters came and tried to evacuate ______ injured.
A. a B. an C. the D. Ø
Câu 9: They decided to ______ their journey till the end of the month because of the epidemic.
A. take up B. turn round C. put off D. do with
Câu 10: The injured ______ to the hospital in an ambulance.
A. were taken B. were taking C. take D. have taken
Câu 11: Students are not allowed ______ jeans at school.
A. to wear B. wearing C. worn D. wear
Câu 12: Tom couldn’t come to our party, ______ ?
A. couldn't he B. could he C.hadn't he D. didn't he
Câu 13: The government should take some measures to______ that tourism develops in harmony
with the environment.
A. promote B. ensure C. facilitate D. improve
Câu 14: With a good ______ of both Vietnamese and English, Miss Loan was assigned the task of
oral interpretation for the visiting American delegation.
A. insight B. knowledge C. command D. proficiency
Câu 15: I broke my teeth when I ______ dinner.
A. had B. was having C. am having D. am having
Câu 16: John will look for a job ____.
A. after he had passed his exams B. as soon as he passes his exams
C. while he was passing his D. before he passed his exams
Câu 17: Tourism is changing rapidly as nature, heritage, and recreational destinations become
more important, and as conventional tourism is forced to ______ tougher environmental
requirements.
A. meet B. impose C. lay D. set
Câu 18: Sometimes she does not agree ______ her husband about child rearing but they soon find
the solutions.
A. for B.on C. with D. of
Câu 19: Jimmy always takes the ______ by the horns at every chance in order to become a famous
pop star, which is why he is so successful now.
A. horse B. bull C. cow D. buffalo
Mark the letter A. B. C. or D on your answer sheet to indicate the word CLOSEST in meaning
to the underlined word in each of the following questions.
Câu 20: Mitchell appeared at the door, asking me to lend him some money.
A. saw B. arrived C. knew D. smile
Câu 21: There's a feeling of accomplishment from having a job and all that goes with it.
A. involvement C. prosperity
B. achievement D. contribution
Mark the letter A. B. C. or D on your answer sheet to indicate the word(s) OPPOSITE in
128
meaning to the underlined word(s) in each of the following questions.
Câu 22: Those who advocate for doctor-assisted suicide say the terminally ill should not have to
suffer.
A. support B. oppose C. annul D. convict
Câu 23: Most families in my village have been on the breadline since the covid-19 pandemic
broke out.
A. healthy B. exhausted C. wealthy D. poor
Mark the letter A. B. C. or D on your answer sheet to indicate the sentence that best completes
each of the following exchanges.
Câu 24: Peter and Mary are talking about what to do after class.
- Peter: “_________” - Mary: “Yes, I love to.”
A. Do you often have time for a drink after class?
B. Do you often go out for a drink after class?
C. Would you like tea or coffee after class?
D. Would you like to have a drink after class?
Câu 25: A student is asking the librarian to help her to fax a report.
- Student: “Could you help me to fax this report?” - Librarian: “_________”
A. Certainly, what’s the fax number? B. Sorry, I have no idea.
C. It’s very kind of you to say so. D. What rubbish! I don’t think it’s helpful.
Read the following passage and mark the letter A. B. C. or D on your answer sheet to indicate
the correct word or phrase that best fits each of the numbered blanks from 26 to 30.
In such a costly and competitive society and world, no one of us can live without money. We need
money to fulfill our basic needs of the life such as buying food, and other (26) _______ basic
necessities of life which are almost impossible to buy without money. People in the society
(27)______are rich and have property are looked as honourable and respectful person of the
society however a poor person is seen as hatred without any good impression.
Money increases the position of the person in the society and (28)______a good impression to him.
All of us want to be rich by earning more money through good job or business in order to fulfil all
the increasing demands of the modern age. (29)______, only few people get this chance of
completing their dreams of being a millionaire.
So, money is the thing of great importance all through the life. Money is required by everyone
whether he/she is rich or poor and living in urban areas or rural areas. People in the urban areas are
earning more money than the people living in backward or rural areas as the people of the urban
areas have more (30)______to the technologies and get more opportunity because of the easy
sources. (Adapted from https://www.indiacelebrating.com)
Câu 26: A. little B. many C. anther D. every
Câu 27: A. where B. what C. which D. who
Câu 28: A. does B. takes C. gives D. draws
Câu 29: A. However B. Besides C.Therefore D.Moreover
Câu 30: A. way B. exit C. access D. order
Read the following passage and mark the letter A. B. C. or D on your answer sheet to indicate
the correct answer to each of the questions from 31 to 35.
In 2007, at a heavily hyped press event in San Francisco, Apple co-founder Steve Jobs stood on
stage and unveiled a revolutionary product that not only broke the mould but also set an entirely
new paradigm for computer-based phones. The look, interface and core functionality of nearly

129
every smartphone to come along since is, in some form or another, derived from the original
iPhone’s innovative touchscreen-centric design.
Among some of the ground-breaking features was an expansive and responsive display from which
to check email, stream video, play audio, and browse the internet with a mobile browser that
loaded full websites, much like what is experienced on personal computers. Apple’s unique iOS
operating system allowed for a wide range of intuitive gesture-based commands and eventually, a
rapidly growing warehouse of downloadable third-party applications.
Most importantly, the iPhone reoriented people’s relationship with smartphones. Up to then, they
were generally geared toward businesspeople and enthusiasts who saw them as an invaluable tool
for staying organized, corresponding over email, and boosting their productivity. Apple’s version
took it to a whole other level as a full-blown multimedia powerhouse, enabling users to play
games, watch movies, chat, share content, and stay connected to all the possibilities that we are all
still constantly rediscovering. (Adapted from https://www.thoughtco.com/)
Câu 31: Which best serves as the title for the passage?
A. Apple co-founder Steve Jobs B. Apple’s iPhone
C. Apple’s unique iOS operating system D. press event in San Francisco
Câu 32: The word “paradigm” in paragraph 1 is closest in meaning to ______.
A. media B. role C. pattern D. area
Câu 33: Which is NOT mentioned in paragraph 1 as a characteristic of the new iPhone?
A. new look B. interface C. core functionality D. excellent sound
Câu 34: The word “that” in paragraph 1 refers to ______.
A. audio B. mobile browser C. internet D. email
Câu 35: The iPhone reoriented people’s relationship with smartphones and geared toward ____.
A. businesspeople B. organizers C. gamers D. producers

Read the following passage and mark the letter A. B. C. or D on your answer sheet to indicate
the correct answer to each of the questions from 36 to 42.
Today, Snyder is seeing the fruits of the collaboration. In a recent issue of the Journal of
Comparative Psychology (Vol. 117, No. 3), the research group published the first study of giant
panda cub-rearing and separation. The study is the first step in examining the impact of a common
breeding practice in China: separating captive cubs before they are six months old so that the
mothers will be able to reproduce again sooner. Cubs in the wild stay with their mothers for 1.5 to
2.5 years.
Snyder and her co-authors, including Maple and psychologist Mollie Bloomsmith, PhD, theorize
that separating cubs from their mothers too early may harm their social development, and could
underlie why so many captive pandas fail to breed. Captive males often show little sexual interest
in females or are too aggressive.
The research team has also conducted urinary and behavioral analyses of female giant pandas
during the breeding season, which is generally in the spring, and is examining the behavior of giant
panda mothers. Graduate student Megan Wilson, who also has worked in Chengdu, is investigating
sequences of play-fighting for her dissertation. And former graduate student and Chengdu
researcher Loraine Tarou, PhD, examined giant panda cognition.
Tarou, now an assistant professor at Grand Valley State University in Michigan, is the first to
examine how captive pandas forage for food and learn to adapt to changes in their environment.
She compared her findings with the cognition of the spectacled bear and found that while both use
spatial memory to find food, the spectacled bears used visual cues the pandas did not pick up on.

130
The finding indicates that giant pandas' reliance on spatial memory alone may cause them to have
difficulty when their food sources are abruptly changed or moved--a big problem for an animal that
consumes nearly 30 pounds of bamboo a day.
While such basic research may not have immediate application, says Tarou, it is contributing to
scientists' growing knowledge of the species. (Adapted from https://www.apa.org/)
Câu 36: Which of the following best serves as the title for the article?
A. Understanding breeding practice of pandas B. Learning about pandas
C. Snyder and her co-authors’ theory D. How captive pandas forage for food
Câu 37: Naturally, panda cubs stay with their mothers for ______.
A. 1 to 1.5 years B. 1.5 to 2.5 years C. 2.5 to 3 years D. 3 to 4 years
Câu 38: The word “underlie” in paragraph 2 most probably means ______.
A. explain B. obtain C. generate D. develop
Câu 39: What analyses has been done on female giant pandas during the breeding season?
A. urinary and behavioral analyses B. spatial memory analyses
C. environment interaction analyses D. food analyses

Câu 40: The word “conducted” in paragraph 3 is closest in meaning to ______.


A. look for B. get on C. carry out D. set off
Câu 41: The word “them” in paragraph 5 refers to ______.
A. difficulties B. giant pandas C. memories D. food sources
Câu 42: Which of the following is infered according to the article?
A. Separating cubs from their mothers early may be beneficial to their social development.
B. Megan Wilson and Loraine Tarou examined panda breeding season.
C. Captive panda cubs in China are often separated from mothers before they are six months.
D. A panda consumes nearly 300 pounds of bamboo a day.
Mark the letter A. B. C. or D on your answer sheet to indicate the underlined part that needs
correction in each of the following questions.
Câu 43: My close friends spend most of their free time helping the homeless people in the
community last year.
A B C D
Câu 44: Some animals are in danger of extinction as humans are hunting it for meat.
A B C D
Câu 45: It is essential to provide high school students with adequate career orientation, so they
A B
can make more informative decisions about their future major.
C D
Mark the letter A. B. C. or D on your answer sheet to indicate the sentence that is closest in
meaning to each of the following questions.
Câu 46: The last time I ate spaghetti was five months ago.
A. I haven’t eaten spaghetti for five months. B. I didn’t eat spaghetti five months ago.
C. I would eat spaghetti in five months. D. I have eaten spaghetti for five months.
Câu 47: “I had lunch with my friends last month,” Jean said.
A. Jean said that she had had lunch with her friends the previous month.
B. Jean said that I had had lunch with my friends last month.
C. Jean said that I had lunch with her friends last month.
D. Jean said that she would have lunch with her friends last month.
131
Câu 48: It is compulsory for every student in my class to finish their homework before class.
A. Every student in my class must finish their homework before class.
B. Every student in my class needn’t finish their homework before class.
C. Every student in my class can’t finish their homework before class.
D. Every student in my class could finish their homework before class.
Mark the letter A. B. C. or D on your answer sheet to indicate the sentence that best combines
each pair of sentences in the following questions.
Câu 49: I lost her phone number. I couldn’t call her.
A. If I had her phone number, I could have called her.
B. If I hadn’t lost her phone number, I could call her.
C. If I hadn’t had her phone number, I could have called her.
D. If I had had her phone number, I could call her.
Câu 50: Emily had just finished saving all the documents. The computer crashed then.
A. Hardly had Emily finished saving all the documents when the computer crashed.
B. The moment Emily started to save all the documents, the computer crashed.
C. Had it not been for the computer crash, Emily could have saved all the documents.
D. No sooner had the computer crashed than Emily finished saving all the documents.

ĐỀ 2
Mark the letter A, B, C, or D on your answer sheet to indicate the sentence that best completes
each of the following exchanges.
Question 1: -“Let’s travel to Da Lat city this weekend.” - “___________________.”
A. It’s bad B. That’s good idea C. Nothing D. Yes, we can
Question 2: Tim and Lan are talking about school exams.
Tim: “I think school exams should be replaced by presentations and projects to assess students
more effectively.” - Lan: “________________.”
A. I totally agree B. No, it’s good C. I share your view D. I don’t like ít
Mark the letter A, B, C, or D on your answer sheet to indicate the correct answer to each of the
following questions.
Question 3: My hometown is ________ place I’ve ever known.
A. more peaceful B. peaceful C. the most peaceful D. the more peaceful
Question 4: All applicants will be considered regardless of age, sex, religion or ________.
A. nationality B. national C. native D. nationally
Question 5: Scary stories ________ by Edgar Allan Poe is popular.
A. wrote B. writing C. to write D. written
Question 6: My mom is always in ________ kitchen whenever I come home.
A. a B. the C. no article D. B & C are correct
Question 7: He ________ some excuse about his daughter being sick.
A. made up B. looked into C. looked after D. put away
Question 8: He stopped ________ two years ago. He had realized his heavy cough then.
A. to smoke B. to smoking C. smoking D. smoke
Question 9: Let’s go to the cinema tonight, ________?
A. don’t we B. do you C. shall we D. do they

132
Question 10: The training programme aims at _________ employees’ awareness about human
rights.A. rising B. raising C. doing D. taking
Question 11: Jim will tell me the truth _________.
A. as soon as he came back to Viet Nam B. after he had come back to Viet Nam.
C. as long as he comes back to Viet Nam D. until he comes back to Viet Nam.
Question 12: I haven’t had the chance to take my lunch ________ today due to the busy workload
and ongoing tasks.
A. gap B. break C. pause D. interval
Question 13: My friends ________ in Nha Trang city when I met them on my business trip.
A. were travelling B. has travelled C. travelled D. are travelling
Question 14: The electricity bill ________ by my father yesterday.
A. paid B. was paying C. was paid D. had been paid
Question 15: A Chinese company has offered to build three ______ tram routes on Hanoi’s Ring
Road No.3 and Thang Long Avenue to help ease traffic congestion.
A. tracked B. guided C. railed D. trackless
Question 16: It’s dangerous ________ her to sleep alone in a tent.
A. for B. with C. to D. of
Question 17: During the meeting, Sarah accurately identified the root cause of the problem. She
really hit ________ on the head.
A. the finger B. the thumb C. the leg D. the nail
Mark the letter A, B, C, or D on your answer sheet to indicate the word whose underlined part
differs from the other three in pronunciation in each of the following questions.
Question 18: A. walk B. call C. take D. talk
Question 19: A. consist B. suggest C. discuss D. realise
Mark the letter A, B, C, or D on your answer sheet to indicate the word that differs from the
other three in the position of stress in each of the following questions.
Question 20: A. modern B. polite C. famous D. angry
Question 21: A. chocolate B. telephone C. butterfly D. computer
Mark the letter A, B, C, or D on your answer sheet to indicate the word CLOSEST in meaning to
the underlined word in each of the following questions.
Question 22: He’s a chaotic sort of a person - always trying to do twenty things at once.
A. disordered B. careful C. particular D. honest
Question 23: Such extravagance is shameful when there are people starving in the world.
A. meaning B. luxury C. excess D. popularity
Mark the letter A, B, C, or D on your answer sheet to indicate the word(s) OPPOSITE in
meaning to the underlined word(s) in each of the following questions.
Question 24: There are effective treatment methods for cardiovascular diseases, including
medication, surgery, and supportive measures.
A. extra-cardiac B. circulatory C. cardiac D. vascular
Question 25: We left late, but thanks to the empty roads, we made good time and arrived at our
destination ahead of schedule.
A. moved quickly B. moved slowly C. drove fast D. climbed slowly
Read the following passage and mark the letter A, B, C, or D on your answer sheet to indicate
the correct word or phrase that best fits each of the numbered blanks from 26 to 30.

133
Over the past two centuries, different generations were born and given different names. Each
generation comes with its characteristics, (26) _______ are largely influenced by the historical,
economic, and social conditions of the country they live in. However, in (27) _______ countries
the following three generations have common characteristics.
Generation X refers to the generation born between 1965 and 1980. When Gen Xers grew up,
they experienced many social changes and developments in history. (28)_______, they are always
ready for changes and prepared to work through changes. Gen Xers are also known as (29)
_______ thinkers because they achieved higher levels of education than previous generations.
Generation Y, also known as Millennials, refers to those born between the early 1980s and late
1990s. They are (30) _______ and ready to accept changes. If there is a faster, better way of doing
something, Millennials want to try it out. They also value teamwork. When working in a team,
Millennials welcome different points of view and ideas from others.
(Adapted from Grade 11 student book)
Question 26: A. when B. which C. who D. that
Question 27: A. many B. some C. few D. much
Question 28: A. So B. However C. As a result D. B & C are correct
Question 29: A. critical B. good C. special D. common
Question 30: A. willing B. famous C. bad D. curious
Read the following passage and mark the letter A, B, C, or D on your answer sheet to indicate
the correct answer to each of the questions from 31 to 35.
Gender education is one of the most pressing issues in society today. Adolescent students are
vulnerable to issues such as rape, underage pregnancy, and other related concerns. Therefore,
gender education plays a crucial role in fostering a society that embraces equality and empowers
individuals irrespective of their gender identities. By providing comprehensive knowledge and
understanding of gender-related issues, this form of education aims to challenge stereotypes,
promote inclusivity, and empower individuals to make informed choices.
Hanoi police have launched an investigation into a case in which a man was accused of raping a
girl on various occasion, and getting her pregnant. The alleged rapist, whose identity has not been
revealed, is now under house arrest, police in Thanh Tri District announced on Sunday. He is a
neighbor of the girl, now 12.
In early January, the girl’s family detected abnormal signs in her physical health, including a
bigger belly, so they took her to a hospital for a checkup. Only then did they learn she was six-
months pregnant. The girl later said the neighbor had raped her on several occasions.
On April 17, she was taken to Hanoi Obstetrics & Gynecology Hospital for a C-section. Those
found guilty of raping a minor can receive a jail sentence of 7 to 15 years. They can receive a death
sentence for gang rape, if the act causes serious physical or mental damage, or if the victim is less
than 10 years old.(Adapted from vnexpress)
Question 31: What is the best title of the passage?
A. Gender education and its urgency.
B. Adolescent students’ underage pregnancy.
C. Adolescent students and social stereotypes.
D. The victim is less than 10 years old and her mental damage.
Question 32: The word their in paragraph 1 refers to _________.
A. boys B. adolescent students C. girls D. adults
Question 33: Which of the following is NOT mentioned in gender education?

134
A. Providing comprehensive knowledge and understanding of gender-related issues.
B. Challenging stereotypes, promote inclusivity and empower individuals.
C. Raising awareness the danger of adolescent students’ gender.
D. The girl who is the victim of raping is 12 years old.
Question 34: The word raping in paragraph 2 is closest in meaning to_________.
A. protecting B. respecting C. safeguarding D. abusing
Question 35: Which is true about gang rape who raped the victim is less than 10 years old?
A. a death sentence B. life imprisonment C. suspended sentence D. innocence
Read the following passage and mark the letter A, B, C, or D on your answer sheet to indicate
the correct answer to each of the questions from 36 to 42.
Mass media, including television, the internet, and social media, plays a significant role in the
lives of teenagers today. While it offers several advantages, it also presents some disadvantages.
One of the key advantages of mass media for teenagers is that it provides access to a vast
amount of information. Through newspapers, TV news, and online platforms, teenagers can stay
updated on current events and expand their knowledge on various subjects. Moreover, mass media
facilitates communication and connection among teenagers around the globe, allowing them to
share ideas and experiences.
Additionally, mass media offers entertainment and relaxation. Television shows, movies, and
online videos provide a means of escapism from the stress of daily life for teenagers. They can also
discover new forms of art, music, and literature, fostering creativity and cultural appreciation.
However, there are also disadvantages to consider. One major concern is the potential for
exposure to inappropriate or harmful content. With unrestricted access to the internet, teenagers
may come across explicit material, violence, or misinformation. Moreover, excessive use of mass
media can lead to sedentary lifestyles, contributing to physical health problems such as obesity and
poor eyesight. Another drawback is the impact on mental health. Constant exposure to idealized
images and lifestyles portrayed in mass media can create unrealistic expectations and lower self-
esteem among teenagers. It may also contribute to cyberbullying and online harassment, affecting
their emotional well-being.
In conclusion, mass media offers numerous advantages for teenagers, including access to
information, entertainment, and global connectivity. However, it is crucial for parents and
guardians to guide and monitor their media consumption to mitigate the potential disadvantages.
By striking a balance, teenagers can harness the benefits of mass media while protecting their well-
being. (Adapted from Internet)
Question 36: Which of the following can be the best title of the passage?
A. Mass media provides relaxation entertainment.
B. The advantages and disadvantages of mass media for teenagers.
C. Mass media causes cyberbullying and online harassment.
D. Mass media offers numerous advantages for teenagers.
Question 37: The phrase them in the second paragraph is closest in meaning to _________.
A. teenagers B. adults C. children D. students
Question 38: What does the word unrestricted in the fourth paragraph refer to?
A. limited B. constrained C. uncontrolled D. controlled
Question 39: According to the passage, how many key advantages does media bring to teenagers?
A. four B. three C. seven D. five
Question 40: The phrase self-esteem in the fourth paragraph is closest in meaning to _________.

135
A. self-assurance B. self - doubt C. low self - worth D. A & B are correct
Question 41: Which of the following is NOT TRUE according to the passage?
A. Mass media causes obesity and poor eyesight.
B. Mass media only brings benefits and has no disadvantages.
C. It is important for parents and guardians to guide and monitor teenagers’ media consumption.
D. Teenagers can control the benefits of mass media.
Question 42: Which of the following can be inferred from the reading passage?
A. Teenagers can benefit greatly from mass media, but it is important for parents or teachers to
guide them and prevent its abuse.
B. The pros of mass media to teenagers’ studying.
C. The mass media affects teenager’s emotional well-being.
D. Mass media can create unrealistic expectations and laziness.
Mark the letter A, B, C, or D on your answer sheet to indicate the sentence that best combines
each pair of sentences in the following questions.
Question 43: You finish your homework. You will be allowed to play video games.
A. Not until you finish your homework will you be allowed to play video games.
B. Only when you will be allowed to play video games do you finish your homework.
C. No sooner did you be allowed to play video games than you finish your homework.
D. Not until you will be allowed to play video games than you finish your homework.
Question 44: Since our products were bad quality, fewer people bought them.
A. If our products were bad quality, more people bought them.
B. If our products had not been bad quality, more people would have bought them.
C. If our products had been bad quality, more people would have bought them.
D. If our products were bad quality, more people would have bought them.
Mark the letter A, B, C, or D on your answer sheet to indicate the underlined part that needs
correction in each of the following questions.
Question 45: This research is conducted by Dr. Park last year.
A B C D
Question 46: Everyone should be cautious with her belongings when traveling alone.
A B C D
Question 47: Internet users have been following a trend of searching for debts in Ms. Trương Mỹ
A B C
Lan’s sea in the past few days, which is not feasible.
D
Mark the letter A, B, C, or D on your answer sheet to indicate the sentence that is closest in
meaning to each of the following questions.
Question 48: It isn’t essential for me to attend the meeting tomorrow.
A. I needn’t attend the meeting tomorrow.
B. I should attend the meeting tomorrow.
C. I may attend the meeting tomorrow.
D. I have to attend the meeting tomorrow.
Question 49: The last time my husband and I visited Paris was one year ago.
A. My husband and I visited Paris for one year ago.
B. My husband and I have visited Paris for one year.

136
C. My husband and I haven’t visited Paris for one year.
D. My husband and I have been visited Paris for one year.
Question 50: “My boyfriend has received a scholarship in the UK.” She said
A. She said my boyfriend had received a scholarship in the UK.
B. She said her boyfriend had received a scholarship in the UK.
C. She said her boyfriend has received a scholarship in the UK.
D. She said my boyfriend received a scholarship in the UK.

ĐỀ 3
Mark the letter A, B, C, or D on your answer sheet to indicate the sentence that best completes
each of the following exchanges.
Question 1: Toan and Sam are talking to each other.
Toan : “I think the important thing is to listen to what others say and share our thoughts
respectfully.” Sam : “________”
A. I couldn't agree more. B. I think you’re wrong. C. You must be kidding. D. My bad.
Question 2: Quy and Gilly are going jogging in the park together.
Jack : “Hey, I'm thinking about trying out a new restaurant this weekend. Would you like to join
me?Chi : “ _________ “
A. Yes, I’d love to. B. I’m just thinking. C. I’m sure about that. D. No, I can’t.
Mark the letter A, B, C, or D on your answer sheet to indicate the correct answer to each of the
following questions.
Question 3: That documentary was _________ film I've seen all year.
A. more interesting B. the most interesting
C. most interesting D. the more and more interesting
Question 4: The school is searching for teachers with extensive experience for the ________ role.
A. teaching B. teacherly C. teachable D. teacher
Question 5: . Eating too much fast food at night, ________ him unhealthy.
A. to keep B. keeping C. kept D. be keeping
Question 6: The children were playing in ________ park at 5pm yesterday.
A. a B. an C. the D. Ø (no article)
Question 7: Before making a decision, it's important to ________ all available options.
A. look into B. hand out C. get by D. bring out
Question 8: The project has ________ by the team, and improvements are being considered.
A.being discussing B. been discussed C. had discussed D. discussed
Question 9: He remembered ________ the stove before leaving the house, ensuring there would be
no accidents while he was away.
A.to turn off B. being turned off C. turned off D. turning off
Question 10: We should leave early to avoid traffic, ________ ?
A. should we B. do not we C. shouldn’t we D. do they
Question 11: The council is considering banning vehicles from the town centre to _______
congestion.
A. abandon B. dispose C. relieve D. discard

137
Question 12: While I was trying to explain to them, they ________ and laughing together.
A. sat B. were sitting C. are sitting D. being sat
Question 13: Parents should set a(an) _______ to their children.
A. example B. fire C. home D. exercise
Question 14: We’ll be able to leave for the airport _______ .
A. as soon as the taxi arrives B.as soon as the taxi had been arrived
C.as soon as the taxi arrived D.as soon as the taxi will arrive
Question 15: We should _______ a reservation at the theater if we want to see the show tomorrow
night.A. take B. get C. play D. make
Question 16: I'm scared _______ telling her what really happened.
A. of B. about C. on D. at
Question 17: My mother’s garden is so beautiful, she has got a green _______ .
A. toe B. thumb C. finger D. hand
Mark the letter A, B, C, or D on your answer sheet to indicate the word whose underlined part
differs from the other three in pronunciation in each of the following questions.
Question 18: A. dream B. wear C. treat D. mean
Question 19: A. science B. comb C. clerk D. craft

Mark the letter A, B, C, or D on your answer sheet to indicate the word that differs from the
other three in the position of stress in each of the following questions.
Question 20: A. machine B. explain C. chemist D. provide
Question 21: A. affectionate B. reasonable C. respectable D. occasional
Mark the letter A, B, C, or D on your answer sheet to indicate the word CLOSEST in meaning to
the underlined word in each of the following questions.
Question 22: The chef was meticulous in following the recipe, measuring each ingredient exactly
to create the perfect food.
A. careless B. serious C. useful D. careful
Question 23: Every time I ask him to help with the household chores, he starts to grumble about
it. A. complain B. compliment C. shout D. agree
Mark the letter A, B, C, or D on your answer sheet to indicate the word(s) OPPOSITE in
meaning to the underlined word(s) in each of the following questions.
Question 24: Her health has always been very fragile.
A. bad B. strong C. pleasant D. unhappy
Question 25: He has found himself in love with two people at once. Now, he's in a pickle to deal
with.A. easy B. difficult C. interested D. relaxed
Read the following passage and mark the letter A, B, C, or D on your answer sheet to indicate
the correct word or phrase that best fits each of the numbered blanks from 26 to 30.
Backpacking offers adventure and freedom as travelers explore (26) _______ parts of the world
with just a backpack. It's about spontaneity, discovering hidden treasures, and connecting with
nature and fellow adventurers. (27) ________ it also requires planning and preparation for a safe
journey. From (28) ________ the right gear and packing essentials to researching destinations
and understanding local customs, successful backpacking trips necessitate a combination of
adventure and responsibility. It's about striking a balance between spontaneity and practicality,
138
(29) ________ allow for flexibility while ensuring safety and comfort along the way. Despite the
challenges, backpacking promises unforgettable experiences and the thrill of (30) ________. So,
pack your essentials and set off on a journey of discovery!
Question 26: A. a lot B. lot of C. many D. much
Question 27: A. Yet B. But C. However D. Because of
Question 28: A. giving B. choosing C. trying D. doing
Question 29: A. which B. that C. where D. what
Question 30: A.imagination B. creativity C. exploration D.inspiration
Read the following passage and mark the letter A, B, C, or D on your answer sheet to indicate
the correct answer to each of the questions from 31 to 35.
Staying up late has become a widespread habit, especially among young people. However, it's
important to understand that staying awake past your usual bedtime can have significant
consequences. It can disrupt your body's sleep pattern, making it difficult to fall asleep when
you want to and leaving you feeling tired during the day. Not getting enough sleep can weaken
your immune system and impair cognitive function. Staying up late often leads to increased
consumption of coffee or energy drinks to stay awake. However, relying on these beverages can
further disrupt sleep and contribute to other health issues. Feeling tired can also affect your
mood and stress levels, impacting your relationships and behavior.
To avoid these negative effects, it's essential to prioritize sleep and establish healthy bedtime
habits. Aim to go to bed at the same time each night and create a relaxing bedtime routine to
signal to your body that it's time to wind down. By ensuring you get enough sleep, you'll not
only feel better physically and mentally but also be better equipped to face the challenges of
daily life.
Question 31: The passage is mainly about_______?
A. the need to establish healthy bedtime habits to mitigate the negative effects of staying up late.
B. the resulting impact on physical and mental health.
C. the relationship between staying up late and chronic sleep deprivation.
D. the negative effects of staying up late and the importance of prioritizing sleep.
Question 32: The word “consequence” is closest in meaning to _______.
A. opinion B. result C. problem D. attitude
Question 33: The word “it” in paragraph 1 refers to ________.
A. sleep pattern B. immune system
C. staying awake past your usual bedtime D. significant consequences
Question 34: According to the passage, Establishing a regular sleep schedule and creating a
relaxing bedtime routine are ________ .
A. essential B. unnecessary C. controlled D. horrible
Question 35: according to the passage, all of the following are true, EXCEPT_______?
A. the need to establish healthy bedtime habits to mitigate the negative effects of staying up late.
B. the resulting impact on physical and mental health.
C. the relationship between staying up late and chronic sleep deprivation.
D. the importance of limiting caffeine consumption to improve sleep quality
Read the following passage and mark the letter A, B, C, or D on your answer sheet to indicate
the correct answer to each of the questions from 36 to 42.

139
In the dynamic landscape of today's economy, freelancing has emerged as a defining mode of
work for many members of Generation Z. Unlike previous generations, who often sought
stability in traditional employment, Gen Z individuals are embracing the flexibility and
autonomy offered by freelancing.
For Gen Z freelancers, the allure lies in the ability to craft their own careers on their terms.
They are leveraging their digital skills and online platforms to offer a wide range of services,
from graphic design and content creation to programming and digital marketing. With access to
a global market and an array of digital tools, Gen Z freelancers are not bound by geographical
limitations and can collaborate with clients and fellow freelancers from around the world.
Moreover, freelancing aligns with the values of independence and work-life balance that are
highly prized by Gen Z. Rather than being tethered to a traditional nine-to-five schedule,
freelancers have the freedom to set their own hours and work from anywhere with an internet
connection. This flexibility allows them to pursue their passions, travel, and maintain a healthy
work-life integration.
However, freelancing also presents its own set of challenges for Gen Z individuals. They must
navigate the complexities of self-employment, including managing finances, marketing
themselves, and handling client relationships. Additionally, the gig economy can be
unpredictable, with fluctuations in income and periods of feast or famine.
Nevertheless, Gen Z freelancers are resourceful and adaptable, leveraging technology and
networking to overcome obstacles and thrive in the freelance world. They are redefining
traditional notions of work and success, proving that you don't need a corner office or a
corporate ladder to build a fulfilling and prosperous career in the digital age.
In conclusion, freelancing has become a cornerstone of the Gen Z work ethic, offering
independence, flexibility, and limitless opportunities for self-expression and growth. As this
generation continues to shape the future of work, the rise of freelancing will undoubtedly remain
a defining characteristic of their professional identity.
Question 36: Which of the following can be the best title for the passage?
A.Embracing Autonomy: Generation Z's Shift Towards Freelancing.
B.Unleashing Potential: Gen Z's Journey in the Freelance Landscape.
C.Digital Mavericks: How Gen Z Redefines Work Through Freelancing.
D.Breaking Boundaries: Gen Z's Path to Independence in Freelancing.
Question 37: Freelancing has emerged as a defining mode of work for many members of
Generation Z, offering _______ .
A. limitations, and a lack of autonomy
B. independence and flexibility
C. dissatisfaction, and minimal opportunities for advancement
D. monotony, and little room for creativity
Question 38: The word that in paragraph 2 refers to _______.
A. freelancing aligns B. independence
C. the values of independence and work-life balance D. gen Z
Question 39: The word navigate in paragraph 3 is closest in meaning to _______.
A. manage B. lose C. request D. block
Question 40: The word prosperous in paragraph 5 is closest in meaning to _______.
A. delighted B. poor C. thriving D. pleased

140
Questio 41: According to the passage, which of the following statements is NOT TRUE ?
A. Freelancing is embraced by Generation Z individuals, who prefer the stability of traditional
employment.
B. Generation Z freelancers are primarily focused on traditional employment and avoid
freelancing due to its lack of stability.
C. Generation Z freelancers face challenges in managing finances, marketing themselves, or
handling client relationships.
D. The gig economy can be unpredictable, with fluctuations in income and periods of feast or
famine.
Question 42: What can be inferred from the passage ?
A. Generation Z freelancers face very few challenges in navigating the freelance world,
including managing finances and client relationships.
B. Gen Z freelancers are bound by geographical limitations and can not collaborate with clients
and fellow freelancers from around the world.
C. This generation continues to shape the future of work, the rise of freelancing will doubtedly
remain a defining characteristic of their professional identity.
D. Generation Z freelancers redefine traditional notions of work and success, proving that
fulfilling careers can be built without conforming to corporate structures.
Mark the letter A, B, C, or D on your answer sheet to indicate the sentence that best combines
each pair of sentences in the following questions.
Question 43: She hugged the pillow all night in frustration. She couldn’t sleep well.
A. If she didn’t hugged the pillow all night in frustration, she could sleep well.
B. If she hadn’t hugged the pillow all night in frustration, she could have slept well.
C. If she had hugged the pillow all night in frustration, she could have slept well.
D. If she hugged the pillow all night in frustration, she couldn’t have slept well.
Question 44: He became a successful businessman. He realized that he needed to stay away from
temptations.
A. Only by he became a successful businessman did he realize the need to stay away from
temptations.
B. Not until he realize that he needed to stay away from temptations did he became a successful
businessman.
C. Not until he became a successful businessman did he realize that he needed to stay away
from temptations.
D. Had he become a successful businessman, he would have realized that he needed to stay
away from temptations.
Mark the letter A, B, C, or D on your answer sheet to indicate the underlined part that needs
correction in each of the following questions.
Question 45: Walking through the bustling city streets, he randomly came across an old friend
they hadn't seen in years.
A. the B. an C. they D. years
Question 46: He has grown up in a privileged and advantageous environment, so he felt a strong
sense of responsibility to use his resources to benefit others in need.
A. privileged B. advantageous C. responsibility D. in need.

141
Question 47:Last month, Mien randomly find her childhood toy while cleaning out the upper floor.
A. randomly B. find C. while D. out
Mark the letter A, B, C, or D on your answer sheet to indicate the sentence that is closest in
meaning to each of the following questions.
Question 48: It is forbidden for people to enter the temple without proper attire and respectful
behavior.
A. People won't enter the temple without proper attire and respectful behavior.
B. People could enter the temple without proper attire and respectful behavior.
C. People mustn’t enter the temple without proper attire and respectful behavior.
D. People are able to enter the temple without proper attire and respectful behavior.
Question 49: The last time I saw my grandmother was at my graduation ceremony.
A. I last saw my grandmother since my graduation ceremony.
B. I haven’t seen my grandmother since my graduation ceremony.
C. I didn’t see my grandmother since my graduation ceremony.
D. I have seen my grandmother was at my graduation ceremony.
Question 50: “I'm going to the store to buy some groceries”, she said.
A. She said that she was going to the store to buy some groceries.
B. She said that she had been going to the store to buy some groceries .
C. She said that she is going to the store to buy some groceries.
D. She said that she has been going to the store to buy some groceries.

ĐỀ 4
Mark the letter A, B, C, or D to indicate the sentence that best combines each pair of sentences
in the following questions.
Question 1. We ran out of money. We came home from holiday early.
A. If we hadn’t run out of money, we would come home from holiday early.
B. If we hadn’t run out of money, we wouldn’t have come home from holiday early.
C. If we hadn’t run out of money, we wouldn’t come home from holiday early.
D. If we didn’t run out of money, we wouldn’t come home from holiday early.
Question 2. Phong had just finished the hotel management course. He started looking for a job.
A. No sooner had Phong finished the hotel management course than he started looking for a job.
B. Not until Phong started looking for a job did he finish the hotel management course.
C. Had it been for Phong’s completion of hotel management course, he would have started
looking for a job.
D. Only after Phong started looking for a job did he finish the hotel management course.
Mark the letter A, B, C, or D on your answer sheet to indicate the correct answer to each of
the following questions.
Question 3. The conference brought together experts from all over ____ world to share ideas and
propel advancements in their fields.
A. a B. Ø C. an D. the
Question 4. Recent research has ____ the idea that water is only to be found on planet Earth;
evidence of it has been found on Mars.

142
A. approved B. disproved C. improved D. reproved
Question 5. Tickets for “Lat Mat 7” _______ in the upcoming weeks.
A. will be selling B. sells C. will sell D. will be sold
Question 6. This sort of job requires good _____ and teamwork skills.
A. communicator B. communicative C. communicate D. communication
Question 7. Jack is not interested in going to university, _______?
A. does he B. doesn’t he C. is he D. isn’t he
Question 8. It’s important ___ us to carry out a campaign against violence in our school.
A. of B. to C. in D. for
Question 9. You can imagine how ______ I felt when I realised I was wearing two different shoes.
A. embarrassed B. shy C. ashamed D. timid
Question 10. The students ______ grammar every day will improve their writing.
A. to practice B. practicing C. was practicing D. practiced
Question 11. If you would like to ______ a complaint, please fill in a form or speak to the
manager.
A. make B. do C. pay D. take
Question 12. Eric ________ in the park when a strange man came up to him.
A. sat B. was sitting C. is sitting D. had sat
Question 13. Six people were injured when a bomb _____ near an army camp last night.
A. took off B. turned up C. went off D. put out
Question 14. Bill won’t leave the office ______________.
A. until he finishes writing his report
B. when he finished writing his report
C. as soon as he had finished writing his report
D. by the time he finished writing his report
Question 15. I was on the ___ of buying the cookbook when I realized that I had it already.
A. verge B. edge C. outset D. cusp
Question 16. Please don’t forget _____ the bedroom door when you go out as the cat keeps going
in there.
A. closing B. to closing C. to close D. close
Question 17. I think Geoff is __________ in the class at chess.
A. worse B. more bad C. the worst D. the worse
Mark the letter A, B, C or D on your answer sheet to indicate the word(s) CLOSEST in
meaning to the underlined word(s) in each of the following questions.
Question 18. When Eileen saw that her mother had bought the dress she wanted for herself, she
was tickled pink.
A. depressed B. surprised C. proud D. pleased
Question 19. This necklace isn’t real gold - I’m sorry to have to tell you that it’s completely
worthless.
A. valueless B. priceless C. invaluable D. worthy
Mark the letter A, B, C, or D on your answer sheet to indicate the word that differs from the
other three in the position of the primary stress in each of the following questions.
143
Question 20. A. allow B. open C. appear D. include
Question 21. A. exciting B. terrific C. attractive D. qualified
Mark the letter A, B, C, or D to indicate the sentence that is closest in meaning to each of the
following questions.
Question 22. I last took my child to the amusement park 2 months ago.
A. I haven’t taken my child to the amusement park for 2 months.
B. I haven’t taken my child to the amusement park since 2 months.
C. I didn’t take my child to the amusement park for 2 months.
D. I took my child to the amusement park 2 months ago.
Question 23. It’s not necessary for you to fill in this part of the form.
A. You mustn’t fill in this part of the form.
B. You shouldn’t fill in this part of the form.
C. You needn’t fill in this part of the form.
D. You have to fill in this part of the form.
Question 24. Rob said: “I will have an important job interview tomorrow.”
A. Rob said he would have an important job interview tomorrow.
B. Rob said he will have an important job interview tomorrow.
C. Rob said he will have an important job interview the following day.
D. Rob said he would have an important job interview the following day.
Read the following passage and mark the letter A, B, C, or D to indicate the correct word or
phrase that best fits each the numbered blanks (from 25 to 29).
Reefs form under water and can be made of coral, sand or rock. The top of the reef sits about two
or three feet below sea level. Coral reefs are found in warm, shallow, clean water and the Great
Barrier Reef is the largest coral reef of all. It (25) _____ for two thousand kilometres and forms a
natural breakwater that protects the coast of Australia from the strong waves of the South Pacific.
There are (26) ___ islands on the Great Barrier Reef formed by piled up coral and sand. They are,
(27) ________, very difficult to reach as sharp coral will tear the bottom out of a boat. One must be
careful not to step on living coral as it grows very slowly, about five centimetres a year. The Great
Barrier Reef is (28) ______ to many types of fish. There are also other aquatic species (29)
_______ depend on coral to protect them.
Question 25. A. lies B. stretches C. locates D. lasts
Question 26. A. many B. much C. little D. another
Question 27. A. however B. therefore C. so D. but
Question 28. A. site B. place C. home D. house
Question 29. A. what B. that C. where D. who
Read the following passage and mark the letter A, B, C, or D on your answer sheet to indicate
the correct answer to each of the questions from 30 to 36.
When it comes to education options for school-leavers, there are several paths to consider.
One common option is pursuing higher education at a university or college. This path typically
involves obtaining a bachelor’s degree in a chosen field of study, which can provide a solid
foundation for various career opportunities. Universities and colleges offer a wide range of
academic programs, allowing school-leavers to explore their interests and specialize in areas that
align with their career aspirations.

144
Another option for school-leavers is vocational education and training (VET) programs. VET
emphasizes providing practical skills and knowledge that directly apply to specific industries or
trades. These programs offer hands-on training, preparing individuals for careers in fields such as
healthcare, information technology, construction, hospitality, and automotive industries. VET
programs often include apprenticeships or internships, allowing students to gain valuable work
experience while studying.
For those who prefer a more experiential approach, apprenticeships can be a valuable
education option. Apprenticeships combine on-the-job training with classroom instruction,
allowing school-leavers to earn while they learn. They provide a unique opportunity to gain
practical skills and industry-specific knowledge directly from professionals in the field.
Apprenticeships are available in a variety of industries, such as construction, culinary arts,
electrical work, and healthcare.
Ultimately, the choice of education option for school-leavers depends on individual interests,
goals, and learning preferences. Whether it’s pursuing higher education at a university, enrolling in
a vocational program, or embarking on an apprenticeship, each path offers distinct advantages and
can lead to rewarding career paths. It's important for school-leavers to explore and evaluate these
options carefully to make an informed decision that aligns with their aspirations and sets them on a
path to success. (Adapted from Education Week)
Question 30. Which of the following can be the best title for the passage?
A. Making Informed Decisions: How to Choose the Right Education Path for School-Leavers
B. Mapping Future Careers: Exploring Opportunities for School-Leavers
C. Exploring Various Education Options for School-Leavers
D. Navigating Pathways after School: Education Options for Postgraduate
Question 31. According to the passage, the common path after leaving school for students is ____.
A. VET programs B. internship C. tertiary education D. apprenticeship
Question 32. The word "align with” in paragraph 1 is closest in meaning to ________.
A. match with B. deal with C. come up with D. go with
Question 33. The word "emphasize" in paragraph 2 is closest in meaning to ________.
A. highlight B. italicize C. understate D. strengthen
Question 34. The word "them" in the last paragraph refers to ________ .
A. school-leavers B. aspirations C. options D. decisions
Question 35. Which of the following is NOT true according to the passage?
A. Pursuing higher education at a university or college typically involves obtaining a bachelor’s
degree.
B. Vocational education and training (VET) programs focus on providing practical skills and
knowledge for specific industries or trades.
C. School-leavers have the opportunity to specialize in areas aligned with their career
aspirations through university or college education.
D. Apprenticeships do not offer any classroom instruction and are solely based on on-the-job
training.
Question 36. It can be inferred from the passage that ________.
A. Vocational education and training (VET) programs are more beneficial for school-leavers
than pursuing a university degree.

145
B. Considering individual interests and career aspirations when choosing an education option for
school-leavers is important.
C. School-leavers should prioritize earning potential when choosing an education option.
D. Apprenticeships provide a more practical learning experience compared to higher education.
Read the following passage and mark the letter A, B, C, or D on your answer sheet to indicate
the correct answer to each of the questions from 37 to 41.
Independence and personal choice are considered as a high value on Australians place.
Therefore, rather than telling students exactly what to do, teachers and course tutors give them a
few different options and suggest they pick the one that works best for them. They also expect
students to take charge if things don't go as planned and to find help and support on their own.
Australians are also prepared to accept a range of opinions rather than believing there is one
truth. This means that in an educational setting, students will be expected to form their own
opinions and defend the reasons for that point of view and the evidence for it.
Australians are uncomfortable with disparity in social status and therefore admire the
concept of treating everyone equally. An example of this is how the majority of Australian
adults address each other by their first names. This emphasis on equality results in Australians
feeling uneasy about taking things too seriously and being willing to make jokes about
themselves.
Australians believe that life should have a balance between work and leisure time. As a
consequence, some students may be critical of others who they perceive as doing nothing but
study. Australian notions of privacy mean that areas such as financial matters, appearance and
relationships are only discussed with close friends. While people may volunteer such
information, they may resent someone actually asking them unless the friendship is firmly
established. Even then, it is considered very impolite to ask someone what they earn.
(Source: Complete IELTS Student Book by Guy Brook Hart and Vanessa Jakeman)
Question 37. The word they in paragraph 1 refers to .
A. teachers and course tutors B. options C. Australians D. students
Question 38. The word disparity in paragraph 3 is closest in meaning to .
A. difference B. independence C. similarity D. equality
Question 39. In paragraph 4, why some students are criticize of others who do nothing but study?
A. they think that life should have a balance between work and leisure time.
B. they want some more privacy.
C. they don’t want others to study better than them.
D. they prefer equality in education.
Question 40. Which of the following is TRUE, according to the passage?
A. Students in Australia will be not expected to form their own opinions.
B. Asking someone what they earn is considered fairly discourteous.
C. A teacher or course tutor will not tell students what to do.
D. Australians usually don’t accept many other opinions.
Question 41. Which best serves as the title for the passage?
A. Education in Australia B. Balancing Work and Leisure in Australia
C. Australian culture D. Australians and their privacy

146
Mark the letter A, B, C or D to indicate the underlined part that needs correction in each of the
following questions.
Question 42. When I lived in this city, I usually shop for groceries at this supermarket.
A B C D
Question 43. We provide the best seats for our attendees, guaranteed to showcase minimum
discomfort. A B C D
Question 44. The dog loves to play fetch in the park and goes for a walk with their owner.
A B C D
Mark the letter A, B, C or D on your answer sheet to indicate the word(s) OPPOSITE in
meaning to the underlined word(s) in each of the following questions.
Question 45. Almost all the pupils at this school come from privileged backgrounds as only the
most affluent parents can afford to pay the huge fees.
A. wealthy B. well off C. disabled D. penniless
Question 46. She was born in an impoverished family. It was a life full of pain and suffering.
A. distress B. misfortune C. discomfort D. pleasure
Mark the letter A, B, C, or D on your answer sheet to indicate the word whose underlined
part differs from the other three in pronunciation in each of the following questions.
Question 47. A. future B. intend C. table D. teacher
Question 48. A. come B. comic C. money D. enough
Mark the letter A, B, C, or D on your answer sheet to indicate the most suitable response to
complete each of the following exchanges.
Question 49. Charlotte and Dave are talking about online learning.
- Charlotte: “I think online learning won’t be effective if students have problems with Internet.”
- Dave: “_________________________”
A. You can say that again. B. No, it’s not true.
C. You’re wrong. D. Really?
Question 50. Ben and Hien are in the school canteen.
- Ben: "_____________________?” - Hien: "No, thanks. I’m not thirsty."
A. Do you want to eat anything B. Do you like orange juice
C. What drink do you like D. Would you like something to drink

ĐỀ 5
Mark the letter A, B, C, or D to indicate the sentence that best completes each of the following
exchanges.
Question 1. David is talking to Linda after a party.
- David: “Would you like me to give you a ride home?”- Linda: “________”
A. That’d be great, thanks. B. Sorry, you’re not my type.
C. Yes, I’m riding home now. D. No, thanks. I don’t like riding
Question 2. Ann and John is talking about social network.
- Ann: "Today social network plays an important role in communication.”- John: “________”
A. It's rubbish B. I'm afraid I don't know
C. I don't agree with you D. You can say that again
147
Mark the letter A, B, C or D to indicate the correct answer to each of the following questions.
Question 3. What is_______ month of the year in England?
A. weter B. the wetter C. wettest D. the wettest
Question 4. To many children, playing computer games is a form of _______.
A. relax B. relaxingly C. relaxation D. relaxed
Question 5. They skipped breakfast, _______ them feeling tired and unfocused.
A. to leave B. left C. leaving D. is left
Question 6. Air pollution is getting more and more serious in _______ big cities such as Hanoi and
Beijing. A. the B. a C. an D. Ø (no article)
Question 7. She had been depressed all day but she started to _______ after she heard that she was
promoted. A. turn up B. cheer up C. take up D. break up
Question 8. The trees _______ more and more in our countryside next year.
A. will plant B. will be planted C. are planted D. plant
Question 9. In the sustainable agriculture, farmers try _______ the use of chemicals and
fertilizers.
A. to limit B. limiting C. limited D. limit
Question 10. His parents don't approve of what he does, _______?
A. do they B. don’t they C. did they D. didn’t they
Question 11. Yesterday the naval authorities _______ the reports in Friday’s newspapers that they
had exploded three bombs near an unknown submarine.
A. published B. confirmed C. re-stated D. agreed
Question 12. Yesterday I _______ in the park when I saw Dick playing football.
A. was walking B. is walking C. has walked D. has been walking
Question 13. An important rule in Tim's family is that they have to pay _______ visits to their
grandparents in the countryside.
A. customary B. normal C. usual D. Regular
Question 14 The training seminar will begin _____________
A. when all the staff were arriving B. as soon as all the staff arrive
C. by the time all the staff arrived D. after all the staff had arrived
Question 15 We're planning to _______ an offer on a beautiful house which is for sale near the
town centre.A. make B. break C. turn D. catch
Question 16. Police said the thieves were obviously well acquainted _______ the alarm system at
the department store.
A. with B. to C. of D. in
Question 17. It’s a good idea in theory, but it’s going to be hard to put in into ________.
A. practice B. trial C. test D. Examination
Mark the letter A, B, C, or D to indicate the word whose underlined part differs from the other
three in pronunciation in each of the following questions.
Question 19. A. method B. healthy C. weather D. thirst
Question 18. A. connect B. comment C. request D. message
Mark the letter A, B, C, or D to indicate the word that differs from the other three in the position
of the primary stress in each of the following questions.

148
Question 20. A. easy B. instant C. tidy D. mature
Question 21. A. sightseeingB. heritage C. snorkling D. solution
Mark the letter A, B, C or D to indicate the word(s) CLOSEST in meaning to the underlined
word(s) in each of the following questions.
Question 22. According to the officials of the MOET, reopening schools is inevitable since the
vaccine rate is high now.
A. unlikely B. doubtful C. unavoidable D. dispensable
Question 23. That matter is so confidential that it must not be discussed outside the office of chief
inspector.
A. secret B. fascinating C. important D. alarming
Mark the letter A, B, C or D to indicate the word(s) OPPOSITE in meaning to the underlined
word(s) in each of the following questions.
Question 24. The industrial revolution caused a complete upheaval of the textile industry, with
manual labor being replaced by mechanization and mass production.
A. stability B. catastrophe C. establishment D. disturbance
Question 25. The teacher will see red when he finds out you haven't done your homework again!
A. be maddened B. annoy C. make things difficult D. be calm
Read the following passage and mark the letter A, B, C, or D to indicate the correct word or
phrase that best fits each the numbered blanks.
ChatGPT: New AI chatbot has everyone talking to it
A new chatbot has passed one million users in less than a week, the project behind it says.ChatGPT
was publicly released on Wednesday by OpenAI, an artificial intelligence research firm (26)_____
founders included Elon Musk.ChatGPT is the latest in a series of AIs which the firm refers to as
GPTs, an acronym which (27)_____ for Generative Pre-Trained Transformer.To develop the
system, an early version was fine-tuned through conversations with human trainers.The system also
learned from access to Twitter data according to a tweet from Elon Musk who is no longer part of
OpenAI's board. The Twitter boss wrote that he had paused access "for now".The results have
impressed (28)_____ who've tried out the chatbot. OpenAI chief executive Sam Altman revealed
the level of interest in the artificial conversationalist in a tweet.The project says the chat format
allows the AI to answer "follow-up questions, admit its mistakes, challenge incorrect premises and
reject inappropriate requests"A journalist for technology news site Mashable who tried out
ChatGPT reported it is hard to provoke the model into saying offensive things.(29)_____, OpenAI
warns that "ChatGPT sometimes writes plausible-sounding but incorrect or nonsensical
answers".Training the model to be more cautious, says the firm, causes it to decline to answer
questions that it can answer correctly.Briefly questioned by the BBC for this article, ChatGPT
revealed itself to be a cautious interviewee (30)_____ of expressing itself clearly and accurately in
English. https://www.bbc.com/news/technology

Question 26. A. where B. which C. who D. whose


Question 27. A. care B. take C. stand D. look
Question 28. A. little B. few C. much D. many
Question 29. A.As a result B. Therefore C. However D.So
Question 30. A. afraid B. fond C. able D. capable

149
Read the following passage and mark the letter A, B, C, or D to indicate the answer to each of
the question.
Bali, the fabled "Island of the Gods", has been enchanting visitors for centuries with its rich
cultural traditions and spectacular panoramas. From lofty, mist enshrouded volcanoes and cool
mountain lakes down through terraced rice fields to a golden strand lapped by azure waters, every
square inch of Bali offers a fresh and unforgettable image.
No less enchanting are its people, some 2.7 million souls whose artistry and piety are
recognized throughout the world Balinese Hinduism, a complex fusion of Indian cosmology.
Tantric Buddhism and homegrown mythology, is the primary faith of Bali's inhabitants, and so
deeply woven into the fabric of their daily lives that the line between the spiritual and the material
is blurry at best.
Those of you keen on delving into the island's fascinating culture will have plenty of
opportunities, as colorful ceremonies and traditional performances occur with regularity of sunrise.
Most hotels offer nightly dance shows of one form or another, tailored to tourist audiences but
none the less exquisite. The hill town of Ubud, the island's premier arts center, also has a full
schedule of performance, and the nearby stone-cutter's village of Batubulan is famed for its Barong
lion dances. The shoppers among you will find Bali a treasure house of handicrafts and fine works
of art. The Balinese are incredibly gifted artists and craftsmen, and their material creations are
imbued with the same sense of wonderment with which they regard their universe. Stone and wood
carvings, traditional and modern paintings and intricately designed jewelry in gold and silver are
readily in shops and galleries throughout the island.
As for recreation, there is no shortage of option. Nature walks, horseback riding, diving,
surfing, even bungy jumping, and white water rafting await the adventurous here.
(Source: https://books.google.com.vn/books)
Question 31. The topic of the given passage is _______.
A. Balinese life B. Scenery in Bali C. Tourism in Bali D. Bali for recreation
Question 32. The second paragraph of the passage mainly discusses _______.
A. Population of Bali B. artistry and piety of Bali
C. Balinese religion D. daily lives of Balinese
Question 33. The word "their" in the second paragraph refers to _______.
A. 2.7 million souls B. Bali's inhabitants
C. daily lives D. the spiritual and the material
Question 34. Which of the following might be a synonym of the word "exquisite" in the third
paragraph?
A. skillful B. clever C. spiritual D. material
Question 35. You can find all these recreational activities in Bali EXCEPT _______.
A. horseback riding B. nature walks C. water rafting D. parachute jumping
Read the following passage and mark the letter A, B, C, or D to indicate the answer to each of
the question.
Advocates of the laws and journalists who cover the issue often neglect to ask what will
replace plastic bags and what the environmental impact of that replacement will be. People still
need bags to bring home their groceries. And the most common substitute, paper bags, may be just
as bad or worse, depending on the environmental problem you are most concerned about.

150
That is leading to a split in the anti-bag movement. Some bills, like in Massachusetts, try to
reduce the use of paper bags as well as plastic, but still favour paper. Others, like in New York
City, treat all single-use bags equally. Even then, the question remains as to whether single-use
bags are necessarily always worse than reusable ones.
Studies of bags’ environmental impacts over their life cycle have reached widely varying
conclusions. Some are funded by plastic industry groups, like the ironically named American
Progressive Bag Alliance. Even studies conducted with the puret of intentions depend on any
number of assumptions. How many plastic bags are replaced by one cotton tote bag? If a plastic
bag is reused in the home as the garbage bag in a bathroom waste bin, does that reduce its footprint
by eliminating the need for another small plastic garbage bag?
If your chief concern is climate change, things get even muddier. One of the most
comprehensive research papers on the environmental impact of bags, published in 2007 by an
Australian state government agency, found that paper bags have a higher carbon footprint than
plastic. That is primarily because more energy is required to produce and transport paper bags.
“People look at paper and say it’s degradable, therefore it’s much better for the environment,
but it’s not in terms of climate change impact,” says David Tyler, a professor of chemistry at the
University of Oregon who has examined the research on the environmental impact of bag use. The
reasons for paper’s higher carbon footprint are complex but can mostly be understood as stemming
from the fact that paper bags are much thicker than plastic bags. “Very broadly, carbon footprints
are proportional to mass of an object,” says Tyler. For example, because paper bags take up so
much more space, more trucks are needed to ship paper bags to a store than to ship plastic bags.
(Adapted from https://www.wired.com/)
Question 36. Which of the following best serves as the title for the article?
A. Paper bags are a good substitute for plastic ones.
B. Is plastic really worse than paper?
C. Activities of American Progressive Bag Alliance
D. What people think about paper bags
Question 37. The word “their” in paragraph 1 refers to _______.
A. people B. groceries C. paper bags D. plastic bags
Question 38. American Progressive Bag Alliance is the name of a _______.
A. plastic bag seller B. paper bag company
C. law company D. plastic industry group
Question 39. The word “muddier” in paragraph 4 most probably means _______.
A. uglier B. luckier C. rarer D. vaguer
Question 40. Which of the following is TRUE, according to the article?
A. Paper bags can be the substitute for plastic bags.
B. New York City prefers single-use bags.
C. Studies reach the same conclusion on bags’ environmental impacts.
D. Plastic bags have a higher carbon footprint than paper according to research in 2007.
Question 41. The word “stemming” in paragraph 5 is closest in meaning to _______.
A. looking for B. asking for C. deriving from D. keeping from
Question 42. What can be inferred the reason for paper’s higher carbon footprint?
A. Paper bags are difficult to ship. B. Paper bags are much thicker than plastic bags.

151
C. Paper bags take less space. D. Paper bags are easily torn.
Mark the letter A, B, C, or D to indicate the sentence that best combines each pair of sentences
in the following questions.
Question 43. He is short-sighted. Therefore, he has to wear the glasses.
A. If only he were short-sighted, he wouldn’t have to wear the glasses.
B. If he hadn’t been short-sighted, he wouldn’t have had to wear the glasses,
C. If he weren’t short-sighted, he wouldn’t have to wear the glasses.
Question 44. The old man confessed to me. I knew that he had killed one of my dogs.
A. No sooner had the old man confessed to me when I knew that he had killed one of my dogs.
B. Only after the old man confessed to me did I know that he had killed one of my dogs.
C. Only when did the old man confess to me I knew that he had killed one of my dogs.
D. Not until the old man confessed to me did I knew that he had killed one of my dogs.
Mark the letter A, B, C or D to indicate the underlined part that needs correction in each of the
following questions.
Question 45. It is essential to provide high school students with adequate career orientation, so
they can make more informative decisions about their future major.
A. essential B. orientation C. informative D. major
Question 46. Neither the Minister nor his colleagues has given an explanation for the chaos in the
financial market last week.
A. Neither B. has given C. for D. financial
Question 47. A turtle differs from all other reptiles in that its body is encased in a protective shell
of their own. A. from B. other C. encased D. their
Mark the letter A, B, C, or D to indicate the sentence that is closest in meaning to each of the
following questions.
Question 48. It is possible that Nancy will come to the fashion show.
A. Nancy may come to the fashion show.
B. Nancy should come to the fashion show.
C. Nancy needn’t come to the fashion show.
D. Nancy mustn’t come to the fashion show.
Question 49. She started driving to work one month ago.
A. She has driven to work since one month.
B. It is one month when she started driving to work.
C. She has driven to work for one month.
D. It was one month after she started driving to work.
Question 50. Tom said: “Why do you keep staring at me, Janet?”
A. Tom asked Janet why did she keep staring at him.
B. Tom asked Janet why she keeps staring at him.
C. Tom asked Janet why she had kept staring at him.
D. Tom asked Janet why she kept staring at him.
ĐỀ 6
Mark the letter A, B, C, or D on your answer sheet to indicate the word whose underlined part
differs from the other three in pronunciation in each of the following questions.

152
1. A. knowledge B. agree C. generous D. surgeon
2. A. equal B. female C. enroll D. being
Mark the letter A , B , C , or D on your answer sheet to indicate the word that differs from the
other three in the position of primary stress in each of the following questions.
Question 3: A. purpose B. compose C. propose D. suppose
Question 4: A. entertain B. engineer C. committee D. referee
Mark the letter A, B, C, or D on your answer sheet to indicate the correct answer to each of the
following questions.
Question 5: The woman English at his school is Ms. Smith.
A. who teach B. teaching C. taught D. was being taught
Question 6: Lan is student in her class.
A. the most tall B. the tallest C. the more tall D. the taller
Question 7: The passengers to board the game when it started to
rain.
A. were waiting B. was waiting C. waited D. would waiting
Question 8: Durian is now ____ harvest season in Mekong Delta provinces.
A. in B. at C. under D. on
Question 9: Director Pham Thien An felt confused when he first studied films, ______?
A. did he B. didn’t he C. had he D. hadn’t he
Question 10: Nobody died in the terrorism, but 20 people were ____________
A. damaged B. injured C. spoiled D. wounded
Question 12: The online game “Dumb ways to die” quickly_____________with young people
after being released in 2013.
A. took on B. caught up C. caught on D. took up
Question 13: It was his own fault, but I couldn’t help ________ sorry for him.
A. feeling B. to feel C. felt D. to have felt
Question 14: He saw them watching him and bared his ___________ in the manner of an angry
dog.
A. mouth B. tooth C. teeth D. nose
Question 15 Question 15: Marry will join us .
A. as soon as she had finished her homework
B. as soon as she will finish her homework
C. as soon as she finished her homework
D. as soon as she finishes her homework
Question 16: Amazon acquired Ring in 2018, ______ the way for the e-commerce giant to get into
the home security business.
A. getting B. putting C. paving D. showing
Question 17: Over 700 durian trucks __________ at the Vietnam-China border last year.
A. are sticking B. were being stuck C. were stuck D. are being stuck
.Question 18: What __________ views do Americans and Asians have about love and marriage?
A. tradition B. traditionally C. traditionalism D. traditional

153
Question 19: His poor standard of play fully justifies his __________from the team for the next
match. A. expulsion B. dismissal C. rejection D. exclusion
Mark the letter A, B, C, or D on your answer sheet to indicate the sentence that best completes
each of the following exchanges.
Question 20: Jane: “Could you show me the way to the station?” - Jack: “___________”
A. About ten minutes. B. The show is not interesting.
C. The station is far from here. D. Go ahead, then turn left.
Question 21: Peter: “The air quality in Hanoi is getting worse and worse”.
Jenny:” ____________. I can’t see anything in the morning because of too much smoke.”
A. I don’t really think so. B. I don’t quite agree.
C. You can say that again. D. That’s not a matter.
Mark the letter A , B , C , or D on your answer sheet to indicate the word (s) OPPOSITE in
meaning to the underlined word ( s ) in each of the following questions.
Question 22: Authorities in Bali will distribute a list of “dos and don’ts” for visitors arriving at
the tourist island.
A. collect B. assign C. circulate D. delete
Question 23: I’m sure it won’t rain, but I’ll take an umbrella just to be on the safe side.
A. careful B. easy C. careless D. difficult
Mark the letter A, B, C, or D on your answer sheet to indicate the word CLOSEST in meaning to
the underlined word in each of the following questions.
Question 24: The charming old buildings and cottages are a throwback to the colonial past.
A. cunning B. exciting C. interesting D. fascinating
Question 25: Greater gender equality brings freedom, boosts prosperity and strengthens global
security. A. wealth B. disadvantage C. stagnation D. safety
Mark the letter A, B, C, or D on your answer sheet to indicate the sentence that is closest in
meaning to each of the following questions.
Question 26. I haven’t heard from him for ages.
A. I didn’t write to him for ages. B. I last wrote to him long time ago.
C. I wrote to him for ages. D. I haven’t written to him long time ago.
Question 27: ‘‘ Where did you go last night?” she said to her boyfriend.
A. She asked her boyfriend where did he go last night.
B. She asked her boyfriend where he went the night before.
C. She asked her boyfriend where had he gone the night before.
D. She asked her boyfriend where he had gone the night before.
Question 28: Jane still wants to go to the supermarket although the fridge is full of fruits.
A. There is no need for Jane to go to the supermarket.
B. Jane should go to the supermarket.
C. Jane hadn’t better go to the supermarket.
D. Jane mustn’t go to the supermarket.
Mark the letter A , B , C , or D on your answer sheet to indicate the sentence that best combines
each pair of sentences in the following questions.
Question 29: We couldn’t see the mountains because of the rain.

154
A. If only it didn’t rain so we could see the mountains.
B. If it hadn’t rained, we could see the mountains.
C. I wish I had been able to see the mountains without rain.
D. But for the rain, I could see the mountains.
Question 30: When we crossed the fields, we saw a tiny house at the foot of the mountain.
A. No sooner had we crossed the fields when we saw a tiny house at the foot of the
mountain.
B. Not until we crossed the fields did we see a tiny house at the foot of the mountain.
C. Scarcely had we crossed the fields than we saw a tiny house at the foot of the mountain.
D. It was not until we crossed the fields did we see a tiny house at the foot of the mountain.
Mark the letter A, B, C, or D on your answer sheet to indicate the correct answer to each of the
following questions.
Question 31: He was arrested at the airport with a kilo of heroine secreted in his clothing.
A. arrested B. with C. heroine D. clothing
.Question 32: The national soccer team make every effort to win the match yesterday.
A. make B. effort C. to win D. yesterday
Question 33: Animals like frogs have waterproof skin that prevents it from drying out quickly in
air, sun, or wind.A. have B. that C. it D. wind

Read the following passage and mark the letter A, B, C, or D on your answer sheet to indicate
the correct word or phrase that best fits each of the numbered blanks.
There are many types of English around the world. Some well-known varieties in Asia include
Chinglish in China, Singlish in Singapore and Japanese English. A group of language experts in
Japan is troubled by how the government uses English. ___(34)_____, it says the government uses
computer or online translation too much. Researchers say many translations create strange and
confusing words and expressions _(35)_______are confusing to English speakers. The researchers
worry this could have a negative impact on Japan’s tourist industry. They even say the increasing
number of unsuitable words is becoming a “national embarrassment” in Japan.
The research team says computer software gives ____(36)_______ or incorrect translations for
individual kanji - the Chinese characters used in Japanese writing. There are____(37)________
examples of this, including “Hello Work” - the name for job centers, and “Go To Travel” a plan to
help tourism in Japan during the COVID-19 pandemic. The team says software creates,
“unintentionally funny translations that could easily be corrected if they were just checked by an
English speaker”. Businesses also create this English. The Christmas message being used by the
Seibu Sogo department store has raised ____(38)____. It says “Stay Positive.” Many people
believe this is the wrong thing to say during coronavirus and “Stay Happy” would be better.
Question 34: A. Consequently B. While C. In particular D. Generally
Question 35: A. whose B. who C. which D. whom
Question 36: A. foul B. odd C. flashy D. precise
Question 37: A. many B. much C. a lot D. plenty
Question 38: A. hands B. fingers C. voices D. eyebrows
Read the following passage and mark the letter A, B, C, or D on your answer sheet to indicate
the correct answer to each of the questions.

155
The Chinese government has removed pangolin scales from its 2020 list of approved ingredients
used in traditional Chinese medicine, a move campaigners describe as a “critical step” towards
saving the world’s most trafficked mammal.
Pangolins are highly valued in Asia for scales, which many people believe can improve blood
circulation and reduce inflammation. Last year alone, authorities confiscated more than 130 tons
of illegal pangolin related products. There are eight species of pangolin found in Asia and Africa.
To date, three species are listed as critically endangered by the International Union for
Conservation of Nature, while the remaining five are listed as either vulnerable or endangered.
According to Chinese media, the latest edition of Chinese Pharmacopoeia - an official government
list of drugs covering approved traditional Chinese and Western medicines - no longer includes
pangolin scales. The decision to remove the pangolin from the official list comes just days after
China’s State Forestry and Grassland Bureau announced that the Chinese pangolin would be
upgraded to a “first-level protected wild animal,” the highest possible protection status alongside
pandas and tigers.
While conservationists from China and other countries have praised the actions of Chinese
authorities, some still felt that they came a bit too late. “Many years have passed. How many
pangolins have already been hunted and killed?” said Sophia Zhang, director of the Pangolin
Working Group at the China Biodiversity Conservation and Green Development Foundation.
(Adapted from edition.cnn.com)
Question 39: What is the best title for this passage?
A. What We Can All Do To Protect Pangolins
B. Why Are Chinese Pangolins Under Threat?
C. Pangolin Scales: How Valuable Are They?
D. A Big Step In The Protection Of Pangolins
Question 40: The word “confiscated” in paragraph 2 is closest in meaning to ___________.
A. robbed B. detained C. arrested D. seized
Question 40: How have conservationists in China and in other countries react to the Chinese
government’s actions?
A. They are pleased but some felt that actions should have been taken sooner.
B. They believe these actions are useless, given the number of pangolins killed.
C. They doubt whether these actions will prevent people from hunting pangolins.
D. They are all glad that Chinese authorities have taken measures at the right time.
Question 42: Which of the following is NOT TRUE, according to the passage?
A. It is widely believed that pangolin scales have medicinal powers.
B. The Chinese government has a list of approved medicines.
C. There are five species of pangolins that are not endangered.
D. Pandas and tigers are first-level protected wild animals in China.
Question 43: The word “they” in paragraph 4 refers to __________.
A. conservationists B. countries C. actions D. authorities
Read the following passage and mark the letter A, B, C, or D on your answer sheet to indicate
the correct answer to each of the questions.
Despite the easing of lockdowns across the world, many businesses continue to operate largely
online. However, this new business model has exposed a problem: cyber attackers are
disproportionately targeting people of colour and ethnic minorities during this pandemic.
Rashad Robinson, the executive director of Color of Change, the United States’ largest online
racial justice organisation, which has lobbied social media companies to address civil rights
violations on their platforms through many public campaigns, says the Black, Asian and minority

156
ethnic (BAME) community faces “immediate danger” when working on the video call platform
Zoom, where hackers are still actively searching for loopholes to specifically target BAME
communities online.
Robinson explained that these are not random pranks by individuals, but a coordinated effort.
“Color of Change have been contacted by at least 15 groups who have had their gatherings
hijacked,” he added. For instance, a virtual casting call for black actors, a meeting between
minority students and lecturers to discuss racism in universities, and a series of online talks about
the aftermath of George Floyd’s death were targeted on Zoom. Another Zoom event with speakers
from White Coats for Black Lives, an organisation of BAME medical students tackling racism
within the US’s healthcare system, was hijacked.
Lydia Amoah, a diversity specialist, was a target of a racially-motivated cyber-attack during a
webinar with 100 of her colleagues, clients and industry experts in April. “I felt sick for five days
after the hackers attacked. I felt so unsafe online, where I have no choice but to work [...] Even
now I feel hot [and] defenseless, they could see me but I couldn’t see them.” Amoah is now
advising businesses and other companies on how to deal with racism in the virtual workspace. Her
new policy, the STOPIT Protocol, offers businesses a step-by-step guide on how to protect
workers online, and has already been implemented by some UK charities and organisations.
Color of Change has been in contact with Zoom and has asked the company to release a specific
plan to combat racial harassment on the platform and apologise to targets of Zoom bombings, but
Lydia believes responsibility also lies with employers who have not taken measures to protect
minorities in the virtual workspace. “The first step is sharing our stories, the next is for companies
to start implementing real and lasting policies that will protect all of their users.”
(Adapted from aljazeera.com)
Question 44: Which best serves as the title for the passage?
A. New Measures That Can Be Taken To Deal With Online Racism
B. Racism In The Virtual World: A Growing Problem
C. Why Do Hackers Always Target Zoom For Security Loopholes?
D. How Online Racism Affects People’s Mental State
Question 45: The word “lobbied” in paragraph 2 can be best replaced by ___________.
A. threatened B. forced C. urged D. deceived
Question 46: Why does Rashad Robinson say in paragraph 2 that the BAME community face
“immediate danger” when using Zoom?
A. Because Zoom has not tried to tackle civil rights violations.
B. Because this community is not being protected by Zoom.
C. Because hackers are planning to attack all Zoom users.
D. Because Zoom still has too many security loopholes.
Question 47: According to paragraph 3, which of the following is NOT an example of gatherings
that have been hijacked by hackers?
A. A virtual casting call on Zoom for actors who are black.
B. A Zoom meeting between minority students to discuss racism.
C. A series of talks about the aftermath of George Floyd’s death.
D. An event with speakers who are BAME medical students.
Question 48: The word “coordinated” in paragraph 3 is closest in meaning to __________.
A. scheduled B. engineered C. programmed D. organised
Question 49: Which of the following can be inferred from the passage?
A. No social media company has addressed civil rights violations.
B. White Coats for Black Lives is very popular with medical students.

157
C. Lydia Amoah was deeply affected emotionally after a cyber-attack.
D. Zoom will soon release its plan to deal with racial harassment.
Question 50: The word “them” in paragraph 4 refers to ___________.
A. colleagues B. clients C. experts D. hackers
ĐỀ 7
Mark the letter A, B, C, or D on your answer sheet to indicate the sentence that best completes
each of the following exchanges.
Question 1: -“Why don’t we have a picnic this weekend?” “__________________.”
A. That’s a good idea B. No, thanks C. I’m leaving now D. I don’t care
Question 2: Lan and Hoa are discussing about life in the future.
Lan: “I strongly agree that robots will help us do housework well. Hoa: “_______.”
A. It is very kind of you to say so B. I’m sorry to hear that
C. It doesn’t matter at all D. There’s no doubt about that.
Mark the letter A, B, C, or D on your answer sheet to indicate the correct answer to each of the
following questions.
Question 3: Frank’s wallet is ________ in his entire collection.
A. the most cheap B. the expensivest C. the most expensive D. the cheaper
Question 4: Players who are not in ________ of the ball are prohibited from impeding the
opponent's movements.
A. possessive B. possessively C. possession D. possesser
Question 5: Fifth grader Nguyen Minh Duc, _______ the test on August 13, scored 8.0 in
listening and speaking, 7.0 in reading and 6.5 in writing.
A. taking B. to take C. to be taken D. take
Question 6: I had left my key in ________ kitchen, I would like you to get for me.
A. no article B. the C. a D. B & C are correct
Question 7: Paul’s grandfather still hasn’t really ________ the death of his grandmother.
A. got over B. taken after C. gone off D. looked after
Question 8: I didn’t regret________ a gap year before entering university.
A. stake B. to take C. taking D. to taking
Question 9: There are a lot of professors attending the video conference, _______?
A. are they B. are there C. aren’t they D. aren’t there
Question 10: She thinks this is a ________ problem. Her laptop screen is out of working.
A. hardware B. software C. device D. equipment
Question 11: We will complete all of tasks _________.
A. as soon as she will come B. as soon as she comes
C. as soon as she came D. as soon as she had come
Question 12: At the memorial ceremony, everyone stood in silence and ________ respect to the
soldiers who sacrificed their lives in the war.
A. paid B. got C. made D. took
Question 13: When she came back her house, she found that her son ________ doing housework.
A. did B. was doing C. is doing D. does

158
Question 14: That bridge ________ by city officials since last year.
A. monitored B. has monitored C. was monitored D. has been monitored
Question 15: Sam attempted to ________ her thoughts prior to responding to my inquiry.
A. install B. gather C. order D. give
Question 16: The concert is anticipated to attract a large audience, so it is advisable to book the
tickets ________ advance.
A. in B. at C. with D. on
Question 17: It appears that Peter will forget his lost wallet. I suppose he is ________.
A. a hot potato B. a lost cause C. bucket down D. at heart
Mark the letter A, B, C, or D on your answer sheet to indicate the word whose underlined part
differs from the other three in pronunciation in each of the following questions.
Question 18: A. hide B. kill C. give D. pick
Question 19: A. choice B. chore C. child D. chemist
Mark the letter A, B, C, or D on your answer sheet to indicate the word that differs from the
other three in the position of stress in each of the following questions.
Question 20: A. effort B. actor C. talent D. success
Question 21: A. dramatic B. expensive C. comfortable D. dependent
Mark the letter A, B, C, or D on your answer sheet to indicate the word CLOSEST in meaning to
the underlined word in each of the following questions.
Question 22: In Phu Quoc island, visitors may see and do a lot for an affordable cost.
A. costive B. exorbitant C. reasonable D. stingy
Question 23: I thoroughly cleaned my room before guests arrived.
A. superficially B. hastily C. carelessly D. completely
Mark the letter A, B, C, or D on your answer sheet to indicate the word(s) OPPOSITE in
meaning to the underlined word(s) in each of the following questions.
Question 24: Despite facing numerous challenges, the resilient individual managed to hang in
there despite the hurricane.
A. stop B. continue C. try D. endure
Question 25: Many young people often grow more plants in order that they can absorb more
carbon dioxide from the air.
A. affect B. release C. uptake D. consume

Read the following passage and mark the letter A, B, C, or D on your answer sheet to indicate
the correct word or phrase that best fits each of the numbered blanks from 26 to 30.
In today's business world, companies often pay other (26) ________ to perform certain tasks or
even whole projects in order to save money or time. This practice is known as outsourcing and is
common in several (27) ________, particularly information technology, customer services and
market research. Outsourcing is not a new phenomenon. It has been occuring in the manufacturing
sector for decades, but the 1990s saw a huge expansion in outsourcing in the service sector.
Advances in the Internet and communications technology made it possible to assign routine tasks
159
to workers in countries such as India, (28) ________ has the largest educated English-speaking
population in the world. Another benefit is the 12-hour time difference between India and the
United States, which means that a job that is submitted at the end of America’s working day arrives
in India early in the morning, and can be completed (29) ________ the American company opens
for business again the next day.
In (30)________ cases work is outsourced overseas,which results in financial gains as salaries are
often considerably lower, not to mention the savings made by not maintaining office space.
(Adapted from Master Mind)
Question 26: A. boards B. assets C. firms D. boats
Question 27: A. fields B. aspects C. duties D. levels
Question 28: A. who B. which C. when D. where
Question 29: A. before B. during C. if D. though
Question 30: A. little B. another C. a lot D. many
Read the following passage and mark the letter A, B, C, or D on your answer sheet to indicate
the correct answer to each of the questions from 31 to 35.
Every year, the Economist Intelligence Unit (EIU) conducts a fascinating survey to
determine which cities around the world provide “the best or worst living conditions”. It uses
factors such as climate, transport, education, safety, and recreational facilities for around 140 world
cities. It gives scores for each, and then ranks the cities in order - from the ‘best’ to the worst’.
This year all of the top ten cities came from either Canada, Australia, or Western Europe.
Vancouver, Canada had the highest score, which means it is the most ‘liveable’city. Two other
Canadian cities, Toronto (4th place) and Calgary (5th), were also in the top five. In second place
was Vienna (Austria) followed by Melbourne (Australia). The final two cities in the top ten were
Adelaide (Australia) in 9th place and Auckland (New Zealand) in 10th. At the bottom of the list
were the cities with the most difficult or dangerous living conditions.
The city with the lowest score was Harare, Zimbabwe, in 140th place. Some other African cities,
such as Lagos, Nigeria, also did badly. This could be because of climate, or the political situation
in these countries. In the middle of the list came big cosmopolitan cities with their transport and
crime problems. These included Paris (17th), London (54th), and New York (56th). The Japanese
cities of Osaka and Tokyo did better, however. These cities (both in the top twenty) also had the
biggest scores in Asia along with Singapore and Hong Kong.
(Adapted from Headway Academic Skills)
Question 31: Which could be the best title for the passage?
A. The best city in the world to live and work
B. Top cities for quality life
C. Annual survey ranks best cities for living and working
D. Canada, Australia, and Western Europe are the best cities to live and work
Question 32: The word it in paragraph 1 refers to _________.
A. the city B. the survey C. EIU D. living condition
Question 33: According to the passage, which cities were ranked comparable to Hong Kong and
Singapore?
160
A. Paris and Tokyo B. Tokyo and London C. Tokyo and Osaka D. London and Nigeria
Question 34: The word cosmopolitan in paragraph 3 is closest in meaning to_________.
A. urban B. provincial C. parochia D. rural
Question 35: Which of the following is NOT mentioned, according to the passage?
A. Climate, transport, education, safety, and recreational facilities are factors to evaluate the best
cities.
B. Paris, London and New York are liveable cities in the world.
C. Some other poorly ranked African cities like Lagos, Nigeria.
D. Osaka and Tokyo had the biggest scores in Asia along with Singapore and Hong Kong.
Read the following passage and mark the letter A, B, C, or D on your answer sheet to indicate
the correct answer to each of the questions from 36 to 42.
In recent times developing commercial revenues has become more challenging for airports due
to a combination of factors, such as increased competition from Internet shopping, restrictions on
certain sales, such as tobacco, and new security procedures that have had an impact on the dwell
time of passengers. Moreover, the global economic downturn has caused a reduction in passenger
numbers while those that are travelling generally have less money to spend. This has meant that the
share of revenue from non-aeronautical revenues actually peaked at 54% at the turn of the century
and has subsequently declined slightly. Meanwhile, the pressures to control the level of
aeronautical revenues are as strong as ever due to the poor financial health of many airlines and the
rapid rise of the low-cost carrier sector.
Some of the more obvious solutions to growing commercial revenues, such as extending the
merchandising space or expanding the variety of shopping opportunities, have already been tried to
their limit at many airports. A more radical solution is to find new sources of commercial revenue
within the terminal, and this has been explored by many airports over the last decade or so. As a
result, many terminals are now much more than just shopping malls and offer an array of
entertainment, leisure, and beauty and wellness facilities. At this stage of facilities provision, the
airport also has the possibility of taking on the role of the final destination rather than merely a
facilitator of access.
At the same time, airports have been developing and expanding the range of services that they
provide specifically for the business traveller in the terminal. This includes offering business
centres that supply support services, meeting or conference rooms and other space for special
events. Within this context, Jarach (2001) discusses how dedicated meetings facilities located
within the terminal and managed directly by the airport operator may be regarded as an expansion
of the concept of airline lounges or as a way to reconvert abandoned or underused areas of terminal
buildings. Previously it was primarily airport
hotels and other facilities offered in the surrounding area of the airport that had the potential to take
on this role and become active as a business space (McNeill, 2009).
(Adapted from IELTS Official Cambridge Guide)
Question 36: What is the best title of the passage?
A. A reduction in passenger numbers for shopping at commercial revenues.
B. Airports and commercial revenues.
161
C. Challenges and strategies for commercial revenues at airports.
D. Airports should develop and expand the range of services.
Question 37: Which of the following is NOT TRUE according to the passage?
A. The revenue from non-aeronautical revenues actually peaked at the turn of the century and
decreased considerably.
B. The sector of budget airlines or affordable air travel increased rapidly.
C. Airports have been exploring new sources of commercial revenue within the terminal like array
of entertainment, leisure, and beauty and wellness facilities.
D. Airports supply support services, meeting or conference rooms and other space for special
events.
Question 38: To grow in commercial revenues at the airports, they_________.
A. provide shopping online services, meeting or conference rooms and other space for special
events.
B. extend the merchandising space or expand shopping opportunities have already been maximized
at many airports.
C. abandon or underuse areas of terminal buildings.
D. create more active business spaces.
Question 39: What does the word “revenues” in the first paragraph refer to?
A. income B. expenses C. costs D. debts
Question 40: What does the word “facilities” in the second paragraph refer to?
A. constrains B. limitations C. restrictions D. equipment
Question 41: What does the word “they” in the third paragraph refer to?
A. services B. airports C. travellers D. centres
Question 42: Which of the following can be inferred from the reading passage?
A. A reduction of commercial revenues is a serious problem.
B. The effects of expanding the range of services are still contrersial
C. Several effective strategies to raise airport commercial revenue.
D. Widening shopping malls and services at the airport is not necessary.
Mark the letter A, B, C, or D on your answer sheet to indicate the sentence that best combines
each pair of sentences in the following questions.
Question 43: Charles had successfully completed the high school program at his school. He was
awarded a valuable scholarship by the institution.
A. No sooner had Charles successfully completed the high school program at his school than he
was awarded a valuable scholarship by the institution.
B. Only after Charles was awarded a valuable scholarship by the institution did he successfully
complete the high school program at his school.
C. Not until Charles was awarded a valuable scholarship by the institution did he successfully
complete the high school program at his school.
D. Had it not been for Charles’s successful completion the high school program at his school, he
would have been awarded a valuable scholarship by the institution.

162
Question 44: Jane had some school projects to do last month. She couldn’t attend her brother’s
wedding.
A. If Jane hadn’t some school projects to do last month, she could attend her brother’s wedding.
B. If Jane had had some school projects to do last month, she couldn’t have attended her brother’s
wedding.
C. If Jane hadn’t had any school projects to do last month, she could have attended her brother’s
wedding.
D. If Jane didn’t have any school projects to do last month, she could attend her brother’s wedding.
Mark the letter A, B, C, or D on your answer sheet to indicate the underlined part that needs
correction in each of the following questions.
Question 45: My sister haven’t got the job she expected because she didn't have the right
qualifications two years ago. A B C
D
Question 46: Even though camels and elephants could not fly, it had other abilities like running.
A B C D
Question 47: The investigation revealed farther evidence of his involvement.
A B C D
Mark the letter A, B, C, or D on your answer sheet to indicate the sentence that is closest in
meaning to each of the following questions.
Question 48: It’s a good idea for him to practice English skills with his spouse every single day.
A. He must practice English skills with his spouse every single day.
B. He should practice English skills with his spouse every single day.
C. He can practice English skills with his spouse every single day.
D. He will practice English skills with his spouse every single day.
Question 49: It started to rain in the early morning and it is still raining.
A. It has been raining in the early morning. B. It has been raining for the early morning.
C. It has been raining at the early morning. D. It has been raining since the early morning.
Question 50: My friend will be taken to the hospital due to his accident.” said the man
A. The man said his friend would be taken to the hospital due to his accident.
B. The man said his friend would take to the hospital due to his accident.
C. The man said my friend will be taken to the hospital due to his accident.
D. The man said his friend will be taken to the hospital due to his accident.

ĐỀ 8
Mark the letter A, B, C, or D to indicate the sentence that best completes each of the following
exchanges.
Question 1. Tom and Susan are talking about their plan to help disadvantaged children.
- Tom: "Why don't we send disadvantaged children in the remote and mountainous areas some
textbooks and warm clothes?" - Susan: "_________”

163
A. I'm sorry to hear that. B. No, they are not available.
C. Great idea! What meaningful gifts! D. You should agree with us.
Question 2. The teacher is discussing the cyberbullying with Jack.
- Jack: "Cyberbullying has serious effects on not only a person's mental health but also his well-
being." - Teacher: “ ______. We have to prevent it as soon as possible.”
A. I don’t think so B. I don’t agree C. That's a good point D. What nonsense
Mark the letter A, B, C or D to indicate the correct answer to each of the following questions.
Question 3. Chocolates and flowers are by far _______ presents for mothers on Mother’s Day in
the UK. A. more popular B. the more popular C. less popular D. the most popular
Question 4. I think that up to now there has not been a real _______ between men and women.
A. equality B. equal C. equalize D. equally
Question 5. A child's application code and the consultant's Zalo phone number are sent to parents
upon registration, _______ them to access Zalo and register personal data.
A. is enabled B. enabling C. to enable D. enabled
Question 6. _______ fifth-grade pupil died of suspected food poisoning on April 5th, 2024 in Nha
Trang.A. The B. An C. A D. x (No article)
Question 7. As Joe's roommate, I find him a fairly nice fellow, even if at times it is not easy to
_______ his noisy behavior.
A. put up with B. look up to C. get on with D. catch up with
Question 8. The candy _______ by a student from the school at a general grocery store yesterday.
A. purchased B. was purchasing C. was purchased D. is purchased
Question 9. They remember _______ an unforgettable party for their grandparents' anniversary.
A. throw B. to throw C. throwing D. thrown
Question 10. People can never feel fulfilled by following someone else's dreams, _______?
A. do they B. don’t they C. can’t they D. can they
Question 11. The growth in employment and wages gives consumers some spending _______ to
absorb the higher cost of energy.
A. energy B. force C. ability D. power
Question 12. I _______ along the street when I suddenly heard footsteps behind me.
A. was walking B. walk C. am walking D. walked
Question 13. Our group _______ a new method to carry out the survey into teenagers’ attitudes
towards online learning.
A. recruited B. employed C. occupied D. engaged
Question 14. ________, she will buy her parents a new TV set.
A. When she receives her salary B. Once she received her salary
C. Until she received her salary D. After she had received her salary
Question 15. I am sure your sister will _______ you a sympathetic ear when you explain the
situation to her.
A. pay B. lend C. borrow D. take
Question 16. People are sometimes sick _______ the Internet users' group arguments in online
forums.A. of B. to C. into D. off

164
Question 17. When students take a step out of their _______ zone, they need some encouragement
from their teachers and parents.
A. climate B. parking C. control D. comfort
Mark the letter A, B, C, or D to indicate the word whose underlined part differs from the other
three in pronunciation in each of the following questions.
Question 18. A. come B. cake C. city D. club
Question 19. A. find B. think C. drive D. mind
Mark the letter A, B, C, or D to indicate the word that differs from the other three in the position
of the primary stress in each of the following questions.
Question 20. A. express B. happen C. employ D. reduce
Question 21. A. advantage B. candidate C. intention D. commitment
Mark the letter A, B, C or D to indicate the word(s) CLOSEST in meaning to the underlined
word(s) in each of the following questions.
Question 22. Fierce storms have been hampering rescue efforts and there is now little chance of
finding more survivors.
A. allowing B. preventing C. encouraging D. promoting
Question 23. With the dawn of space exploration, the notion that atmospheric conditions on Earth
may be unique in the solar system was strengthened.
A. outcome B. beginning C. expansion D. continuation
Mark the letter A, B, C or D to indicate the word(s) OPPOSITE in meaning to the underlined
word(s) in each of the following questions.
Question 24. Various programs have been designed in order to facilitate the storage and analysis
of research data.
A. ease B. block C. speed D. build
Question 25. He really seems to have fallen on his feet. He got a new job and found a flat within a
week of arriving in the city.
A. have been successful B. have been satisfied
C. have been unlucky D. have been disappointed
Read the following passage and mark the letter A, B, C, or D to indicate the correct word or
phrase that best fits each the numbered blanks.
Every year millions of migrants travel vast distances using borrowed money for their airfares
and taking little or no cash with them. This flow of migrant money has a huge economic and social
(26) ______ on the receiving countries. It provides cash for food, housing and necessities. It funds
education and healthcare and contributes towards the upkeep of the elderly. Extra money is sent for
special events such as weddings, funerals or urgent medical procedures and (27) ______
emergencies. (28) ______, it sometimes becomes the capital for starting up a small enterprise. A
twofold benefit would be achieved by a developing country or a large charitable society, (29)
______ sells bonds guaranteeing a return of three or four percent, provided that the invested money
is utilized for building infrastructure within that same country. Migrants would make a financial
gain and see their savings put to work in the development of their country of (30) ______.
(Adapted from ‘sending money home’ from ielts of British Council.)
Question 26. A. result B. impact C. consequence D. outcome
Question 27. A. other B. another C. much D. little
Question 28. A. However B. Moreover C. Therefore D. Although
165
Question 29. A. which B. who C. whose D. when
Question 30. A. derivation B. foundation C. supply D. origin
Read the following passage and mark the letter A, B, C, or D to indicate the answer to each of
the question.
There is nothing to suggest that Evelyn Glennie is profoundly deaf. She insists that her
deafness is irrelevant to her musicianship, but there is no doubt that her obvious handicap has
turned a remarkable career into a miraculous one.
Glennie was eight when her hearing began to fail; by twelve she had lost it completely and
feared she would have to give up the music she loved. But a doctor’s suggestion that she should
become an accountant rather than follow a hopeless musical career strengthened her will to
succeed.
As it turned out, music was one of the most advantageous careers she could have chosen and is
one reason why her speech remains so extraordinarily correct, despite her deafness. Occasionally
she listens to recordings by holding a cassette player between her knees, interpreting the vibrations
and the shaking movements. Her deafness is one of the reasons for her unique style, for she cannot
listen and be influenced by other performances and she has often declared that getting her hearing
back would be the worst thing that could happen to her.
As a result of her devotion to her music and her determination to succeed, she has doubled the
range of works available for percussion music in Europe and introduced instruments previously
unheard of in the west. She has also asked composers to write more than fifty new pieces of music
for these instruments, and has set up a library of three hundred works for other musicians to use.
(Adapted from Richmond FCE Practice Tests)
Question 31. Which of the following can be the best title for the passage?
A. How to be a Successful Singer B. The Disadvantages of Deafness
C. Developing Musical Skills at School D. Overcoming a Severe Disability
Question 32. The word it in paragraph 2 refers to ______.
A. hearing B. handicap C. career D. music
Question 33. The word unique in paragraph 3 is closest in meaning to ______.
A. special B. boring C. common D. healthy
Question 34. According to paragraph 4, Glennie has ______.
A. refused to introduce new instruments to the west
B. written over fifty new pieces of music by herself
C. established a library to support other musicians
D. given up the hope to succeed in her musical career
Question 35. Which of the following is TRUE according to the passage?
A. People can easily tell that Glennie is completely deaf.
B. Glennie’s deafness turns out to be an advantage to her career.
C. Glennie wishes she could hear clearly like a normal person.
D. Glennie’s doctor encouraged her to become a musician.
Read the following passage and mark the letter A, B, C, or D to indicate the answer to each of
the question.
The authors noted that while other forms of pollution are decreasing, noise pollution has been
increasing. This conclusion is reinforced by the fact that there has been an increase in the number

166
of people who have been complaining about excessive noise in the WHO region. Populations that
are exposed to high noise levels can be afflicted by other symptoms such as: stress reactions, sleep-
stage changes, and clinical symptoms like hypertension and cardiovascular diseases. All these
impacts can contribute to premature mortality. It is important to note that these adverse health
problems impact all age groups including children and adolescents. In fact, it has been reported that
children who live and or study in an area afflicted with noise pollution tend to suffer from stress,
impairments in memory and attention as well as difficulty reading.
“In Western Europe, the guidelines say, traffic noise results in an annual loss of at least one
million healthy years”. Traffic noise is currently ranked second among environmental threats to
public health.
The danger of noise pollution is more present to us when we are asleep. Because the human ear
is so sensitive, it never rests, it is always working, picking up and transmitting sounds for our
brains to interpret. This always on working process is where the danger lies, though you may be
sleeping, sounds are still being picked up and processed. The most common side effects of this
phenomenon are sleep disturbance and tiredness, impaired memory judgment, and psychomotor
skills. The other more serious outcomes of this can be the triggering of the body’s acute stress
response, which raises blood pressure and heart rate as the body and brain go into a state of
hyperarousal. According to the European Environment Agency, at least 10,000 cases of premature
deaths from noise exposure occur each year, although incomplete data mean this number is
significantly underestimated.
Question 36. Which of the following is the best title for the passage?
A. Noise in the WHO Region B. environmental threats
C. Increase in Noise, Increase in Risk D. Side Effects of Tiredness
Question 37. According to paragraph 2, in Western Europe, an annual loss of at least one million
healthy years is resulted from _______.
A. guidelines B. environment threats C. traffic noise D. public health
Question 38. The word premature in paragraph 1 is closest in meaning to _______.
A. slow B. late C. quick D. early
Question 39. The word it in paragraph 3 refers to _______.
A. human ear B. sound C. the danger D. side effect
Question 40. The word afflicted in paragraph 1 is closest in meaning to _______.
A. assumed B. illustrated C. affected D. described
Question 41. Which of the following is NOT mentioned in the article as a symptom caused by
exposure to high noise levels?
A. staying highly alert B. stress reactions
C. cardiovascular diseases D. hypertension
Question 42. Which of the following can be inferred from the passage?
A. The danger of noise pollution is less serious when we are asleep.
B. Adverse health problems impact children and adolescents only.
C. Many forms of pollution, except for noise pollution, are decreasing.
D. At least 100,000 cases of premature deaths from noise exposure occur each year.
Mark the letter A, B, C, or D to indicate the sentence that best combines each pair of sentences
in the following questions.

167
Question 43. Laura didn’t take part in the marathon. She was badly injured in a car accident last
week.
A. Without her bad injury in a car accident last week, Laura wouldn’t have taken part in the
marathon.
B. If Laura hadn’t been badly injured in a car accident last week, she would have taken part in
the marathon.
C. If only Laura weren’t badly injured in a car accident last week, she could have taken part in
the marathon.
D. Laura could take part in the marathon in case she were badly injured in a car accident last
week.
Question 44. Peter failed the interview again. He was fully aware of the importance of making
careful preparation only then.
A. No sooner had Peter fully been aware of the importance of making careful preparation than
he failed the interview again.
B. Hardly had Peter fully been aware of the importance of making careful preparation when he
failed the interview again.
C. Not until Peter was fully aware of the importance of making careful preparation did he fail
the interview again.
D. Only after Peter had failed the interview again was he fully aware of the importance of
making careful preparation.
Mark the letter A, B, C or D to indicate the underlined part that needs correction in each of the
following questions.
Question 45. Some children still face discrimination at school because of its sexual orientation
A. Some B. face C. at school D. its
Question 46. After going through a protection period that may last from several months to a year,
new recruits will be offered a long-term contract.
A. months B. protection C. contract D. recruits
Question 47. Yesterday, for the first time in his life, Mike refuses to follow his parents' advice.
A. first time B. refuses C. to follow D. advice
Mark the letter A, B, C, or D to indicate the sentence that is closest in meaning to each of the
following questions.
Question 48. They are not allowed to use our personal information for their own purposes.
A. They don’t have to use our personal information for their own purposes.
B. They can use our personal information for their own purposes.
C. They mustn’t use our personal information for their own purposes.
D. They may use our personal information for their own purposes.
Question 49. Linda last went to the university library three months ago.
A. Linda hasn’t gone to the university library for three months.
B. Linda went to the university library for three months.
C. Linda has gone to the university library for three months.
D. Linda didn’t go to the university library for three months.
Question 50. Jack told me, "I want to see this movie with my girlfriend tomorrow."
A. Jack told me he wanted to see that movie with my girlfriend tomorrow.
168
B. Jack told me he wanted to see that movie with his girlfriend tomorrow.
C. Jack told me he wanted to see that movie with my girlfriend the following day.
D. Jack told me he wanted to see that movie with his girlfriend the following day.
ĐỀ 9
Mark the letter A, B, C or D on your answer sheet to indicate the word whose underlined part
differs from the other three in pronunciation in each of the following questions.
Question 1: A. chairman B. chemist C. challenge D. childhood
Question 2: A. like B. hide C. pick D. shine
Mark the letter A, B, C or D on your answer sheet to indicate the sentence that best completes
each of the following exchanges.
Question 3: Linda and Daisy are talking about the environment.
- Linda: "Our environment is getting more and more polluted." - Daisy: "___. It's really worrying."
A. Surely that's not the case B. Actually, I can't make it
C. Sorry to hear that D. I totally agree with you
Question 4: William and Janet have just finished the meeting with their customers.
-William: "Shall I bring your laptop back to the office?" - Janet: “_____. Please put it on my desk."
A. Thank you. That would be great B. No. I forgot my laptop
C. I can't carry it D. Oh, I can help you
Mark the letter A, B, C or D on your answer sheet to indicate the sentence that is closest in
meaning to each of the following questions.
Question 5: It is unnecessary for you to finish the report until tomorrow afternoon.
A. You should finish the report until tomorrow afternoon.
B. You have to finish the report until tomorrow afternoon.
C. You mustn't finish the report after tomorrow afternoon.
D. You needn't finish the report until tomorrow afternoon.
Question 6: Daisy asked Tom: "Did you buy a new car last year?".
A. Daisy asked Tom if you had bought a new car the previous year.
B. Daisy asked Tom if he would buy a new car the next year.
C. Daisy asked Tom whether he had bought a new car the previous year.
D. Daisy asked Tom whether you bought a new car last year.
Question 7: The last time I updated my Facebook status was 6 months ago.
A. I have updated my Facebook status for 6 months.
B. I have 6 months to update my Facebook status.
C. I haven't updated my Facebook status for 6 months.
D. I started updating my Facebook status 6 months ago.
Mark the letter A, B, C or D to indicate the underlined part that needs correction in each of the
following questions.
Question 8: My cousin was going to the supermarket to buy some sugar at present.
A. My cousin B. was going C. the D. some
Question 9: Farmers are concerned that their food-bearing plants may be subjective to the
difficulties of a severe winter.
A. concerned B. subjective C. difficulties D. severe
Question 10: Electric bikes are becoming increasingly popular nowadays thanks to its
convenience.
A. are becoming B. popular C. thanks D. its
Mark the letter A, B, C or D on your answer sheet to indicate the word that differs from the
other three in the position of primary stress in each of the following questions.
169
Question 11: A. develop B. devastate C. integrate D. organise
Question 12: A. area B. effort C. actor D. disease
Read the following passage and mark the letter A, B, C or D on your answer sheet to indicate the
correct answer to each of the questions from 13 to 17.
The Singapore Science Centre is located on a six-hectare site in Jurong. At the centre, we can
discover the wonders of science and technology in fun way. Clap your hands and colourful bulbs
will light up. Start a wheel spinning and it will set off a fan-churning. It is a place to answer our
curiosity and capture our imagination.
The centre features over four hundred exhibits covering topics like solar radiation,
communication, electronics, mathematics, nuclear energy, and evolution. It aims to arouse interest
in science and technology among us and the general public. The centre is the first science one to be
established in Southeast Asia. It was opened in 1977 and it now receives an average of one
thousand two hundred visitors a day. The exhibits can be found in four exhibition galleries. They
are the Lobby, Physical Sciences, Life Sciences and Aviation. These exhibits are renovated
annually so as to encourage visitors to make return visits to the centre.
Instead of the usual "Hands off" notices found in exhibition halls, visitors are invited to touch
and feel the exhibits, push the buttons, turn the cranks or pedals. This is an interesting way to learn
science even if you hate the subject. A Discovery Centre was built for children between the ages of
three and twelve. This new exhibition gallery was completed in 1985. Lately this year a stone-age
exhibit was built. It shows us about the animals and people which were extinct.
(Adapted from language123.blogspot.com)
Question 13: What can be the best title of the passage?
A. Science Centre B. Discovery Centre
C. Singapore Science Centre D. Physical Sciences
Question 14: The word It in paragraph 2 refers to______.
A. Solar radiation B. The general public C. Evolution D. The centre
Question 15: According to paragraph 2, which of the following is NOT true about the Singapore
Science Centre?
A. The centre was not opened until 1977.
B. The centre is the first one to be established in the world.
C. Visitors are encouraged to return to the centre.
D. The exhibits in the centre cover a wide range of topics.
Question 16: What does the word renovated in paragraph 2 mostly mean?
A. shown B. upgraded C. changed D. painted
Question 17: What can visitors do in the Singapore Science Centre?
A. hate the subject B. build a new exhibition gallery
C. touch and feel the exhibits D. invite other guests
Read the following passage and mark the letter A, B, C or D on your answer sheet to indicate the
correct answer to each of the questions from 18 to 24.
We get great pleasure from reading. The more advanced a man is the greater delight he will
find in reading. The ordinary man may think that subjects like philosophy or science are very
difficult and that if philosophers and scientists read these subjects, it is not for pleasure. But this is
not true. The mathematician finds the same pleasure in his mathematics as the school boy in an
adventure story. For both, it is a play of the imagination, a mental recreation and exercise. The
pleasure derived from this activity is common to all kinds of reading. But different types of books
give us different types of pleasure. First in order of popularity is novel-reading. Novels contain
pictures of imaginary people in imaginary situations, and give us an opportunity of escaping into a

170
new world very much like our world and yet different from it. Here we seem to live a new life, and
the experience of this new life gives us a thrill of pleasure. Next in order of popularity are travel
books, biographies and memoirs. These tell us tales of places we have not seen and of great men in
whom we are interested. Some of these books are as wonderful as novels, and they have an added
value that they are true.
Such books give us knowledge, and we also find immense pleasure in knowing details of lands
we have not seen and of great men we have only heard of. Reading is one of the greatest
enjoyments of To book- lovers, nothing is more fascinating than a favourite book. And, the
ordinary educated man who is interested and absorbed in his daily occupation wants to
occasionally escape from his drudgery into the wonderland of books for recreation and
refreshment. (Adapted from http://www.importantindia.com)
Question 18: What does the passage mainly discuss?
A. Reading as a pleasurable activity B. Different types of books
C. Reading as an exercise for the brain D. Different kinds of reading
Question 19: According to paragraph 1, which of the following is NOT true?
A. Philosophers and scientists do not read for pleasure.
B. Ordinary people may think that philosophy and science are difficult.
C. Reading about mathematics is mentally entertaining for a mathematician.
D. A more advanced person takes greater pleasure in reading.
Question 20: The word derived in paragraph 1 is closest in meaning to_________.
A. differed B. established C. obtained D. bought
Question 21: The word it in paragraph 1 refers to______.
A. a new life B. a thrill of pleasure C. our world D. an opportunity
Question 22: The word immense in paragraph 2 is closest in meaning to_______.
A. personal B. great C. limited D. controlled
Question 23: According to the passage, travel books, biographies and memoirs_____.
A. tell stories of well-known places B. are less popular than novels
C. are more valuable than novels D. are wonderful novels
Question 24: Which of the following can be inferred from the last paragraph?
A. People can get not only knowledge but also pleasure from books.
B. Reading is the greatest enjoyments of life.
C. Nothing is more important than a good book.
D. People occasionally escape from their drudgery for recreation and refreshment.
Mark the letter A, B, C or D on your answer sheet to indicate the correct answer to each of the
following questions.
Question 25: Although he knew very little about linguistics, he pretended_____ an expert in the
field.A. being B. to be C. been D. be
Question 26: I have left my book in______ kitchen and I would like you to get it for me.
A. the B. a C. an D.(no article)
Question 27: The report on the vulnerable species_________ by the research team tomorrow.
A. will be completed B. completed C. will complete D. is completing
Question 28: I think that he is______ than his opponent.
A. intelligent B. most intelligent C. more intelligent D. as intelligent
Question 29: These cats look immensely lovely,______?
A. are these B. are they C. don't they D. don’t these
Question 30: Luckily, they successfully_______ the fire and saved all of the children.
A. took out B. took off C. take off D. put out

171
Question 31: _____, she will buy her parents a new TV.
A. After she had received her salary B. Until she received her salary
C. When she receives her salary D. Once she received her salary
Question 32: Now that he's met the love of his life, he finally________ a chance at real happiness.
A. makes B. runs C. has D. keeps
Question 33: My grandmother_________ in the kitchen when she saw a mouse.
A. has cooked B. was cooking C. cooks D. is cooking
Question 34: The little girl______ with my son in the living room is my brother's daughter.
A. was played B. to play C. was playing D. playing
Question 35: Many people have objected to the use of animals in______ experiments.
A. scientific B. scientifically C. sciences D. scientist
Question 36: After Tom's boss had assessed his performance, he was offered a written______ of
employment.
A. certificate B. licence C. contract D. degree
Question 37: Environmental groups try to stop farmers from using harmful______ on their crops.
A. instruments B. lands C. investments D. chemicals
Question 38: This place is famous________ its beautiful landscape and friendly inhabitants.
A. with B. for C. to D. at
Question 39: When hearing the news, Tom tried his best to keep a________ on his surprise.
A. roof B. lid C. hat D. hood
Mark the letter A, B, C or D on your answer sheet to indicate the word(s) OPPOSITE in
meaning to the underlined word(s) in each of the following questions.
Question 40: Many women prefer to use cosmetics to enhance their beauty and make them look
younger.A. maximize B. worsen C. enrich D. improve
Question 41: Peter racked his brains, trying to find a solution to this Maths question.
A. damaged his brains B. operated his brains
C. thought very hard D. thought in a relaxing way
Mark the letter A, B, C or D on your answer sheet to indicate the sentence that best combines
each pair of sentences in the following questions.
Question 42: I don't pay attention to the teacher. I don't understand the lesson.
A. I would understand the lesson if I didn't pay attention to the teacher.
B. Although I pay attention to the teacher, I don't understand the lesson
C. Unless I understood the lesson, I would pay attention to the teacher.
D. I would understand the lesson if I paid attention to the teacher.
Question 43: Peter told us about his leaving the school. He did it on his arrival at the meeting.
A. No sooner had Peter arrived at the meeting than he told us about his leaving the school.
B. Only after his leaving the school did Peter inform us of his arrival at the meeting.
C. Hardly had Peter informed us about his leaving the school when he arrived at the meeting.
D. Not until Peter told us that he would leave the school did he arrive at the meeting.
Mark the letter A, B, C or D on your answer sheet to indicate the word CLOSEST in meaning to
the underlined word in each of the following questions.
Question 44: I think it's impossible to abolish school examinations. They are necessary to evaluate
students' progress.
A. stop B. distinguish C. continue D. organise
Question 45: Tom is an outstanding player. He's won 3-matches in this year's tournament.
A. normal B. kind C. polite D. excellent

172
Read the following passage and mark the letter A, B, C or D on your answer sheet to indicate the
correct word or phrase that best fits each of the numbered blanks from 46 to 50.
WAVE POWER
Engineers are building machinery to harness the power of ocean waves. As an abundant source
of clean energy, wave power can be categorised along with (46)_______ sustainable energy
sources, such as wind and solar power.
Wave power is extracted by wave energy converters, or WECs, which are placed along ocean
(47)________ that produce strong waves. The first wave farm, (48)____ comprises three WECs,
was built off the coast of Portugal in 2008. It was set to produce over two megawatts of energy -
enough to power 2,000 homes. Sadly, the project hit a stumbling block two months into
(49)______ and has remained offline ever since, demonstrating the high probability of technical
difficulties that can still arise.
Engineers are continuing to build wave farms worldwide (50)_______it is worth cultivating the
extraction of wave power. The amount of power that could potentially be extracted is three
terawatts- enough for billions of homes and businesses.
(Adapted from "Use of English for Advanced" by Malcomn Mann and Steve Taylor-Knowles)
Question 46: A. others B. every C. other D. another
Question 47: A. streams B. currents C. floods D. brooks
Question 48: A. which B. why C. who D. that
Question 49: A. operation B. presentation C. demonstration D. preparation
Question 50: A. despite B. so C. but D. because

ĐỀ 10

Mark the letter A, B, C, or D on your answer sheet to indicate the word whose underlined part
differs from the other three in pronunciation in each of the following questions.

Question 1. A. lush B. rule C. bunch D. cut


Question 2. A. chairman B. exchange C. schedule D. chapter

Mark the letter A, B, C, or D on your answer sheet to indicate the word that differs from the
other three in the position of the primary stress in each of the following questions.
Question 3. A. applaud B. allow C. approach D. answer
Question 4. A. marvellous B. sceptical C. fantastic D. positive

Mark the letter A, B, C, or D on your answer sheet to indicate the correct answer to each of the
following questions.
Question 5. The examination is going to take place at the end of June, ______ ?
A. isn’t it B. doesn’t it C. wasn’t it D. won’t it
Question 6. Sam has the habit of adding ______ sugar on his coffee.
A. a B. an C. Ø (no article) D. the
Question 7. We will respond to you about the complaint ______ .
A. after the manager had made a decision B. when the manager made a decision
C. before the manager made a decision D. as soon as the manager makes a decision
Question 8. Jack was the last one ______ the room but he didn't turn off the lights.
A. leaves B. left C. was leaving D. to leave
Question 9. Mary is dedicated ______ helping disabled children in her neighbourhood.
173
A. for B. with C. to D. by
Question 10. The restaurant ______ to accommodate more diners.
A. should be enlarged B. will enlarge C. has enlarged D. enlarges
Question 11. This is ______ storm that we’ve ever experienced.
A. the heavier B. the heaviest C. heavier D. heaviest
Question 12. After a three-hour operation, she hasn’t ______ yet.
A. cut down B. taken off C. put out D. come around
Question 13. You should consider ______ another employee to help with the project.
A. to hire B. hiring C. hire D. to hiring
Question 14. You’d better leave him to his own ______ ; he’s already a grown-up man.
A. devices B. instruments C. appliances D. gadgets
Question 15. Since Chat GPT was launched, many university students ______ advantage of it to
hone their writing skills.
A. had taken B. would take C. were taking D. have taken
Question 16. During the Covid-19 pandemic, bus ______ were required to wear face masks.
A. guests B. clients C. customers D. passengers
Question 17. We have ______ significant progress in finding a cure for cancer.
A. kept B. made C. put D. taken
Question 18. The manager seemed favourably ______ towards the idea of flexible working hours.
A. dislodged B. disposed C. discerned D. discarded
Question 19. Many young people should be equipped with necessary soft skills so as to ______ in
life.
A. successfully B. succeed C. successful D. success
Mark the letter A, B, C, or D on your answer sheet to indicate the underlined part that needs
correction in each of the following questions.
Question 20. During her illustrative career as a medical practitioner, she has always extolled the
A B C
virtue of consuming less fat and doing more exercise.
D
Question 21. The restaurant where we have dinner last Friday is famous for its fresh ingredients.
A B C D
Question 22. An increased number of university graduates return to its hometowns after finishing
A B C
tertiary education.
D
Mark the letter A, B, C, or D on your answer sheet to indicate the most suitable response to
complete each of the following exchanges.
Question 23. Kate and Kathy are talking about books.
- Kate: “I think printed books will be replaced by e-books in the near future.”
- Kathy: “ ______ . Many people still enjoy reading printed books.”
A. For sure B. I can’t agree with you more
C. I don’t go along with you D. It’s hard to say
Question 24. Ben is inviting Bob to his birthday party.
- Ben: “Would you like to come to my birthday party on Wednesday?”
- Bob: “ ______ . I have arranged a meeting on that day.”
A. Of course B. I’d love to C. I’m sorry D. No, I won’t

174
Mark the letter A, B, C, or D on your answer sheet to indicate the sentence that is closest in
meaning to each of the following questions.
Question 25. “Where did you see our advertisement?” The man asked me.
A. The man asked me where had I seen their advertisement.
B. The man asked me where did I see their advertisement.
C. The man asked me where I saw their advertisement.
D. The man asked me where I had seen their advertisement.
Question 26. I started to use this computer when I was 18.
A. I didn’t use this computer at the age of 18.
B. I have used this computer since I was 18.
C. I haven’t used this computer since I was 18.
D. I have used this computer for 18 years.
Question 27. You are permitted to bring your calculator into the exam room.
A. You should bring your calculator into the exam room.
B. You can bring your calculator into the exam room.
C. You mustn’t bring your calculator into the exam room.
D. You mayn’t bring your calculator into the exam room.
Mark the letter A, B, C, or D on your answer sheet to indicate the sentence that best combines
each pair of sentences in the following questions.
Question 28. Tony really wants to attend the concert. He doesn’t have a ticket.
A. Unless Tony has a ticket, he can’t attend the concert.
B. If Tony didn’t have a ticket, he couldn’t attend the concert.
C. Provided that Tony had had a ticket, he could have attended the concert.
D. Tony wishes he had a ticket so that he could attend the concert.
Question 29. He participated in the environmental campaign. He realised how much damage
humans had caused to the environment only then.
A. Only when he realised how much damage humans had caused to the environment did he take
part in the environmental campaign.
B. Only after he had been involved in the environmental campaign did he realise how much
damage humans had caused to the environment.
C. Not until he had withdrawn from the environmental campaign did he realise how much
damage humans had caused to the environment.
D. No sooner had he joined the environmental campaign than he failed to understand how much
damage humans had caused to the environment.
Mark the letter A, B, C, or D on your answer sheet to indicate the word(s) OPPOSITE in
meaning to the underlined bold word(s) in each of the following questions.
Question 30. I have always respected my father's dedication to his work. He has no days off and
never complains.
A. commitment B. quality C. tradition D. indifference
Question 31. Her constant nagging really gets on my nerves. It’s about time I moved out.
A. criticises me heavily B. annoys me a lot C. comforts me greatly D. praises me highly
Mark the letter A, B, C, or D on your answer sheet to indicate the word(s) CLOSEST in meaning
to the underlined word(s) in each of the following questions.
Question 32. Having read the letter, Anna tore it and threw it away furiously.
A. easily B. angrily C. quietly D. simply
Question 33. We were very happy when the judge complimented on our brilliant performance.
A. exhausting B. terrific C. popular D. disappointing

175
Read the following passage and mark the letter A, B, C, or D on your answer sheet to choose the
word or phrase that best fits each of the numbered blanks from 34 to 38.
Not too long ago, the dinner table was the centre of family life. Parents and children would
gather around the table, enjoy a good home-made meal and discuss the day's events. Nowadays,
(34) ______ , the family dinner table has become a thing of the past. The reason for this is the fact
that parents don't spend as much time with their children as they once used to. In most households,
both parents work and are simply too tired to organise meals and (35)______domestic matters.
Nevertheless, many experts insist that time spent at the family dining table is essential to the
well- being of the family unit. Gathering for meals gives children a (36) ______ of security and
helps them understand and appreciate family life. Experts also claim that children not only get a
nourishing meal but they also get emotional, intellectual and spiritual nourishment. So, whether it
is once a month or once a week, busy parents should (37) ______ aside a few hours of their time to
organise a family meal and spend quality time with their children. After all, the family (38)_____
eats together stays together. (Adapted from FCE Use of English by Virginia Evans)
Question 34. A. however B. as a result C. because D. additionally
Question 35. A. much B. other C. few D. each
Question 36. A. sensation B. scenario C. scene D. sense
Question 37. A. step B. leave C. put D. keep
Question 38. A. whose B. that C. what D. when

Read the following passage and mark the letter A, B, C, or D on your answer sheet to indicate
the correct answer to each of the questions from 39 to 43.
Arriving in Freiburg by train, you might notice the solar panelled towers of the station
building. These solar panels are everywhere; on the football stadium, hotels, the conference centre,
and even a factory. With 1,800 hours of sunshine per year, solar power is the main source of
energy in Freiburg, making it one of the most ecologically friendly cities in the world. In fact,
some say that Freiburg generates nearly as much solar power as the whole of the UK.
In Freiburg it all began in the 1970s, when local people were strongly against plans for a
nuclear power station nearby. The historic centre has been car-free since 1973, with over 400 km of
cycle paths! However, it is the modern residential districts on the outskirts which most interest
environmentalists. In the area called Rieselfeld, residents live in small blocks of south-facing flats
called ‘passive houses’. The design of these houses keeps them warm or cool, without needing
central heating or air conditioning at all. The houses cost around 10% more to build, but over many
years they cut energy bills and energy loss by 90%. Vauban is another quiet area, with solar panels
everywhere and a large number of passive houses. It’s a safe, clean area. It’s busy, but with people
and bicycles instead of vehicles. Officially, only one person in four has a car and it costs around
18,000 euros a year to keep one outside town, in a 'solar- garage'.
In Freiburg, being green is part of the economy, and local people buy shares in different green
projects, for example the solar football stadium. Not surprisingly, this remarkable eco-city has won
many awards and other German cities are now competing to become more eco-friendly. So why
aren't more countries around the world doing the same?
(Adapted from Gateway by Annie Cornford and Frances Watkins)

Question 39. Which would be the best title for the passage?
A. The Future of a Big City B. No Energy, No Life!
C. A Green City D. Initiatives to Go Green
Question 40. The word it in paragraph 1 refers to ______ .

176
A. sunshine B. Freiburg C. solar power D. energy
Question 41. According to paragraph 2, what is special about ‘passive houses’?
A. They’re built from recyclable materials.
B. They’re located in one of the wealthiest areas in Freiburg.
C. They don’t need heating or air conditioning.
D. They’re designed to keep people safe from diseases.
Question 42. The word interest in paragraph 2 mostly means ______ .
A. attract B. compete C. achieve D. consider
Question 43. Which of the following is NOT true, according to the passage?
A. Freiburg has won many awards for its approach to go green.
B. Most people in Vauban use bicycles to travel instead of cars.
C. The production of solar in Freiburg is as much as that in the UK.
D. Great importance is attached to going green in Freiburg.
Read the following passage and mark the letter A, B, C, or D on your answer sheet to indicate
the correct answer to each of the questions from 44 to 50.
Every day we face challenges at school, work or in our personal lives, and we need to develop
the skills to deal with them. Computer skills, for example, are vital for most activities we carry out
in our daily lives as well as helping you with studying. When we are young, we are able to learn
basic social skills but there are other important skills that can be more challenging to acquire.
Surprisingly, video games can often help us to master these.
For example, there are many video games which can help us to develop problem-solving skills.
Some games help develop patience and perseverance skills by continuing to increase the level of
difficulty. They constantly place obstacles in your path until you reach the solution. As a result of
playing these games, players can develop confidence, as being able to overcome the obstacles gives
you a real sense of achievement. Patience, perseverance and overcoming difficulties are three
essential skills in everyday life. As humans we often learn through mistakes, however some
mistakes would be best avoided. Video games can expose you to making decisions in the virtual
world by allowing you to choose your characters' actions and then showing you the results of these
actions. This may help you make more informed choices in the real world.
However beneficial these games may be, there are things to remember. Too much screen time
can isolate you from the real world and if you play multi-player games online, then you need to be
sure who you are playing with. It's also vital to make sure that you are not running up a huge bill in
games that you pay for online.
Links between schools and video games developers have resulted in more games focused on
developing life skills. The not-for-profit organisation, Makerspace, has developed an education
programme based around games in conjunction with Minecraft. The programme claims that 30
minutes of video play in the classroom can improve planning skills, memory and motor skills. As
well as developing life skills, having excellent computer skills could also help you with a future
career. So, play on and learn two valuable lessons at the same time.
(Adapted from Optimise by Malcom Mann and Steve Taylore-Knowles)

Question 44. Which best serves as the title for the passage?
A. A New Era of Game Addiction B. Video Games and Life Skills
C. The Combination of Work and Leisure D. The Importance of Computer Literacy
Question 45. According to paragraph 1, computer skills ______ .
A. play an important role in dealing with daily tasks
B. are easier to be mastered at a very young age

177
C. ensure a potential candidate a steady job
D. are necessary to get a promotion at workplace
Question 46. The word perseverance in paragraph 2 is closest in meaning to ______ .
A. recognition B. admittance C. consistency D. persistence
Question 47. The word They in paragraph 2 refers to ______ .
A. games B. skills C. obstacles D. players
Question 48. The word informed in paragraph 3 can be best replaced by ______ .
A. knowing B. wise C. valuable D. core
Question 49. Which of the following is TRUE according to the passage?
A. Making errors in a game may help players understand their consequences.
B. Overcoming obstacles in a game helps players recognise their weaknesses.
C. Spending excessive time on playing video games can lead to serious ailments.
D. Makerspace has encouraged schools to increase the hours of students playing games.
Question 50. Which of the following can be inferred from the passage?
A. Playing video games in the classroom can be beneficial if there is a time limit.
B. Educational institutions often take little notice of the benefits of playing games.
C. Your addiction to video games can be caused by your preference for living alone.
D. Studying and playing video games simultaneously equip students with vital skills.

**** THE END ****

178

You might also like